SlideShare a Scribd company logo
1 of 80
ĐẠI HOC THÁI NGUYÊN
TRƯ NG ĐẠI HOC KHOA HOC
..................................................
Tải tài liệu tại sividoc.com
Viết đề tài giá sinh viên – ZALO:0973.287.149-TEAMLUANVAN.COM
Nguyen Thị Tuyet Mai
XAP XỈ DIOPHANTINE
VÀ PHÂN SO LIÊN TỤC
TRONG GIẢI
PHƯƠNG TRÌNH PELL
LU N VĂN THẠC SĨ TOÁN HOC
Thái Nguyên - 2017
ĐẠI HOC THÁI NGUYÊN
TRƯ NG ĐẠI HOC KHOA HOC
..................................................
Tải tài liệu tại sividoc.com
Viết đề tài giá sinh viên – ZALO:0973.287.149-TEAMLUANVAN.COM
Nguyen Thị Tuyet Mai
XAP XỈ DIOPHANTINE
VÀ PHÂN SO LIÊN TỤC
TRONG GIẢI
PHƯƠNG TRÌNH PELL
Chuyên ngành: PHƯƠNG PHÁP TOÁN SƠ CAP
Mã so : 60 46 01 13
LU N VĂN THẠC SĨ TOÁN HOC
NGƯ I HƯ NG DȀN KHOA HOC:
TS. NGUYEN ĐÌNH BÌNH
Thái Nguyên - 2017
i
Viết đề tài giá sinh viên – ZALO:0973.287.149-TEAMLUANVAN.COM
Mnc lnc
L I CẢM ƠN iii
M ĐAU i
1 PHƯƠNG TRÌNH PELL 1
1.1. M®t so khái ni m và ket quả ve phương trình Pell . . . . . . . 1
1.1.1. Phương trình Pell Loại I . . . . . . . . . . . . . . . . . 1
1.1.2. Phương trình Pell Loại II . . . . . . . . . . . . . . . . 3
1.1.3. Phương trình Pell với tham so n . . . . . . . . . . . . . 4
1.2. Phân so liên tục - Phân so liên tục tőng quát - Phân so liên
tục đơn giản . . . . . . . . . . . . . . . . . . . . . . . . . . . . 7
1.2.1. M®t trường hợp của phương trình Pell . . . . . . . . . 7
1.2.2. Phân so liên tục ..................................................................... 18
1.3. Bài toán áng dụng.......................................................................... 29
2 XAP XỈ DIOPHANTINE, M R NG PHƯƠNG TRÌNH
PELL VÀ ỨNG DỤNG 35
2.1. Chu kì của phân so liên tục............................................................ 35
2.1.1. Bő đe chính......................................................................... 36
2.1.2. Chu kì phân so liên tục....................................................... 40
2.2. Xap xỉ Diophantine và phân so liên tục đơn giản......................... 46
2.2.1. Phân so liên tục đơn giản của
√
D .......................................46
ii
Viết đề tài giá sinh viên – ZALO:0973.287.149-TEAMLUANVAN.COM
2.2.2. Xap xỉ Diophantine và phân so liên tục đơn giản..............50
2.3. Ve m®t tiêu chuȁn cho sự ton tại nghi m của phương trình Pell 54
2.4. M®t so mở r®ng của xap xỉ Diophantine.......................................55
2.4.1. Tiêu chí vô t.......................................................................55
2.4.2. Bat đȁng thác Liouville ...................................................59
2.4.3. Bat đȁng thác Liouville b c hai .......................................60
2.5. M®t áng dụng giải phương trình Pell âm......................................62
Tài li u tham khảo 72
iii
Viết đề tài giá sinh viên – ZALO:0973.287.149-TEAMLUANVAN.COM
L I CẢM ƠN
Được sự phân công của khoa Toán- Tin, trường Đại hoc Khoa Hoc Thái
Nguyên và sự đong ý của thay giáo hướng dan TS. Nguyen Đình Bình, tôi đã
thực hi n đe tài "Xap xỉ Diophantine và phân so liên tục trong giải phương
trình Pell".
Đe hoàn thành lu n này, tôi xin chân thành cảm ơn Ban Giám Hi u, khoa
Toán - Tin và phòng đào tạo của trường Đại hoc Khoa Hoc Thái Nguyên.
Tôi xin trân trong cảm ơn các thay cô giáo đã t n tình hướng dan, giảng dạy
trong suot quá trình hoc t p, nghiên cáu và rèn luy n ở trường Đại hoc Khoa
Hoc Thái Nguyên.
Xin chân thành cảm ơn thay giáo hướng dan TS. Nguyen Đình Bình đã
t n tình, chu đáo hướng dan tôi hoàn thành lu n văn này. Dù rat b n r®n
trong công vi c, song thay van dành nhieu thời gian và tâm huyet hướng dan,
đ®ng viên, khuyen khích tôi trong quá trình nghiên cáu đe tài.
Cuoi cùng tôi xin chân thành cảm ơn đen gia đình, bạn bè, nhǎng người
không ngàng đ®ng viên, ho trợ tạo moi đieu ki n tot nhat cho tôi trong suot
thời gian hoc t p và nghiên cáu lu n văn.
Tôi xin trân trong cảm ơn!
Thái Nguyên, ngày ... tháng ... năm ...
Tác giả
Nguyen Thị Tuyet Mai
i
Viết đề tài giá sinh viên – ZALO:0973.287.149-TEAMLUANVAN.COM
M ĐAU
1. Lý do chon đe tài
Trong lịch sả phát trien của So hoc, phương trình Pell được biet đen là
m®t phương trình női tieng trong dạng toán ve phương trình nghi m nguyên.
Phương trình Pell được phát minh cách đây 1000 năm ở An Đ® cő đại bởi
Brahmaguta. Trong nhieu năm sau đó, các nhà toán hoc bat đau nghiên cáu
tìm lời giải cho phương trình này. Đen năm 1770, Lagrange đã phát trien lí
thuyet tőng quát ve phương trình dựa trên phân so liên tục. Bên cạnh đó, các
nhà toán hoc lớn như Legendre(1798), É. Borel(1903) cũng quan tâm nghiên
cáu và có nhieu đóng góp cho vi c hoàn thi n và phát trien phương trình
Pell. Ngày nay rat nhieu tài li u nghiên cáu sâu ve phương trình Pell ra đời
như: Computational aspects of number theory( H. Cohen, 2001), The higher
arithmetic (H. Davenport, 2008), Solving the Pell equation (M.J.Jacobson,
Jr and H.C.Williams, 2009) tham khảo trong tài li u [4]. Tuy có rat nhieu
công trình nghiên cáu ve phương trình Pell cũng như phương trình nghi m
nguyên, song đó van còn là m®t ȁn so thách thác các nhà toán hoc cũng như
các bạn trẻ yêu thích môn toán.
Có the nói, phương trình Pell khá phong phú và đa dạng ve lịch sả ra đời,
định nghĩa, trong phương pháp giải và cả áng dụng của nó trong So hoc. Bản
thân nó đóng góp nhieu áng dụng trong vi c giải các bài toán ve So hoc hay
và khó. Nhieu bài toán ve phương trình Pell qua các kì thi Olimpic toán quoc
ii
Viết đề tài giá sinh viên – ZALO:0973.287.149-TEAMLUANVAN.COM
te, khu vực và trong nước ngày càng mới lạ thu hút sự quan tâm cũng như
thách thác trí tu , sáng tạo của moi bạn trẻ. Và đe giải nó không nhǎng can
nam được lí thuyet mà còn can các kĩ năng. Tuy nhiên hi n nay các bạn hoc
sinh, đ c bi t là các bạn hoc sinh lớp chuyên, lớp chon còn biet rat ít ve dạng
phương trình Pell. Đ c bi t, chúng ta có rat ít sách ve phương trình Pell và
áng dụng của nó, chủ yeu là tham khảo các tài li u, bài báo nước ngoài.
Do v y, dưới sự góp ý của thay hướng dan TS. Nguyen Đình Bình, tác giả
chon đe tài “Xap xỉ Diophantine và phân so liên tục trong giải phương trình
Pell”. Do phương trình Pell không còn là đe tài mới nên trong lu n văn tác
giả sě trình bày ngan gon các ket quả và ví dụ ve phương trình Pell cơ bản,
xap xỉ Diophantine và phân so liên tục trong giải phương trình Pell. Đong
thời lu n văn cũng phân tích mở r®ng phương trình và áng dụng của nó. Do
thời gian có hạn và trình đ® còn hạn che nên lu n văn chỉ dàng lại ở vi c
trình bày ket quả nghiên cáu ve xap xỉ Diophantine và phân so liên tục trong
giải phương trình Pell, giới thi u áng dụng giải phương trình Pell âm.
2. Mnc tiêu nghiên cfíu của lu n văn
Mục tiêu của lu n văn là nghiên cáu ve phương trình Pell cơ bản, nghiên
cáu xap xỉ Diophantine, phân so liên tục trong giải phương trình Pell. Đong
thời lu n văn cũng phân tích mở r®ng của phương trình Pell và áng ụng của
nó.
3. Đoi tư ng và phạm vi nghiên cfíu
- Đoi tượng nghiên cáu của lu n văn là phương trình Pell.
- Phạm vi nghiên cáu của lu n văn là giới thi u xap xỉ Diophantine và
phân so liên tục trong giải phương trình Pell, áng dụng giải phương trình
Pell âm.
iii
Viết đề tài giá sinh viên – ZALO:0973.287.149-TEAMLUANVAN.COM
4. Phương pháp nghiên cfíu
- Đoc sách liên quan đen đe tài và tìm kiem tài li u.
- Đoc, hieu và dịch tài li u tà tieng Anh sang tieng Vi t.
- Sả dụng phương pháp tőng quát đe h thong và trình bày các ket quả
chính trong các tài li u tham khảo.
5. Bo cnc lu n văn
Ngoài phan mở đau và ket lu n, n®i dung của lu n văn chia thành 2 chương:
- Chương 1 trình bày m®t so khái ni m và ket quả của phương trình Pell
cơ bản, h thong lí thuyet ve phân so liên tục.
- Chương 2 trình bày ve xap xỉ Diophantine, phân so liên tục đơn giản trong
giải phương trình Pell và áng dụng trong giải phương trình Pell. Chương 2
là chương trong tâm của lu n văn.
1
Viết đề tài giá sinh viên – ZALO:0973.287.149-TEAMLUANVAN.COM
Chương 1
PHƯƠNG TRÌNH PELL
Trong chương này tác giả sě trình bày m®t so khái ni m và ket quả ve
phương trình Pell cơ bản, phân so liên tục. Đong thời tác giả trình bày m®t
so bài t p áng dụng là các bài toán trong các kì thi hoc sinh giỏi các năm
được chon loc. N®i dung chính được tham khảo tà các tài li u tham khảo [1],
[2], [3], [4].
1.1. M t so khái ni m và ket quả ve phương trình Pell
Trong mục này, tác giả sě đưa ra h thong các định nghĩa và định lí ve
công thác nghi m của phương trình Pell cơ bản, cùng m®t so ví dụ có kèm lời
giải cho tàng loại phương trình Pell. N®i dung chính được tham khảo trong
tài li u [1], [3].
1.1.1. Phương trình Pell Loại I
Phương trình Pell loại I là phương trình có dạng:
x2
− Dy2
= 1, ( trong đó D là so nguyên dương). (1.1)
Định lí 1.1.
1. Neu D là so chính phương, D = m2
, m ∈ Z thì (1.1) không có nghi m
2
Viết đề tài giá sinh viên – ZALO:0973.287.149-TEAMLUANVAN.COM
nguyên dương.
2. Neu D là so nguyên âm thì (1.1) không có nghi m nguyên dương.
3. Phương trình (1.1) có nghi m nguyên dương khi và chỉ khi D là so nguyên
dương và không chính phương.
Định lí 1.2. Giả sả (a, b) là nghi m nhỏ nhat của phương trình x2
− Dy2
=
1 nghĩa là b là so nguyên bé nhat đe 1 + Db2
là so chính phương. Xét dãy xn
và yn cho bởi h thác truy hoi sau:
x0 = 1, x1 = a, xn+2 = 2axn+1 − xn, n = 0, 1, ...
y0 = 0, y1 = b, yn+2 = 2ayn+1 − yn, n = 0, 1, ...
(1.2)
Khi đó (xn, yn) là tat cả các nghi m nguyên dương của phương trình Pell
x2
− Dy2
= 1.
Định lí 1.3. Cho phương trình Pell x2
− Dy2
= 1. Goi r là chu kì bieu
dien phân so liên tục
√
D,
pk
qk là phân so đơn giản thá k của
√
D.
Neu r chȁn thì tat cả các nghi m của phương trình Pell là:
x = pkr− 1, y = qkr− 1.
Neu r lẻ thì tat cả các nghi m của phương trình Pell là:
x = p2tr− 1, y = q2tr− 1, t ∈ N∗
.
Lưu ý.
Neu r là so chȁn thì (pr− 1, qr− 1) là nghi m nhỏ nhat.
Neu r là so lẻ thì (p2r− 1, q2r− 1) là nghi m nhỏ nhat.
Ví dn 1.1. Giải phương trình nghi m nguyên:
x2
− 7y2
= 1.
Lời giải. Ta có
√
7 = [2, 1, 1, 1, 4]. Chu kì r = 4 là so chȁn. V y ta có nghi m
nhỏ nhat là (8;3).
3
Viết đề tài giá sinh viên – ZALO:0973.287.149-TEAMLUANVAN.COM
V y tat cả nghi m nguyên dương của phương trình xác định theo công thác:
x0 = 1, x1 = 8, xn+2 = 16xn+1 − xn, n = 0, 1, ...
y0 = 0, y1 = 3, yn+2 = 16yn+1 − yn, n = 0, 1, ...
1.1.2. Phương trình Pell Loại II
Phương trình Pell loại II có dạng:
x2
− Dy2
= − 1, (1.3)
với D là so nguyên dương . Cũng giong như khi xét phương trình Pell loại I,
ở đây ta chỉ quan tâm nghi m nguyên dương của phương trình loại II.
Định lí 1.4. Phương trình Pell loại II không có nghi m nguyên dương khi
D = m2
, m ∈ Z (khi D là so chính phương).
Định lí 1.5. Phương trình Pell loại II không có nghi m nguyên dương khi
D có ước nguyên to p = 4k + 3.
Định lí 1.6. Neu D là so nguyên to thì phương trình Pell loại II có nghi m
nguyên dương khi và chỉ khi D không có dạng 4k + 3.
Định lí 1.7. (Đieu ki n đe phương trình Pell loại II có nghi m)
Goi (a, b) là nghi m nhỏ nhat của phương trình Pell loại I liên ket với phương
trình Pell loại II. Khi đó phương trình Pell loại II có nghi m khi và chỉ khi
h (I) sau:
có nghi m nguyên dương.
a = x2
+ Dy2
b = 2xy
(1.4)
Định lí 1.8. Công thác nghi m của phương trình Pell loại II.
Xét phương trình Pell loại II (1.3). Cùng với đó xét phương trình Pell loại I
(1.1) liên ket với nó. Giả sả (a, b) là nghi m nguyên bé nhat của (1.1). Xét
h phương trình (I). Khi đó lay hai phương trình trong h (I) trà ve với ve
4
Viết đề tài giá sinh viên – ZALO:0973.287.149-TEAMLUANVAN.COM
√
−
2uv = 2
có nghi m và (u, v) là nghi m duy nhat của nó. Xét hai dãy so nguyên dương
{xn}, {yn} sau đây:
x0 = u, x1 = u3
+ 3Duv2
, xn+2 = 2axn+1 − xn, n = 0, 1, ...
y0 = v, y1 = Dv3
+ 3u2
v, yn+2 = 2ayn+1 − yn, n = 0, 1, ...
Khi đó (xn, yn) là ngi m nguyên dương của phương trình Pell loại II.
Định lí 1.9. Phương trình x2
− Dy2
= − 1 có nghi m khi và chỉ khi chukì
r của bieu dien phân so liên tục của D là so lẻ. Trong trường hợp ay các
nghi m của nó là x = p(2tr− r− 1), y = q(2tr− r− 1) với t = 1, 2, 3....
Ví dn 1.2. Xét phương trình x2
− 34y2
= − 1.
Lời giải.
Ta có:
√
34 = [5; 1, 4, 1, 10]. Chu kỳ r = 4 là so chȁn. V y phương trình vô
nghi m.
Ví dn 1.3. Giải phương trình: x2
− 2y2
= − 1.
Lời giải. Phương trình Pell loại (II) liên ket x2
2y2
= 1.
Ta có:
√
2 = [1; 2]. Có chu kỳ r = 1. Có nghi m nhỏ nhat (3; 2).
Xét h phương trình:
u2
+ 2v2
= 3
De dàng thay (u, v) = (1; 1) là nghi m dương bé nhat của nó. Theo công thác
nghi m ở định lí 1.8, thì phương trình Pell loại II x2
− 2y2
= − 1 có nghi m
là:
x0 = 1; x1 = 7; xn+2 = 6xn+1 − xn...
y0 = 1; y1 = 5; yn+2 = 6yn+1 − yn...
1.1.3. Phương trình Pell v i tham so n
Xét phương trình x2
− Dy2
= n, ở đây D là so nguyên dương và khôngchính
phương, còn n là so nguyên. Phương trình này goi là phương trình Pell
5
Viết đề tài giá sinh viên – ZALO:0973.287.149-TEAMLUANVAN.COM
với tham so n. Neu n = 1 ho c n = − 1 thì ta có phương trình Pell loại I và
loại II.
Định lj 1.10. Xét phương trình Pell với tham so n:
x2
− Dy2
= n. (1.5)
Phương trình (1.5) ho c vô nghi m ho c có vô so nghi m.
V y đe tìm ra công thác tőng quát tat cả các nghi m của phương trình Pell
có tham so n ta có các định lí sau:
Định lj 1.11. Xét phương trình Pell với tham so n (1.5). Goi (x0, y0) là
nghi m nguyên dương nhỏ nhat của (1.5). Ta có:
y0 ≤ max{nb2
;
na2
D
},
ở đây (a, b) là nghi m bé nhat của phương trình Pell loại I (1.1) áng với (1.5).
Định lj 1.12. Xét phương trình Pell với tham so n (1.5). Giả sả (1.5) có
nghi m và (α1, β1), (α2, β2), ..., (αm, βm) là tat cả các nghi m của (1.5) thỏa
mãn bat đȁng thác
2 2 − na2
βi ≤ max{nb ,
D
}.
Xét m dãy sau đây. Dãy thá i : {xn,i, yn,i}, với i = 1, m được xác định như
sau:
x0,i = αi, y0,i = βi,
xn+1,i = xn,ia + Dyn,ib,
yn+1, i = xn,ib + yn.ia,
ở đây (a, b) là nghi m bé nhat của phương trình Pell loại I (1.1) áng với (1.5).
Khi đó các dãy nghi m {xn,i; yn,i} sě là tat cả các nghi m nguyên dương của
phương trình Pell với tham so n.
Ví dn 1.4. Giải phương trình Pell x2
− 5y2
= − 4.
Lời giải.
−
6
Viết đề tài giá sinh viên – ZALO:0973.287.149-TEAMLUANVAN.COM
−
≤
2
2
Xét phương trình Pell với tham so n = − 4 sau đây
x2
− 5y2
= − 4. (i)
Phương trình Pell loại I liên ket với nó có dạng:
x2
− 5y2
= 1. (ii)
Phương trình (ii) có nghi m nguyên dương nhỏ nhat là (a, b) = (9, 4). Khi
đó:
max{nb2
;
na2
2
d
} = max{− 4.4 ;
4.92
5
} =
4.81
.
5
So nguyên dương β lớn nhat thỏa mãn β2 4.81
5
là β = 8. Xét phương trình
(i):
Neu y = 1 ⇒ x = 1; y = 2 ⇒ x = 4; y = 3; 4; 7; 8 thì (i) không dan đen x
nguyên; y = 5 ⇒ x = 11.
Như the bang cách thả trực tiep nói trên, ta thay có 3 nghi m (1, 1); (4, 2);
(11, 5) của phương trình (i) thỏa mãn đieu ki n:
β ≤ max{nb ;
na2
D
}
Theo định lý 1.12, phương trình Pell áng với n = − 4 có 3 dãy nghi m:
x0,1 = 1; y0,1 = 1; xn+1,1 = 9xn,1 + 20yn,1; yn+1,1 = 4xn,1 + 9yn,1
x0,2 = 4; y0,2 = 2; xn+1,2 = 9xn,2 + 20yn,2; yn+1,2 = 4xn,2 + 9yn,2
x0,3 = 11; y0,3 = 5; xn+1,3 = 9xn,3 + 20yn,3; yn+1,3 = 4xn,3 + 9yn,3
Ba dãy này vét het tat cả các nghi m của phương trình (i).
Ket lu n: Tác giả đã trình bày m®t cách h thong khái ni m và m®t so ket
quả của phương trình Pell cơ bản. Moi dạng của phương trình Pell, tác giả
đã giới thi u m®t vài ví dụ đe làm sáng tỏ công thác nghi m, tà đó ta có the
mở r®ng các bài toán khó tà bài toán tìm nghi m thông thường.
−
7
Viết đề tài giá sinh viên – ZALO:0973.287.149-TEAMLUANVAN.COM
1
1.2. Phân so liên tnc - Phân so liên tnc tong quát - Phân
so liên tnc đơn giản
Trong mục này tác giả sě trình bày h thong ve lí thuyet phân so liên tục,
cụ the hơn là phân so liên tục tőng quát, phân so liên tục đơn giản liên quan
đen phương trình Pell. N®i dung chính được tham khảo trong tài li u [4].
1.2.1. M t trư ng h p của phương trình Pell
Cho D là m®t so nguyên dương không chính phương.
√
D là so vô tỉ.
Phương trình Diophantine có dạng:
x2
− Dy2
= ± 1, (1.6)
trong đó ȁn x, y ∈ Z được goi là phương trình Pell.
1.2.1.1. Ví dn ve phân so liên tnc đơn giản
Ví dn 1.5. Cho D = a2
b2
+ 2b, ở đó a, b là các so nguyên dương. M®t
nghi m của
x2
− (a2
b2
+ 2b)y2
= 1,
là c p (x, y) = (a2
b + 1, a).
Ta thay dạng khai trien phân so liên tục đơn giản của
√
D là
√
a2b2 + 2b = [ab, a, 2ab].
Tà đó
t =
√
a2b2 + 2b ⇔ t = ab +
1
.
a +
t + ab
Có the xét ví dụ tương tự D = a2
+ 2 (đ t b = 1) và D = b2
+ 2b (đ t a = 1).
Cho a = 1 và b = c − 1, ta cũng được ví dụ D = c2
− 1.
8
Viết đề tài giá sinh viên – ZALO:0973.287.149-TEAMLUANVAN.COM
⇔
⇔
≡
Ví dn 1.6. Đ t D = a2
b2
+ b, ở đó a, b là các so nguyên dương. M®t
nghi m của
x2
− (a2
b2
+ b)y2
= 1,
là c p (x, y) = (a2
b + 1, 2a).
Dạng khai trien phân so liên tục của
√
D là
√
a2b2 + b = [ab, 2a, 2ab].
Tà đó t =
√
a2b2 + b t = ab +
1
.
1
2a +
t + ab
Xét ví dụ tương tự D = b2
+ b (đ t a = 1).
Trường hợp b = 1, D = a2
+ 1 là trường hợp đ c bi t. M®t nghi m nguyên
của phương trình
x2
− (a2
+ 1)y2
= − 1,
là (x, y) = (a, 1). Dạng khai trien phân so liên tục của
√
D là
√
a2 + 1 = [a, 2a].
V y t =
√
a2 + 1 t = a +
1
.
t + a
Ví dn 1.7. Cho a, b là hai so nguyên dương sao cho b2
+ 1 chia 2ab + 1.
Ví dụ b = 2 và a 1 (mod 5). Viet 2ab + 1 = k(b2
+ 1) và đ t D = a2
+ k.
Dạng khai trien phân so liên tục của
√
D là [a, b, b, 2a]. Suy ra t =
√
D
thỏa mãn
1
t = a + 1
b + 1
b +
= [a, b, b, a + t].
a + t
M®t nghi m của phương trình x2
− Dy2
= − 1 là x = ab2
+ a + b, y = b2
+ 1.
Trong trường hợp a = 1 và b = 2 (vì v y k = 1), phân so liên tục có duy nhat
chieu dài chu kì 1 :
√
5 = [1, 2].
9
Viết đề tài giá sinh viên – ZALO:0973.287.149-TEAMLUANVAN.COM
.√
D −
x.
<
1
.
y
. .
Ba ví dụ trên đây là nhǎng trường hợp đ c bi t bởi O.Per-ron và liên quan
đen các trường b c hai thực dạng Richard - Degert.
1.2.1.2. Sfi ton tại của nghi m nguyên
Cho D là so nguyên dương không chính phương. Ta can cháng minh rang
phương trình Pell (1.6) có m®t nghi m không nhỏ (x, y) ∈ Z × Z, đó là m®t
nghi m (± 1, 0).
M nh đe 1.1. Cho D là so nguyên dương không chính phương, ton tại
(x, y) ∈ Z2
với x > 0 và y > 0 sao cho x2
− Dy2
= 1.
Chúng minh.
Bước đau tiên của cháng minh là chỉ ra rang ton tại m®t so nguyên khác không
k sao cho phương trình Pell x2
− Dy2
= k có vô so nghi m (x, y) ∈ Z × Z,
liên h các nghi m nguyên của phương trình Pell với xap xỉ hǎu t x/y của
√
D.
Ta thay
√
D là so vô t , ta suy ra có vô so (x, y) ∈ Z × Z với y > 0 (và do đó
x > 0) thỏa mãn:
y2
Đoi với moi c p (x, y) như v y ta có:
0 < x < y
√
D + 1 < y(
√
D + 1).
Do đó,
0 < |x2
− Dy2
| = |x − y
√
D|.|x + y
√
D| < 2
√
D + 1.
Tà đó có duy nhat so nguyên k /= 0 trong khoảng
− (2
√
D + 1) < k < 2
√
D + 1.
M®t trong so các dạng của chúng là x2
− Dy2
có vô so c p (x, y).
Bước thá hai nh n thay, các t p con của (x, y) (mod k) trong (Z/kZ)2
là
hǎu hạn, là m®t t p con vô hạn E ⊂ Z × Z của các nghi m cho x2
− Dy2
= k
10
Viết đề tài giá sinh viên – ZALO:0973.287.149-TEAMLUANVAN.COM
= (
1 1
2 2
có cùng (x (mod k), y (mod k)). Cho (u1, v1) và (u2, v2) là hai thành phan
riêng bi t trong E. Xác định (x, y) ∈ Q2
bởi
√ u1 + v1
√
D
x + y
Tà u2
− Dv2
= k ta suy ra
D =
u2 + v2
√
D
.
2 2
Do đó,
x + y
√
D
1
u
k 1 + v1
√
D)(u — v2
√
D).
x =
u1u2 − Dv1v2
, y =
− u1v2 + u2v1
.
k k
Tà u1 ≡ u2 (mod k), v1 ≡ v2 (mod k) và
u2
− Dv2
= k, u2
− Dv2
= k.
Ta suy ra
1 1 2 2
u1u2 − Dv1v2 ≡ u2
− Dv2
≡ 0 (mod k),
1 1
và
− u1v2 + u2v1 ≡ − u1v1 + u1v1 ≡ 0 (mod k).
Do đó x, y ∈ Z. Hơn nǎa
x2
− Dy2
= (x − y
√
D)(x + y
√
D)
(u1 − v1
√
D)(u1 + v1
√
D)
= √ √
(u2 − v2 D)(u2 + v2 D)
u2
− Dv2
u2
− Dv2
= 1.
Đây là đieu ki n đe kiem tra y /= 0. Neu y = 0 thì x = ± 1, u1v2 = u2v1, u1u2 −
Dv1v2 = ± 1, và
ku1 = ± u1(u1u2 − Dv1v2) = ± u2(u2
− Dv2
) = ± ku2,
1 1
mở r®ng (u1, u2) = (v1, v2), mâu thuan.
Cuoi cùng, neu x < 0 ta thay x bởi − x, tương tự neu y < 0, ta thay y bởi
2
=
11
Viết đề tài giá sinh viên – ZALO:0973.287.149-TEAMLUANVAN.COM
1 1
1 1
− y. Q
Do v y, khi ta có m®t nghi m nguyên không tam thường (x, y) cho phương
trình Pell, ta sě có vô so nghi m, thu được bang cách xét đieu ki n của
x + y
√
D.
1.2.1.3. Tat cả các nghi m nguyên
M®t trình tự tự nhiên cho các nghi m nguyên dương của phương trình Pell
có the xác định bang nhieu cách: sap xep chúng bang cách tăng giá trị của x
ho c tăng giá trị của y ho c tăng giá trị x + y
√
D - ta de dàng kiem tra rang
đó là các cách như nhau.
M®t nghi m nguyên dương toi thieu (x1, y1) được goi là nghi m cơ bản của
phương trình Pell (1.6). Trong cách tương tự ta có m®t nghi m cơ bản của
phương trình Pell (1.1).
M nh đe 1.2. Giả sả (x1, y1) là nghi m cơ bản cho phương trình Pell
(1.6). Tà đó tat cả các nghi m nguyên dương cho ket quả là dãy (xn, yn)n≥1,
khi đó xn và yn được cho bởi
xn + yn
√
D = (x1 + y1
√
D)n
, (n ∈ Z, n ≥ 1).
Trong các đieu ki n khác, xn và yn được xác định bởi công thác phép truy
toán:
Hơn nǎa,
xn+1 = xnx1 + Dyny1 và yn+1 = x1yn + xny1, (n ≥ 1).
+) Neu x2
− Dy2
= 1, thì (x1, y1) là nghi m cơ bản của phương trình Pell loại
(I) (1.1), và không là nghi m nguyên của phương trình Pell loại (II) (1.3).
+) Neu x2
− Dy2
= − 1, thì (x1, y1) là nghi m cơ bản của phương trình
Pell loại(II) (1.3), và nghi m nguyên của phương trình Pell loại (I)(1.1) là
(x2, y2). T p hợp các nghi m nguyên dương của phương trình Pell loại(I)
(1.1) là {(xn, yn); n ≥ 2 chȁn }, trong đó t p hợp các nghi m nguyên dương
12
Viết đề tài giá sinh viên – ZALO:0973.287.149-TEAMLUANVAN.COM
của phương trình Pell (1.3) là {(xn, yn); n ≥ 1 lẻ }. T p hợp tat cả các nghi m
(x, y) ∈ Z × Z của phương trình Pell (1.6) là t p hợp (± xn, yn)n∈ Z khi xn và yn
cho bởi công thác
xn + yn
√
D = (x1 + y1
√
D)n
, (n ∈ Z).
Nghi m tam thường (1, 0) là (x0, y0), nghi m (− 1, 0) là m®t phan tả xoan
b c hai trong nhóm các đơn vị của vành Z[D].
Chúng minh.
Cho (x, y) là m®t nghi m nguyên dương của phương trình Pell x2
− Dy2
= ± 1.
Kí hi u n ≥ 0 là so nguyên lớn nhat sao cho
(x1 + y1
√
D)n
≤ x + y
√
D.
Vì v y, x + y
√
D < (x1 + y1
√
D)n+1
. Xác định (u, v) ∈ Z × Z bởi
u + v
√
D = (x + y
√
D)(x1 − y1
√
D)n
.
Tà u2
− Dv2
= ± 1 và 1 ≤ u + v
√
D < x1 + y1
√
D, ta suy ra u = 1 và v = 0. Vì v
y x = xn, y = yn. Q
1.2.1.4. Trên nhóm các đơn vị của Z[
√
D]
Cho D là so nguyên dương không chính phương. Vành Z[
√
D] là vành con
của R sinh bởi
√
D. Ánh xạ σ : z = x + y
√
D − → x − y
√
D là tự đȁng cau Galois
của vành này. Quy tac N : Z[
√
D] − → Z xác định bởi N(z) = zσ(z).
Do đó
N(x + y
√
D) = x2
− Dy2
.
Hạn che của N với nhóm của đơn vị Z[
√
D]×
của vành Z[
√
D] là m®t
đong cau tà nhóm nhân Z[
√
D]×
đoi với nhóm của đơn vị Z×
của Z. Tà khi
Z×
= {± 1} ta thay rang:
Z[
√
D]×
= {z ∈ Z[
√
D]; N(z) = ± 1}.
13
Viết đề tài giá sinh viên – ZALO:0973.287.149-TEAMLUANVAN.COM
±
Do v y Z[
√
D]×
không khác t p của x + y
√
D khi (x, y) chạy qua t p cácnghi
m so nguyên của phương trình Pell (1.6).
M nh đe 1.1 nghĩa là Z[
√
D]×
không giảm tại m®t nhóm con 1, trong khi
m nh đe 1.2 đưa ra rat nhieu thông tin chính xác rang nhóm Z[
√
D]×
là m®t
(nhân lên) nhóm Abelian b c 1: đó ton tại đơn vị cơ bản u ∈ Z[
√
D]×
sao
cho
Z[
√
D]×
= {± un
; n ∈ Z}.
Đơn vị cơ bản u > 1 là x1 + y1
√
D, trong đó (x1, y1) là nghi m cơ bản của
phương trình Pell (1.6).
Phương trình Pell (1.6) có nghi m nguyên neu và chỉ neu đơn vị cơ bản có
tiêu chuȁn -1. B c Z[
√
D]×
toi đa 1 là hình ảnh ánh xạ.
Z[
√
D]×
− → R2
z −→ (log |z|, log |z′
|)
là riêng bi t trong R2
và cháa trong giới hạn t1 + t2 = 0 của R2
. Cách cháng
minh này là không thực sự khác bi t so với cách cháng minh ta đưa ra ở
m nh đe 1.2. Cháng minh rang nhóm con rời rạc của R có b c nhỏ hơn ho c
bang 1 dựa vào sự phân chia của Euclid.
Chú ý 1.1. Cho d là so nguyên không hǎu tỉ nhưng không là bình phương
của so nguyên. Thì d không là bình phương của so hǎu tỉ và trường k = Q(
√
d)
là m®t phan mở r®ng b c hai của Q (nghĩa là m®t không gian vectơ Q hai
chieu). M®t yeu to α ∈ k là m®t so nguyên đại so neu và chỉ neu nó thỏa mãn
các đieu ki n tương đương sau:
(i) α là nghi m của m®t đa thác loi với các h so nguyên.
(ii) Đa thác loi không thu gon của α trên Q có h so nguyên.
(iii) Đa thác không thu gon của α trên Z là loi.
(iv) Vành Z[
√
α] là modul Z được tạo ra m®t cách hǎu hạn.
(v) Vành Z[
√
α] được cháa trong m®t vành con của k là phan tả được tạo ra
Z- modul.
14
Viết đề tài giá sinh viên – ZALO:0973.287.149-TEAMLUANVAN.COM
.
.
.
.
√
√
1
4
.
T p Zk của các so nguyên đại so của k là vành
Zk =
Z + Z
√
d
1 +
√
d
neu d ≡ 2 ho c 3 (mod 4)
Z + Z
2
neu d ≡ 1 (mod 4)
Do v y Zk = Z + Zα, trong đó α là m®t trong hai nghi m bat kì của X2
− d
neu d ≡ 2 ho c 3 (mod 4) và m®t trong hai nghi m của đa thác
X2
− X − (d − 1)/4 = (2X − 1)2
− d,
neu d ≡ 1 (mod 4).
Bi t thác Dk của k là bi t thác của vành so nguyên k.
2 0
det
.
= 4d neu d ≡ 2 ho c 3 (mod 4)
Dk = 0 2d
2 1
det = d neu d 1 (mod 4)
. 1 (1 + d)/2 .
Vì v y bi t thác luôn phù hợp với 0 ho c 1 modul 4 và trường b c hai là
k = Q[
√
Dk].
Nhóm các đơn vị của k theo định nghĩa là nhóm các đơn vị Z×
K của vành
Zk. Đoi với d < 0 de dàng kiem tra xem nhóm các đơn vị trong k là nhóm
hǎu hạn của nghi m hợp nhat trong k.
•{1, i, − 1, − i} neu k có bi t thác -4, nghĩa là k = Q(i).
•{1, Q, Q2
, − 1, − Q, − Q2
} neu k có bi t thác -3, trong đó Q là nghi m của X2
+
X + 1. Trường b c hai với bi t thác -3 là k = Q(
√
Q) = Q( − 3) và Q là
nghi m nguyên thủy của phan tả đơn vị.
•{± 1} theo cách khác.
Giả sả d > 0. Nghi m của phan tả đơn vị trong k chỉ là ±1 và nhóm Z×
k của
các đơn vị Zk là m®t Z-mođul của b c 1. Do đó, nó là đȁng cau với {± 1} × Z. Đoi
với d ≡ 2 và d ≡ 3 (mod 4), các đơn vị Z×
k của k là các phan tả x + y Dk
∈ k sao cho (x, y) ∈ Z × Z là m®t nghi m của phương trình Pell
≡
15
Viết đề tài giá sinh viên – ZALO:0973.287.149-TEAMLUANVAN.COM
√
−
y
x2
− Dky2
= ±1. Đoi với d ≡ 1 (mod 4), nhóm các đơn vị Z×
k của k là t p
hợp các phan tả x + y Dk ∈ k sao cho (x, y) ∈ Z × Z là m®t nghi m của phương
trình Pell x2
− Dky2
= ± 4.
1.2.1.5. Ket noi v i xap xỉ hfiu t
Bo đe 1.1. Cho D là so nguyên dương không chính phương. Giả sả x và
y là các so nguyên dương, các đieu ki n sau đây là tương đương:
(i) x2
− Dy2
= 1.
x
(ii) 0 <
y
1
D <
2y2
√
D
.
(iii) 0 <
x √
D <
y
Chúng minh.
1
1
y2
√
D + 1
.
1 2 2
Ta có 2y2
√
D
<
y2
√
D + 1
. Do đó (ii) kéo theo (iii). (i) kéo theo x > Dy ,
do đó x > y
√
D và vì the
x √ 1 1
√
(iii) kéo theo
0 <
y
− D =
y(x + y
√ 1
D)
<
2y2
√
D
.
√ 2
x < y
và
D +
y
√
D
< y
D + ,
y
Do v y
y(x + y
√
D) < 2y2
√
D + 2.
0 < x2
− Dy2
= y(
x
−
√
D)(x + y
√
D) < 2.
Tà đó x2
− Dy2
là m®t so nguyên, nó bang với 1. Q
Bo đe 1.2. Cho D là so nguyên dương không chính phương. Cho x và y
là các so nguyên dương. Các đieu ki n sau là tương đương:
(i) x2
− Dy2
= − 1.
√
−
16
Viết đề tài giá sinh viên – ZALO:0973.287.149-TEAMLUANVAN.COM
— − −
y
y
y
x
√ x 1
(ii) 0 < D −
y
<
2y2
√
D − 1
.
√ x 1
(iii) 0 < D −
y
<
y2
√
D
.
Chúng minh.
1 1
Ta có 2y2
√
D − 1
<
y2
√
D
. Do v y (ii) kéo theo (iii). Đieu ki n (i) kéo theo
y
√
D > x. Ta sả dụng ước tính tam thường
2
√
D > 1 +
1
y2
và viet
x2
= Dy2
1 > Dy2
2
√
D +
1
y2
= (y
√
D
1
)2
.
y
Do v y xy > y2
√
D − 1. Tà (i) suy ra
1 = Dy2
− x2
= (y
√
D − x)(y
√
D + x)
>
√
D −
x
(y2
√
D + xy)
>
√
D −
x
(2y2
√
D − 1).
(iii) kéo theo x < y
√
D và
y(y
√
D + x) < 2y2
√
D.
Do đó
0 < Dy2
− x2
= y(
√
D −
x
)(y
√
D + x) < 2.
Tà đó Dy2
− x2
là so nguyên, nó là 1. Q
H quả 1.1: Cho D là so nguyên dương không chính phương. Cho x và
y là các so nguyên dương. Các đieu ki n sau là tương đương:
(i) x2
− Dy2
= ± 1.
(ii) .
√
D −
y
. <
1
2y2
√
D − 1
.
(iii)
.√
D −
x.
< √
1
.
.
y
. y2 D + 1
17
Viết đề tài giá sinh viên – ZALO:0973.287.149-TEAMLUANVAN.COM
x
√
.
−
Chúng minh.
Neu y > 1 ho c D > 3 ta có 2y2
√
D − 1 > y2
√
D + 1, nghĩa là (ii) kéo theo
m®t phan nhỏ của (iii) và ta có the áp dụng các bő đe 1.1 và 1.2.
Neu D = 2 và y = 1 thì moi đieu ki n (i),(ii) và (iii) thỏa mãn khi và chỉ
khi x = 1. Đieu ki n này bat nguon tà
2 −
√
2 >
1
2
√
2 − 1
>
1
√
2 + 1
>
√
2 − 1.
Neu D = 3 và y = 1 thì moi đieu ki n (i),(ii) và (iii) thỏa mãn khi và chỉ
khi x = 2. Đieu ki n này bat nguon tà
3 −
√
3 >
√
3 − 1 >
1
2
√
3 − 1
>
1
√
3 + 1
> 2 −
√
3.
Q
Bo đe 1.3. Cho D là so nguyên dương không chính phương. Cho x và y
là các so nguyên dương. Thì
.√ 1
Chúng minh.
. D −
y
. >
2y2
√
D + 1
.
Neu x/y <
√
D thì x ≤ y
√
D và tà
1 ≤ Dy2
− x2
= (y
√
D + x)(y
√
D − x) ≤ 2y
√
D(y
√
D − x).
Suy ra √ x 1
D −
y
>
Ta khȁng định neu x/y >
√
D thì 2y2
√
D
.
x 1
y
D >
2y2
√
D + 1
.
Th t vây, ước tính đúng neu x − y
√
D ≥ 1/y, v y ta giả sả rang x − y
√
D <
1/y. Khȁng định của ta sau đó tà
1 ≤ x2
− Dy2
= (x + y
√
D)(x − y
√
D) ≤ (2y
√
D + 1/y)(x − y
√
D).
18
Viết đề tài giá sinh viên – ZALO:0973.287.149-TEAMLUANVAN.COM
0
√
Q
Đieu này cho thay xap xỉ hǎu t x/y đen D, chỉ yeu hơn giới hạn bởi bat
đȁng thác Liouville, sě tạo ra nghi m của phương trình Pell (1.6). Khoảng
cách |
√
D − x/y| không nhỏ hơn 1/(2y2
√
D + 1), nhưng nó có the nhỏ như
1/(2y2
√
D − 1) và cho rang nó chỉ can nhỏ hơn 1/(y2
√
D + 1).
1.2.2. Phân so liên tnc
Trước tiên xét các phân so liên tục tőng quát của bieu mau
b1
a0 +
b2
.
Mà ta bieu dien bởi
a1 +
a2 +
b3
...
a +
b1|
+
b2|
+
b3|
.
|a1 |a2 ...
Ta thu hep trường hợp đ c bi t khi b1 = b2 = ... = 1. Các phân so liên tục
đơn giản có dạng
a +
1|
|a1
+
1|
|a2
+ ... = [a0 , a1 , a2, ...].
1.2.2.1. Phân so liên tnc tong quát
Được bat đau với a0, ..., an, ... và b1, ..., bn, ... là bien đ®c l p. Sau đó, ta sě
chuyen hóa các so nguyên dương (trà a0 là âm). Xem xét 3 phân so hǎu t :
Ta viet chúng như sau:
a0, a0 +
b1
a1
và a0
b1
+
b2
.
a1 +
2
A0
,
A1 và
A2
.
B0 B1 B2
a
0
19
Viết đề tài giá sinh viên – ZALO:0973.287.149-TEAMLUANVAN.COM
!
! ! !
!
!
với
A0 = a0, A1 = a0a1 + b1, A2 = a0a1a2 + a0b2 + a2b1;
B0 = 1, B1 = a1, B2 = a1a2 + b2.
Quan sát thay
A2 = a2A1 + b2A0, B2 = a2B1 + b2B0.
Viet h thác của chúng như sau:
A2
=
A1 A0
B2 B1 B0
a2
.
b2
L p lại quá trình đe thu n ti n làm vi c với ma tr n 2 × 2 và viet
A2 A1
B2 B1
=
A1 A0
B1 B0
a2 1
.
b2 0
Xác định phép quy nạp hai dãy của đa thác với h so hǎu t dương An và
Bn cho n ≥ 3 bởi
An An− 1
!
=
An− 1 An− 2
!
an
!
. (1.7)
An = anAn− 1 + bnAn− 2, Bn = anBn− 1 + bnBn− 2.
Quan h trên giǎ cho n ≥ 2. Nó sě giǎ cho n = 1 neu ta đ t A− 1 = 1 và
B− 1 = 0:
A1 A0
=
B1 B0
! !
và nó sě giǎ cho n = 0 neu ta đ t b0 = 1, A− 2 = 0 và B− 2 = 1:
A0 A− 1
!
=
1 0
!
a0 1
!
B0 B− 1 0 1 b0 0
!
1
Nghĩa là
Bn Bn− 1 Bn− 1 Bn− 2 bn 0
a0 1 a1 1
1 0 b1 0
20
Viết đề tài giá sinh viên – ZALO:0973.287.149-TEAMLUANVAN.COM
0
0
0
0
De thay m®t định nghĩa tương đương với
An An− 1
!
=
a0 1
!
a1 1
!
...
an− 1 1
!
an 1
!
(1.8)
Bn Bn− 1 b0 0 b1 0 bn− 1 0 bn 0
Quan h (1.8) giǎ cho n ≥ − 1, với ket quả rong (cho n = − 1) là ma tr n
đong nhat. Do v y An ∈ Z[a0, ..., an, b1, ..., bn] là đa thác trong 2n + 1 bien
trong khi Bn ∈ Z[a1, ..., an, b2, ..., bn] là đa thác trong 2n − 1 bien.
Bo đe 1.4. Với n ≥ 0,
a +
b1|
+ ... +
bn|
=
An
.
Chúng minh.
|a1 |an Bn
Bang phương pháp quy nạp. Ta kiem tra ket quả với n = 0, n = 1 và n = 2.
Giả sả công thác đúng với n − 1 khi n ≥ 3. Ta viet
a +
b1|
+ ... +
bn−1|
+
bn|
= a +
b1|
+ ... +
bn−1|
.
0
|a1 |an−1 |a
n
|a1 |x
với x = an− 1
nghĩa (1.7)
+
bn
. Ta có giả thiet theo phương pháp quy nạp và theo định
an
a +
b1|
+ ... +
bn−1|
=
An− 1
=
an− 1An− 2 + bn− 1An− 3
.
|a1 |an−1 Bn− 1 an− 1Bn− 2 + bn− 1Bn− 3
Tà đó An− 2, An− 3, Bn− 2 và Bn− 3 không phụ thu®c vào bien an− 1, ta suy ra
a +
b1|
+ ... +
bn−1|
=
xAn− 2 + bn− 1An− 3
.
|a1 |an− 1
Ket quả của tả so bởi an là
xBn− 2 + bn− 1Bn− 3
T
(anan− 1 + bn)An− 2 + anbn− 1An− 3 =
=
an(an− 1An− 2 + bn− 1An− 3) + bnAn− 2
anAn− 1 + bnAn− 2 = An.
ương tự, ket quả của mau so bởi an là
(anan− 1 + bn)Bn− 2 + anbn− 1Bn− 3 = an(an− 1Bn− 2 + bn− 1Bn− 3) + bnBn− 2
= anBn− 1 + bnBn− 2 = Bn.
21
Viết đề tài giá sinh viên – ZALO:0973.287.149-TEAMLUANVAN.COM
Σ
Σ
0
Q
Tà (1.8), lay định thác, ta suy ra cho n ≥ − 1,
n+1
AnBn − An− 1Bn = (− 1) b0...bn, (1.9)
có the viet với n ≥ 1,
An An− 1 (− 1)n+1
b0...bn
Bn
−
B
=
n− 1 Bn− 1Bn
. (1.10)
Thêm tőng như trên ta được, với n ≥ 0,
n k+1
An
= A0
n
+
(− 1) b0...bk
. (1.11)
k=1
Bk− 1Bk
Bây giờ, ta thay a0, a1, ... và b1, b2, ... so nguyên, tat cả đeu lớn hơn ho c bang
1, ngoại trà a0 có the nhỏ hơn ho c bang 0. Ta bieu thị bởi pn giá trị của An,
tương tự ta bieu thị bởi qn giá trị Bn cho các giá trị đ c bi t này. Do v y pn
và qn là so nguyên với qn > 0 cho n ≥ 0. Ket quả của bő đe 1.4 là:
pn
= a +
b1|
+ ... +
bn|
cho n ≥ 0.
qn
Ta suy ra tà (1.7)
|a1 |an
pn = anpn− 1 + bnpn− 2, qn = anqn− 1 + bnqn− 2, cho n ≥ 0.
và tà (1.9),
pnqn− 1 − pn− 1qn = (− 1)n+1
b0...bn, cho n ≥ − 1.
Có the viet được, cho n ≥ 1,
pn pn− 1 (− 1)n+1
b0...bn
qn
−
q
=
n− 1 qn− 1qn
. (1.12)
Thêm tőng sả dụng (1.11)) ta được tőng:
n k+1
pn
= a0
n
+
(− 1) b0...bk
. (1.13)
k=1
qk− 1qk
B
q
22
Viết đề tài giá sinh viên – ZALO:0973.287.149-TEAMLUANVAN.COM
Σ
q
n
qn− 1 qn− 1qn 2n−n0 bnbn−1...bn +1qn qn −1
0 0
Nhac lại cho các so thực a, b, c, d với b, d là các so dương, ta có:
a c a
b
<
d
⇒
b
<
a + c
<
b + d
c
. (1.14)
d
Tà đó an và bn là so dương với n ≥ 0, ta suy ra rang cho n ≥ 2, so hǎu t
pn
=
anpn− 1 + bnpn− 2
qn anqn− 1 + bnqn− 2
nam giǎa
pn− 1
qn− 1
và
pn− 2
. Vì v y, ta có
qn− 2
p2
<
p4
q2 q4
< ... <
p2n
q2n
< ... <
p2m+1
q2m+1
<
p3
q3
<
p1
. (1.15)
q1
Tà (1.12) suy ra, cho n ≥ 3, thì qn− 1 > qn− 2, do v y
qn > (an + bn)qn− 2.
Suy lu n trên là hợp l mà không có bat kì hạn che nào, giờ ta giả sả an ≥ bn cho
tat cả n đủ lớn, nói n ≥ no. Sau đó cho n ≥ n0, sả dụng qn > 2bnqn− 2, ta được
.
pn
−
pn− 1
. =
b0...bn bn...b0
<
0 0 0
=
bn0...b0
,
2n−n0 qn qn −1
và ve phải tien dan đen 0 khi n tien đen vô cùng. Do đó chuoi (pn/qn)n≥0 có
giới hạn, ta bieu dien bởi
b1| bn− 1| bn|
x = a0 + + ... + + + ...
|a1
Tà (1.13), x được đưa ra bởi m®t dãy:
∞
|an−1
k+1
|an
x = a0 +
(− 1) b0...bk
.
k=1
qk− 1qk
Bây giờ ta cháng minh rang x là so vô t . Xác định với n ≥ 0,
xn = an
+
bn+1|
+ ...
|an+1
23
Viết đề tài giá sinh viên – ZALO:0973.287.149-TEAMLUANVAN.COM
0
sao cho x = x0 và cho moi n ≥ 0,
xn = an +
bn+1
, x
xn+1
n+1
bn+1
= ,
xn − an
và an < xn < an + 1. Do v y, cho n ≥ 0, xn là so hǎu t neu và chỉ neu xn+1là
so hǎu t , và do đó neu x hǎu t thì tat cả xn với n ≥ 0 là so hǎu t . Giảsả x
là so hǎu t . Xét các so hǎu t xn với n ≥ n0 và chon m®t giá trị của n mà
neu mau so v của xn là toi thieu, nói xn = u/v. Tà
xn+1
bn+1
=
xn − an
bn+1v
=
u − anv
, với 0 < u − an v < v,
sau đó xn+1 có mau so nhỏ hơn v, mâu thuan. Vì v y x là so vô t .
Ngược lại, cho m®t so vô t x và m®t chuoi b1, b2, ... của so nguyên dương,
có duy nhat m®t so nguyên dương a0 và duy nhat dãy a1, ..., an, ... của so
nguyên dương thỏa mãn an ≥ bn cho moi n ≥ 1, sao cho:
x = a0 +
b1|
+ ... +
bn−1|
+
bn|
+ ...
|a1 |an−1 |an
Thât v y, nghi m duy nhat được đưa vào theo quy nạp như sau: a0 =
[x♩ , x1 = b1/{x}, và m®t lan nǎa a0, ..., an− 1 và x1, ..., xn đã được biet, sau đó
an và xn+1 được cho bởi
an = [xn♩ , xn+1 = bn+1/{xn},
sao cho n ≥ 1, ta có 0 < xn − an < 1 và
x = a0 +
b1|
+ ... +
bn−1|
+
bn|
.
Đây là đieu cháng minh.
|a1 |an−1 |xn
M nh đe 1.3. Cho so nguyên a0 và hai chuoi a0, a1, ... và b1, b2, ... của so
nguyên dương với an ≥ bn cho moi n đủ lớn, phân so liên tục vô hạn
a +
b1|
+ ... +
bn−1|
+
bn|
+ ...
|a1 |an−1 |an
24
Viết đề tài giá sinh viên – ZALO:0973.287.149-TEAMLUANVAN.COM
0
ton tại và là so vô t .
Ngược lại, cho m®t so x vô t và m®t chuoi b1, b2, ... của so nguyên dương,
có duy nhat so nguyên dương a0 ∈ Z và duy nhat chuoi a1, ..., an, ... của so
nguyên dương thỏa mãn an ≥ bn cho moi n ≥ 1, sao cho
b1| bn− 1| bn|
x = a0 + + ... + + + ...
|a1 |an−1 |an
Sau đây là ví dụ khác ve chuoi so nguyên:
√
1
= 1 +
2|
+
4|
+
6|
+
8|
+ ... = 1541494082...
e − 1 |3 |5 |7 |9
1
=
1|
+
2|
+
3|
+
4|
+ ... = 0581976706...
e − 1 |1 |2 |3 |4
Chú ý 1.2. M®t bien the của thu t toán các phân so liên tục đơn giản tiep
theo. Cho hai dãy (an)n≥0 và (bn)n≥0 của các phan tả trong m®t trường K và
m®t phan tả x trong K, m®t định nghĩa của chuoi (có the hǎu hạn)(xn)n≥1
của các phan tả trong K như sau: neu x = a0, chuoi rong. Neu không thì
x1 được xác định bởi x = a0 + b1/x1. Bang quy nạp, moi x1, ..., xn được xác
định, có hai trường hợp:
• Neu xn = an, thu t toán dàng lại.
• Cách khác, xn+1 được xác định bởi xn+1 =
bn+1
xn − an
. Do đó
xn = an +
bn+1
.
xn+1
Neu thu t toán không dàng lại, thì cho bat kì n ≥ 1, có
b1| bn− 1| bn|
x = a0 + + ... + +
|a1 |an−1 |an
Trong trường hợp đ c bi t khi a0 = a1 = ... = b1 = b2 = ... = 1, t p của x
sao cho thu t toán dàng sau hǎu hạn bước là t p (Fn+1/Fn)n≥1 của thương
của các so Fibonacci liên tiep. Trong trường hợp đ c bi t này, giới hạn của
a +
b1|
+ ... +
bn−1|
+
bn|
|a1 |an−1 |an
25
Viết đề tài giá sinh viên – ZALO:0973.287.149-TEAMLUANVAN.COM
!
1 0
!
1 0
là tỉ so Golden, với x đ®c l p.
1.2.2.2. Phân so liên tnc đơn giản
Ta đã hạn che suy lu n của mục 1.2.2.1 với trường hợp b1 = b2 = ... =
bn = ... = 1. Ta giǎ các kí hi u An và Bn mà đa thác trong Z[a0, a1, ..., an]
và Z[a1, ..., an] tương áng và khi thay đői đen so nguyên a0, a1, ..., an, ... với
an ≥ 1, cho n ≥ 1 ta sả dụng kí hi u pn và qn cho giá trị của An và Bn. Các moi
quan h trong (1.7), với n ≥ 0,
An An− 1
Bn Bn− 1
=
An− 1 An− 2
Bn− 1 Bn− 2
!
an 1
!
. (1.16)
Trong khi (1.8) trở thành, cho n ≥ − 1,
An An− 1
!
=
ao 1
!
a1 1
!
...
an− 1 1
!
an 1
!
. (1.17)
Bn Bn− 1 1 0 1 0 1 0 1 0
Tà bő đe 1.4, suy ra với n ≥ 0,
[a0, ..., an
] =
An
.
Bn
Lay định thác trong (1.17), ta suy ra trường hợp đ c bi t của (1.9):
AnBn− 1 − An− 1Bn = (− 1)n+1
.
Sự thay đői các moi quan h này với các giá trị tách rời của a0, a1, a2, ...,
pn pn− 1
qn qn− 1
=
pn− 1 pn− 2
qn− 1 qn− 2
!
an 1
!
với n ≥ 0, (1.18)
pn pn− 1
!
=
ao 1
!
a1 1
!
...
an− 1 1
!
an 1
!
với n ≥ − 1,
qn qn− 1 1 0 1 0 1 0 1 0
[a0, ..., an
] =
pn
qn
với n ≥ 0, (1.19)
26
Viết đề tài giá sinh viên – ZALO:0973.287.149-TEAMLUANVAN.COM
−
−
!
!
và
pnqn− 1 − pn− 1qn = (− 1)n+1
với n ≥ − 1. (1.20)
Tà (1.20) sau đó với n ≥ 0, phân so pn/qn là thap nhat: gcd(pn, qn) = 1. Qua
(1.19), với n ≥ − 1,
pn qn
!
=
an 1
!
an− 1 1
!
...
a1 1
!
a0 1
!
,
pn− 1 qn− 1 1 0 1 0
tà đó ta suy ra, với n ≥ 1,
1 0 1 0
[an, ..., a0
pn
] =
pn− 1
và [an , ..., a0
qn
] = .
qn− 1
Bo đe 1.5. Với n ≥ 0,
pnqn− 2 − pn− 2qn = (− 1)n
an.
Chúng minh.
Ta nhân cả hai m t của (1.18) ở bên trái bởi sự nghịch đảo của ma tr n:
đó là:
pn− 1 pn− 2
,
qn− 1 qn− 2
Ta nh n được
( 1)n qn− 2 − pn− 2
.
− qn− 1 pn− 1
( 1)n pnqn− 2 − pn− 2qn pn− 1qn− 2 − pn− 2qn− 1
− pnqn− 1 + pn− 1qn 0
!
=
an 1
!
.
Q
1.2.2.2.1. Phân so liên tnc đơn giản của so hfiu t
Cho u0 và u1 là hai so nguyên với u1 là so dương. Bước đau tiên trong
thu t toán Euclid đe tìm gcd của u0 và u1 bao gom trong vi c chia uo bởi u1.
u0 = a0u1 + u2,
1 0
27
Viết đề tài giá sinh viên – ZALO:0973.287.149-TEAMLUANVAN.COM
với a0 ∈ Z và 0 ≤ u2 < u1. Nghĩa là:
u0
= a0
1
+
u2
.
u1
Với giá trị đe chia so hǎu t x0 = u0/u1 với thương a0 và phan còn lại
u2/u1 < 1. Thu t toán này tiep tục với
um = amum+1 + um+2.
Trong đó am là phan không tách rời của xm = um/um+1 và 0 ≤ um+2 < um+1,
cho đen khi ul+2 là 0, trong trường hợp các thu t toán dàng lại với
ul = alul + 1.
Tà đó gcd của um và um+1 giong như gcd của um+1 và um+2, sau gcd của
u0 và u1 là ul+1. Đây là cách chính qui đe khai trien phân so liên tục x0 =
[a0, a1, ..., al], trong đó l = 0 trong trường hợp x0 là so nguyên, trong khi
al ≥ 2 neu x0 là so hǎu t không là so nguyên.
M nh đe 1.4. Bat kì m®t phân so liên tục hǎu hạn m®t phan
[a0, a1, ..., an],
trong đó a0, a1, ..., an là so hǎu t với ai ≥ 2 cho 1 ≤ i ≤ n và n ≥ 0, the hi n m®t
so hǎu t . Ngược lại, bat kì so hǎu t x có hai đại di n như m®t phânso
liên tục, đau tiên, đưa bởi thu t toán Euclid, là
x = [a0, a1, ..., an],
và thá hai là
x = [a0, a1, ..., an− 1, an − 1, 1].
Neu x ∈ Z thì n = 0 và hai phân so liên tục đơn giản đại di n của x là [x]
và [x − 1, 1], trong khi neu x không là so nguyên thì n ≥ 1 và an ≥ 2.
u
28
Viết đề tài giá sinh viên – ZALO:0973.287.149-TEAMLUANVAN.COM
1.2.2.2.2. Phân so liên tnc vô hạn đơn giản của so vô t
Cho so nguyên a0 và m®t dãy vô hạn các so nguyên dương a1, a2, ..., phân
so liên tục
[a0, a1, ..., an, ...],
đại di n cho so vô t . Ngược lại, đưa so vô t x, có bieu dien duy nhat của x
như m®t phân so liên tục vô hạn đơn giản
x = [a0, a1, ..., an, ...].
Các định nghĩa. Các so an là thương riêng, so hǎu t
là các h®i tụ và các so
pn
= [a0
n
, a1 , ..., an],
xn = [an, an+1, ...],
là tỉ so hoàn chỉnh.
Tà các định nghĩa này, ta suy ra, với n ≥ 0,
x = [a0, a1, ..., an , xn+1
] =
xn+1pn + pn− 1
. (1.21)
xn+1qn + qn− 1
Bo đe 1.6. Với n ≥ 0,
(− 1)n
Chúng minh.
Tà (1.21) suy ra
qnx − pn =
xn+1qn
.
+ qn− 1
qn xn+1pn + pn− 1 pn (− 1)n
x −
p
= xn+1qn + qn− 1 —
qn
=
(xn+1qn + qn− 1
.
)qn
Q
H quả 1.2. Cho n ≥ 0,
1 1
qn+1 + qn
< |qnx − pn| < .
qn+1
q
n
29
Viết đề tài giá sinh viên – ZALO:0973.287.149-TEAMLUANVAN.COM
n a q
Chúng minh.
Tà an+1 là m®t phan không thieu của xn+1, ta có
an+1 < xn+1 < an+1 + 1.
Tà qn+1 = an+1qn + qn− 1, ta suy ra
qn+1 < xn+1qn + qn− 1 < an+1qn + qn− 1 + qn = qn+1 + qn.
Q
Đ c bi t, tà xn+1 > an+1 và qn− 1 > 0>, m®t suy lu n tà bő đe 1.6:
1
(an+1 + 2) + q2
< |x
pn
—
qn
|
1
< 2
. (1.22)
n+1 n
Do đó bat kì h®i tụ p/q của x thỏa mãn |x − p/q| < 1/q2
. Hơn nǎa neu an+1
lớn, thì xap xỉ pn/qn là de nh n. Do đó, phan lớn thương so tot cho xap xỉ
hǎu t bởi cat bỏ dạng khai trien phân so liên tục ngay trước khi đưa ra m®t
phan thương nhat định.
đây, tác giả đã đưa ra tóm tat lí thuyet ve phân so liên tục, đong thời
áng dụng lí thuyet phân so liên tục trong giải phương trình Pell khi ket hợp
với xap xỉ hǎu t .
1.3. Bài toán fíng dnng
Trong mục này, tác giả trình bày m®t so bài t p sưu t p được qua m®t
so kì thi hoc sinh giỏi của các năm kèm theo lời giải chi tiet. N®i dung được
tham khảo trong tài li u [2], [3].
Bài toán 1.3. (CANADA). Cho hai dãy so {xn} và {yn} xác định như
sau:
x0 = 1; x1 = 2; xn+1 = 4xn − xn− 1;y0
= 1; y1 = 2; yn+1 = 4yn − yn− 1.
Cháng minh rang với moi so nguyên n ta có: y2
= 3x2
+ 1.
n n
Lời giải:
30
Viết đề tài giá sinh viên – ZALO:0973.287.149-TEAMLUANVAN.COM
n n
Xét phương trình Pell loại I :X2
− 3Y 2
= 1. Phương trình này có nghi m nhỏ
nhat là (2; 1) nên tat cả các nghi m của phương trình là (Xn; Yn) sao cho:
X0 = 1; X1 = 2; Xn+1 = 4Xn − Xn− 1;
Y0 = 1; Y1 = 2; Yn+1 = 4Yn − Yn− 1.
Do đó Xn = yn;Yn = xn hay (xn; yn) là nghi m của phương trình Pell loại I
trên. V y y2
= 3x2
+ 1. Q
Bài toán 1.4.(VMO 1999) Cho hai dãy so {xn} và {yn} xác định như sau:
x0 = 1; x1 = 4; xn+2 = 3xn+1 − xn;
y0 = 1; y1 = 2; yn+2 = 3yn+1 − yn.
Giả sả a, b là các so nguyên dương thỏa mãn a2
− 5b2
+ 4 = 0. Cháng minh
rang ton tại so tự nhiên k đe xk = a, yk = b.
Lời giải:
Xét phương trình Pell
x2
− 5y2
= − 4. (i)
Như ta đã biet , ba dãy so sau vét het các nghi m của phương trình (i):
x0,1 = 1; y0,1 = 1; xn+1,1 = 9xn,1 + 20yn,1; yn+1,n = 4n,1 + 9n,1,
x0,2 = 1; y0,2 = 1; xn+1,2 = 9xn,2 + 20yn,2; yn+2,n = 4n,2 + 9n,2,
x0,3 = 1; y0,3 = 1; xn+1,3 = 9xn,3 + 20yn,3; yn+3,n = 4n,3 + 9n,3.
Ta cháng minh (xn, yn) cũng vét het tat cả các nghi m nguyên dương của (i).
Với moi so tự nhiên n thì n = 3m +r với r = 0; 1; 2. Ta cháng minh (xn, yn) =
(xm,r+1; ym,r+1). Ta có:
(x0, y0) = (x0,1; y0,1 = (1; 1);
(x1, y1) = (x0,2; y0,2 = (4; 2);
(x2, y2) = (x0,3; y0,3 = (11; 5).
31
Viết đề tài giá sinh viên – ZALO:0973.287.149-TEAMLUANVAN.COM
−
2
√
x
!
Phương trình đ c trưng của dãy {xn} và {yn} là
X2
3X + 1 = 0, có hai nghi m X =
3 ±
√
5
.
2
Nên:
xn = x3m+r = α
3 −
√
5
!3m+r
3 +
√
5
3m+r
+ β
3 −
√
5
!r
(9 − 4
√
5)m
+ β
3 +
√
5
!r
(9 + 4
√
5)m
Đ t um = x3m+r Ta có dãy {um} có phương trình đ c trưng có 2 nghi m là
9 ± 4 5 nên:
Suy ra:
Tương tự:
um+1 = 18um − um− 1.
x3(m+1)+r = 18x3m+r − x3(m− 1)+r. (*)
y3(m+1)+r = 18y3m+r − y3(m− 1)+r. (**)
Vi c còn lại ta cháng minh {xm,i; ym,i} cũng thỏa mãn (*) và (**) với i =
1; 2; 3. Ta có:
xm+1,i = 9xm,i + 20ym,i
ym+1,i = 4xm,i + 4ym,i
⇒
y =
xm+1,i − 9xm, i (a)
y
m,i
m− 1,i
20
= m,i − 9xm− 1,i
20
(b)
2
2
2
ym,i = 4m− 1,i + 9ym− 1,i (c)
= α
32
Viết đề tài giá sinh viên – ZALO:0973.287.149-TEAMLUANVAN.COM
0
}
0
≤
0 0
The (a), (b) vào (c) ta suy ra: xm+1,i = 18xm,i − xm− 1,i và ym+1,i = 18ym,i −
ym− 1,i. Như v y {xn} ;{yn} vét het tat cả các nghi m của phương trình (i).
Do đó luôn ton tại k đe {xk = a} ;{yk = b}. Q
Bài toán 1.5. (VMO 2012) Xét các so tự nhiên lẻ a, b mà a là ước so của
b2
+ 2 và b là ước so của a2
+ 2. Cháng minh rang a là các so hạng của dãy
so tự nhiên (vn) xác định bởi v1 = v2 = 1 và vn = 4vn− 1 − vn − 2, n ≥ 2.
Lời giải:
Giả sả (a, b) là c p so tự nhiên mà a là ước so của b2
+ 2 và b là ước so của
a2
+ 2. Trước het ta cháng minh (a, b) = 1.
Th t v y, đ t d = (a, b) thì d là ước của a và a là ước của b2
+ 2 nên d là
ước của b2
+ 2 suy ra d là ước của 2. Mà a, b lẻ nên d lẻ suy ra d = 1. Xét so
N = a2
+ b2
+ 2 thì do a2
+ 2 chia het cho b nên N chia het cho b. V y ton
tại so nguyên dương k sao cho
a2
+ b2
+ 2 = kab. (i)
Tiep theo ta cháng minh với k = 4.
Th t v y, đ t A = a + b|(a, b) ∈ N∗2
, a2
+ b2
+ 2 = kab . Theo giả sả ở trên
thì A /= ∅ . Do tính sap thá tự tot của N, A có phan tả nhỏ nhat. Giả sả a0, b
0
là c p so thỏa mãn đieu ki n (i) với a0 + b0 nhỏ nhat.
Không mat tính tőng quát, có the giả sả a0 ≥ b0. Xét phương trình a2
+
kb0a + b2
= 0 có nghi m a0. Theo định lí Vi-ét thì phương trình trên có m®t
nghi m nǎa là a1kb0 − a0 =
b2
+ 2
. Theo công thác nghi m thì rõ ràng a1
a0
nguyên dương. Như v y (a1, b0) cũng là nghi m của phương trình (i). Do tính
nhỏ nhat của a0 + b0,ta có a0 + b0 ≤ a1 + b0. Tác là a0 ≤ kb0 − a0 suy ra
a0 k
b0 2
Ta có : a2
+ b2
+ 2 = ka0b0 suy ra
a0
+
b0
+
2 = k. (ii)
b0 a0 a0b0
33
Viết đề tài giá sinh viên – ZALO:0973.287.149-TEAMLUANVAN.COM
2
1
0 0
Do
a0
<
k
b0 2
và a0 ≥ b0 ≥ 1 nên tà đây ta có k <
k
+ 1 + 2, suy ra k ≤ 6.
M t khác áp dụng bat đȁng thác AM-GM, a2
+ b2
≥ 2a0b0. Nên k ≥ 2.
Neu k /= 4 thì (a0, b0) /= (1, 1) do đó a0b0 ≥ 2. Lại dùng (ii) đe đánh giá,
ta có k < + 1 + 2, suy ra k ≤ 4. V y các giá trị k = 5, 6 bị loại.
2
Neu k = 3 thì do a2
+ b2
+ 2 = 3a0b0, nên suy ra a2
+ b2
+ 2 chia het cho
0 0 0 0
3, suy ra m®t trong hai so a0 và b0 chia het cho 3, so còn lại không chia het
cho 3.
Neu b0 = 1 thì a0 chia het cho 3, khi đó ve trái không chia het cho 9 còn ve
phải chia het cho 9, mau thuan.
V y b0 > 1. Tà đó suy ra a0b0 ≥ 6. Lại sả dụng (ii) đe đánh giá ,ta có
k 2 3
k ≤
2
+ 1 +
6
⇒ k <
8
. Mà k nguyên suy ra k ≤ 2, mau thuan. Như v y ta
cháng minh được neu a, b là các so tự nhiên lẻ thỏa mãn đieu ki n đe bài thì:
a2
+ b2
+ 2 = 4ab. (iii)
0 0
Đ t ȁn phụ z = a − 2b. Phương trình (iii) trở thành:
z2
− 3b2
= − 2. (iv)
Giải phương trình Pell (iv) với tham so n = − 2, ta cháng minh được hoàn
toàn bài toán. Q
Nh n xét: Nhǎng bài toán trên là các bài toán được sưu tam tà các đe thi
hoc sinh giỏi toán trong nước và ngoài nước, có kèm theo lời giải giúp bạn
đoc thay được nhǎng áng dụng của phương trình Pell. Qua đó bạn đoc cũng
thay được phương trình Pell nói riêng cũng như phương trình nghi m nguyên
nói chung không có quy tac giải tőng quát ho c chỉ có với nhǎng dạng đơn
giản.
Bài t p đe nghị
34
Viết đề tài giá sinh viên – ZALO:0973.287.149-TEAMLUANVAN.COM
2xy = zu
n
Bài 1.6. (IMO Shorthist) Xét h phương trình
x + y = z + u
Tìm giá trị lớn nhat của hang so thực m sao cho m ≤
nguyên dương (x, y, z, u) của h mà x ≥ y.
x
với moi nghi m
y
Bài 1.7. Cháng minh rang neu 5x2
+ 4 ho c 5x2
− 4 là so chính phương khi
và chỉ khi x là so hạng của dãy Fibonacci.
Bài 1.8. (Bulgaria 1999). Cháng minh rang phương trình
x2
+ y2
+ z2
+ t2
= 1999
có vô so nghi m ngyên dương.
Bài 1.9. (IMO Shorthist) Cháng minh rang ton tại hai dãy so nguyên dương
tăng (an), (bn) sao cho an(an + 1) là ước của b2
+ 1 với moi n.
Ket lu n:
Trong chương 1, tác giả đã trình bày m®t cách h thong m®t so khái ni m và
ket quả của phương trình Pell cơ bản, h thong lí thuyet ve phân so liên tục
và m®t so bài toán áng dụng của phương trình Pell là các bài toán trong các
kì thi hoc sinh giỏi trong nước và ngoài nước.
35
Viết đề tài giá sinh viên – ZALO:0973.287.149-TEAMLUANVAN.COM
Chương 2
XAP XỈ DIOPHANTINE, M
R NG PHƯƠNG TRÌNH PELL VÀ
ỨNG DỤNG
Trong chương này tác giả sě trình bày ve h thong các bő đe, h quả, m nh
đe ve xap xỉ Diophantine, phân so liên tục trong giải phương trình Pell và
áng dụng của nó. N®i dung được tác giả tham khảo trong tài li u [4], [5].
2.1. Chu kì của phân so liên tnc
Trong mục này tác giả sě trình bày bő đe chính nham ho trợ trong vi c
cháng minh các m nh đe, h quả có trong các mục đe làm sáng tỏ n®i dung
chính của chương: các van đe ve xap xỉ Diophantine và phân so liên tục, mở
r®ng của xap xỉ Diophantine. N®i dung chính được tham khảo trong tài li u
[4].
36
Viết đề tài giá sinh viên – ZALO:0973.287.149-TEAMLUANVAN.COM
c d 1 0 1 0 1 0
2.1.1. Bo đe chính
Bo đe 2.1. Cho ϵ = ± 1 và a, b, c, d là so nguyên thỏa mãn:
ad − bc = ϵ,
và d ≥ 1 thì có duy nhat m®t dãy hǎu hạn của các so nguyên a0, ..., as với
s ≥ 1 và a1, ..., as− 1 dương sao cho:
a b
!
=
a0 1
!
a1 1
!
...
as 1
!
. (2.1)
Các so nguyên này được đ c trưng bởi:
b
d
= [a0, a1 , ..., a
c
s− 1],
d
= [as, ..., a1 ], (− 1)s+1
= ϵ. (2.2)
Ví dn 2.1. Khi d = 1, cho b và c là các so nguyên,
bc + 1 b
!
=
b 1
!
c 1
!
,
và
bc − 1 b
!
=
b − 1 1
!
1 1
!
c − 1 1
!
.
c 1
Chúng minh bő đe 2.1.
1 0 1 0 1 0
Với tính duy nhat: Neu a0, .., as thỏa mãn các ket lu n của bő đe 2.1, thì
bang cách sả dụng (2.1), ta thay
b c
d
= [a0, a1, ..., as− 1],
d
= [as, ..., a1].
Tiep theo tính định thác ta có được
(− 1)s+1
= ϵ.
b c
Theo sự co định đȁng thác cuoi ve tính chȁn lẻ của s và moi so hǎu tỉ , có
d d
m®t phan khai trien phân so liên tục mà chieu dài đưa ra tính chȁn lẻ (theo
c 1 1 0 1 0
37
Viết đề tài giá sinh viên – ZALO:0973.287.149-TEAMLUANVAN.COM
−
m nh đe 1.4). Đieu này cháng tỏ tính duy nhat của sự phân tích thành nhân
tả khi nó ton tại.
c
Với tính ton tại. Ta xét dạng khai trien phân so liên tục đơn giản của
c d
với chieu dài của tính chȁn lẻ đưa ra bởi công thác (2.2) là
b d
= [as, .., a1].
Cho a0 là so nguyên sao cho khoảng cách giǎa
d
và [a0, a1, ..., as− 1] là nhỏ
1 ′ ′ ′ ′
hơn ho c bang . Xác định a , b , c , d
2
bởi
a′ b′ !
=
a0 1
!
a1 1
!
...
as 1
!
.
Ta có
d′
> 0, a′
d′
b′
c′
= ϵ,
c′
d′
= [as, ..., a
c
1] =
d
.
Tà gcd(c, d) = gcd(c′
, d′
) = 1,
c
d
=
c′
, d > 0, d′
> 0 ta suy ra c′
= c, d′
= d.
d′
Tà sự bình đȁng giǎa các định thác, ta suy ra a′
= a + kc, b′
= b + kd với
m®t so k ∈ Z và tà
b′
b
d′
−
d
= k.
V y k = 0, (a′
, b′
, c′
, d′
) = (a, b, c, d). Do v y (2.1) được cháng minh. Q
H quả 2.1. Giả sả các giả thiet của bő đe 2.1 là thỏa mãn
(a)Neu c > d thì as ≥ 1 và
(b)Neu b > d thì a0 ≥ 1 và
a
c
= [a0, a1, ..., as].
a
b
= [as, ..., a1, a0].
Ví dụ sau cho thay giả thiet của h quả là không thàa
1 b
!
=
b 1
!
0 1
!
0 1
c′ d′
1 0 1 0 1 0
1 0 1 0
38
Viết đề tài giá sinh viên – ZALO:0973.287.149-TEAMLUANVAN.COM
1 0 1 0
b − 1 b
!
=
b − 1 1
!
1 1
!
0 1
!
1 1 1 0 1 0 1 0
và
c − 1 1
!
=
0 1
!
1 1
!
c − 1 1
!
.
c 1
Chúng minh.
1 0 1 0 1 0
Bat kì so hǎu tỉ u/v > 1 có hai phân so liên tục. M®t trong chúng bat đau
với 1 chỉ khi u/v = 1 và phân so liên tục là [0, 1]. Do v y, giả sả c/d bao
hàm as > 0. Đieu này cháng tỏ phan (a), phan (b) là chuyen vị (ho c l p lại
cháng minh). Q
H quả 2.2. Cho a, b, c, d là các so nguyên với ad − bc = ± 1 và c > d > 0.
Cho x, y là hai so vô tỉ thỏa mãn y > 1 và
ay + b
x = .
cy + b
Cho x = [a0, a1, ...] là dạng khai trien phân so liên tục đơn giản của x. Thì
ton tại s ≥ 1 sao cho
a = ps, b = ps− 1, c = qs, r = qs− 1, y = xs+1.
Chúng minh.
Sả dụng bő đe 2.1, ta viet
a b
!
=
a′
0 1
!
a′
1
1
!
...
a′
s
1
!
,
với a′
, ..., a′ b
dương và = [a′
, a′
, ..., a′
],
c
= [a′
, ..., a′
]. Tà c > d và h
1 s− 1 d 0 1 s− 1
d s 1
quả 2.1, ta suy ra a′
s > 0 và
a
= [a′
, a′
, ..., a′
] =
ps
′
, x =
ps
′
y + ps
′
−1
= [a′
, ..., a′
, y].
c 0 1 s
qs
′
qs
′ y + qs
′
−1
Tà y > 1 là ai
′
= ai, p′
i = qi
′
với 0 ≤ i ≤ s và y = xs+1. Q
c d 1 0
,
0 s
39
Viết đề tài giá sinh viên – ZALO:0973.287.149-TEAMLUANVAN.COM
≥
0 1 q 1
Nh n xét 2.1. Đưa ra các so nguyên a0, a1, ... với ai > 0 cho i ≥ 1 và viet
với n 0,
pn
qn
hai công thác:
= [a0, a1 , ..., an ] ta kiem tra bởi phương pháp quy nạp trên n,
1 ao
!
1 0
!
...
1 an
!
=
pn− 1 pn
!
neu n chȁn
0 1 a1 1 0 1 qn− 1 qn
(2.3)
ao
!
1 0
!
...
1 0
!
=
pn
!
neu n lẻ
Xác định hai ma tr n U và L trong GL2(R) của định thác +1 bởi
U =
1 1
!
và L =
1 0
!
.
0 1 1 1
Cho p và q trong Z, ta có
Up
=
1 p
!
và Lq
=
1 0
!
.
Do đó các công thác (2.3) có dạng:
Ua0
La1
...Lan
=
pn−1 pn
qn− 1 qn
!
neu n chȁn,
và
Ua0
La1
...Lan
=
pn pn−1
qn qn− 1
!
neu n lẻ.
Đieu liên quan với thu t toán Euclid là:
U− p a b
!
=
a − pc b − pd
!
và L− q a b
!
=
a b
!
.
c d c d c d c − qa d − qb
1 pn− 1
0 1 a1 1 an 1 qn qn− 1
40
Viết đề tài giá sinh viên – ZALO:0973.287.149-TEAMLUANVAN.COM
0
2.1.2. Chu kì phân so liên tnc
M®t dãy so vô hạn an
n≥0
được cho là chu kỳ sau cùng neu ton tại
n0 ≥ 0 và s ≥ 1 sao cho:
với moi n ≥ n0.
an+s = an, (2.4)
T p của s thỏa mãn thu®c tính (2.1.2) là t p b®i so dương của so nguyên
s0 và (an0 , an0+1, ..., an0+s0−1) được goi là chu kỳ cơ bản. M®t phân so liên tục
với dãy của các thương so thỏa mãn (2.4) sě được viet như sau:
[a0, a1, ..., an0−1, an0, ..., an0+s−1].
Ví dn 2.2. Cho D là so nguyên dương không chính phương, t p a0 =
[
√
D♩ , ta có bởi định lý 2.1
a +
√
D = [2a , a1, ..., as− 1 ] và
1
√
D − a0
= [a1, ..., as− 1 , 2a0].
Bo đe 2.2. (Euler, 1737 ). Neu phân so vô hạn liên tục x = [a0, a1, ..., an, ...]
là chu kỳ sau cùng, thì x là so vô t b c hai.
Chúng minh.
Tà phân so liên tục của x là vô hạn, x là so vô t . Giả sả đau tiên phân so
liên tục là chu kì, trong (2.4) với n0 = 0,
x = [a0, ..., as− 1].
Ta có the viet
Do v y
Suy ra
x = [a0, ..., as− 1, x].
x =
ps− 1x + ps− 2
.
qs− 1x + qs− 2
qs− 1x2
+ (qs− 2 − ps− 1)x − ps− 2,
0
41
Viết đề tài giá sinh viên – ZALO:0973.287.149-TEAMLUANVAN.COM
n− 1 n− 1
n− 1
là m®t đa thác b c hai khác không với h so nguyên có x như là nghi m a.
Vì x là so vô t , đa thác này không rút gon và x là b c hai.
Trong trường hợp tőng quát tại (2.4) với n0 > 0,ta viet:
x = [a0, a1, ..., an0−1, an0, ..., an0+s−1] = [a0, a1, ..., an0−1, y],
với y = [an0, ..., an0+s−1] là chu kì phân so liên tục, vì v y x là b c hai. Nhưng
x =
pn0− 1 + pn0− 2
,
qn0− 1 + qn0− 2
do v y x ∈ Q(y) là so vô t b c hai. Q
Bo đe 2.3. (Lagrange, 1770) Neu x là so vô t b c hai thì phân so liên tục
x = [a0, a1, ..., an, ...]
là chu kì sau cùng.
Chúng minh.
Cho n ≥ 0, xác định dn = qnx − pn. Theo h quả 1.2, ta có | dn |<
1
.
qn+1
Ax2
+ Bx + C với A > 0 là đa thác b c hai không rút gon có x là nghi m.
Với n ≥ 2, ta suy ra tà (1.21) rang h®i tụ xn là nghi m của đa thác b c hai
Anx2
+ Bnx + Cn với
An = Ap2
+ Bpn− 1qn− 1 + Cq2
,
Bn = 2Apn− 1pn− 2 + B(pn− 1qn− 2 + pn− 2qn− 1) + 2Cqn− 1qn− 2,Cn
= An− 1.
Cho Ax2
+ Bx + C = 0, ta suy ra:
An = (2Ax + B)dn− 1qn− 1 + Ad2
,
Bn = (2Ax + B)(dn− 1qn− 2 + dn− 2qn− 1) + 2Adn− 1dn− 2,
có công thác tương tự bieu dien A, B, C như tő hợp tuyen tính đong nhat
của An, Bn, Cn và tà (A, B, C) (0, 0, 0) ta có (An, Bn, Cn) =
/ (0, 0, 0). Tà
khi xn là so vô t , suy ra An 0. Tà bat đȁng thác
qn− 1 | dn− 2 |< 1, qn− 2 | dn− 1 |< 1, qn− 1 < qn, | dn− 1dn− 2 |< 1,
42
Viết đề tài giá sinh viên – ZALO:0973.287.149-TEAMLUANVAN.COM
suy ra
max{|An|, |Bn/2, |Cn|} < A + |2Ax + B|.
Đieu này cho thay |An|, |Bn|, |Cn| là giới hạn đ®c l p của n. Bởi v y ton tại
n0 ≥ 0 và s > 0 sao cho xn0 = xn0+s. Tà đó ta suy ra phân so liên tục của x
là chu kì sau cùng. Q
Bo đe 2.4. M®t phân so liên tục
x = [a0, a1, ..., an...],
là chu kì tuan hoàn khi và chỉ khi x là so vô t b c hai giảm. Trong trường
hợp này, neu x = [a0, a1, ..., as− 1] và neu x′
là liên hợp Galois của x thì
−1/x′
= [as− 1, ..., a1, a0].
Chúng minh.
Giả sả phân so liên tục của x là chu kì tuan hoàn
x = [a0, a1, ..., as− 1].
Tà as = a0 ta suy ra a0 > 0, do v y x > 1. Tà x = [a0, ..., as− 1] và tính duy
nhat của dạng khai trien phân so liên tục ta suy ra
x =
ps− 1x + ps− 2
qs− 1x + qs− 2
và x = xs, do đó x là nghi m của đa thác b c hai
Ps(x) = qs− 1x2
+ (qs− 2 − ps− 1)x − ps− 2.
Đa thác Ps này có nghi m dương, cụ the x > 1 và nghi m âm x′
với ket quả
xx′
= −ps− 2/qs− 1. Ta hoán vị moi quan h
ps− 1 ps− 2
!
=
a0 1
!
a1 1
!
...
as− 1 1
!
qs− 1 qs− 2
và có được
1 0 1 0 1 0
ps− 1 qs− 1
!
=
as− 1 1
!
...
a1 1
!
a0 1
!
.
ps− 2 qs− 2 1 0 1 0 1 0
43
Viết đề tài giá sinh viên – ZALO:0973.287.149-TEAMLUANVAN.COM
Định nghĩa như sau
đe y > 1,
y = [as− 1, ..., a1, a0],
y = [as− 1, ..., a1 , a0 , y] =
ps− 1y + qs− 1
,
ps− 2y + qs− 2
và y là nghi m dương của đa thác
Qs(x) = ps− 2x2
+ (qs− 2 − ps− 1)x − qs− 1.
Đa thác Ps, Qs có quan h bởi Qs(x) = −x2
Ps(−1/x). Do v y y = −1/x′
.
Ngược lại, giả sả x > 1 và −1 < x′
< 0. Cho (xn)n≥1 là dãy các thương so
hoàn chỉnh của x. Cho n ≥ 1, xác định x′
n như liên hợp Galois của xn. Bang
quy nạp suy ra x′
n = an + 1/x′
xn
′
+1 và −1 < x′
n < 0( do v y xn giảm) và an
là phan nguyên của −1/x′
n+1. Neu khai trien phân so liên tục của x là chu kì
không tuan hoàn, ta có
x = [a0, ..., ah− 1, ah, ..., ah+s− 1],
với ah− 1 /= ah+s− 1. Bang tính chu kì ta có xh = [ah, ..., ah+s− 1, xh], do v y
xh = xh+s, x′
h = xh
′
+s. Tà x′
h = x′
h+s lay phan nguyên, ta suy ra ah− 1 = ah+s− 1,
mâu thuan. Q
H quả 2.3. Neu r > 1 là so hǎu t không chính phương thì khai trien
phân so liên tục của
√
r có dạng
√
r = [a0, a1, ..., as− 1, 2a0],
với a1, ..., as− 1 có vai trò như nhau và a0 = [
√
r♩ . Ngược lại, neu khai trien
phân so liên tục của so vô t t > 1 có dạng
t = [a0, a1, ..., as− 1, 2a0],
với a1, ..., as− 1 có vai trò như nhau thì t2
là so hǎu t .
Chúng minh.
44
Viết đề tài giá sinh viên – ZALO:0973.287.149-TEAMLUANVAN.COM
−
Neu t2
= r là so hǎu t lớn hơn 1 thì cho a0 = [
√
t♩ và so x = t + a0 là giảm.
Tà đó t + t′
= 0, ta có
Do v y
1
−
x′
=
1
.
x − 2a0
1
x = [2a0, a1, ..., as− 1], −
x′
= [as− 1, ..., a1, 2a0],
với a1, ..., as− 1 có vai trò như nhau.
Ngược lại, neu t = [a0, a1, ..., as− 1, 2a0] với a1, ..., as− 1 có vai trò như nhau thì
x = t + a0 là chu kì giảm và liên hợp Galois của x′
thỏa mãn
1
−
x′
= [a1, ..., as− 1, 2a0] =
1
,
x − 2a0
nghĩa là t + t′
= 0, do v y t2
∈ Q. Q
Bo đe 2.5. (Serret, 1878) Cho x, y là hai so vô t với phân so liên tục
x = [a0, a1, ..., an...] và y = [b0, b1, ..., bm, ...]
tương áng thì hai thu®c tính sau tương đương:
(i) Ton tại ma tr n
a b
c d
!
, với h so nguyên và định thác ± 1 sao cho
ax + b
y = .
cx + d
(ii) Ton tại n0 ≥ 0 và m0 ≥ 0 sao cho an0+k = bm0+k với moi k ≥ 0.
Đieu ki n (i) có nghĩa là x và y là modul tương đương sự tác đ®ng của GL2(Z)
bởi sự đong nghĩa. Đieu ki n (ii) có nghĩa là ton tại so nguyên n0, m0 và so
thực t > 1 sao cho
x = [a0, a1, ..., an0−1, t] và y = [b0, b1, ..., bm0−1, t].
Ví dụ như neu x = [a0, a1, x2] thì
x =
[− a0 − 1, 1, a1 − 1, x2], a1 ≥ 2
[− a0 − 1, 1 + x2], a1 = 1
(2.5)
45
Viết đề tài giá sinh viên – ZALO:0973.287.149-TEAMLUANVAN.COM
! ! !
Chúng minh.
Tà công thác (1.21) neu xn là m®t thương hoàn toàn của x, thì x và xn là
modul tương đương GL2(Z), đieu ki n (ii) nghĩa là m®t phan thương của x
và m®t phan thương của y là bình đȁng. Bởi tính bac cau của GL2(Z) tương
đương, (ii) kéo theo (i).
Ngược lại, giả sả (i):
ax + b
y = .
cx + d
Cho n là m®t so đủ lớn, tà
với
a b pn pn− 1
c d qn qn− 1
=
un un− 1
,
vn vn− 1
Ta suy ra
un = apn + bqn, un− 1 = apn− 1 + bqn− 1,vn
= cpn + dqn, vn− 1 = cpn− 1 + dqn− 1.
y =
unxn+1 + un− 1
.
vnxn+1
+ vn− 1
Ta có vn = (cx + d)qn + cδn với δn = pn − qnx. Ta có qn → ∞, q ≥ qn− 1 + 1và
δn → 0 như n → ∞. Do v y, cho n đủ lớn, ta có vn > vn− 1 > 0. Tà phan 1
của h quả 2.1, ta suy ra:
un un−1
!
=
a0 1
!
a1 1
!
...
as 1
!
,
vn vn− 1 1 0 1 0 1 0
với a0, ..., as ∈ Z và a1, ..., as dương. Do đó: y = [a0, a1, ..., as, xn+1]. Q
Cháng minh khác được đưa ra bởi Bombieri. Ông sả dụng thực te là GL2(Z)
được tạo bởi hai ma tr n:
1 1
!
và
0 1
!
.
0 1 1 0
46
Viết đề tài giá sinh viên – ZALO:0973.287.149-TEAMLUANVAN.COM
Các phép bien đői tuyen tính phân đoạn có liên quan là K và J xác định bởi:
1
K(x) = x + 1 và J(x) = .
x
Ta có J2
= 1 và
K([a0, t]) = [a0 + 1, t], K− 1
([a0, t]) = [a0 − 1, t].
Có J([a0, t]) = [0, a0, t] neu a0 > 0 và J([0, t]) = [t]. Theo (2.5), phân so liên
tục của x và − x chỉ khác nhau bởi dạng đau tiên. Cháng minh hoàn thành.
Ket lu n: Tác giả đã trình bày bő đe chính và chu kì phân so liên tục sě
được dùng làm ket quả đe cháng minh các kien thác của phan sau.
2.2. Xap xỉ Diophantine và phân so liên tnc đơn giản
Trong phan này tác giả sě trình bày lí thuyet phân so của
√
D, xap xỉ
Diophantine và phân so liên tục đơn giải trong giải phương trình Pell. N®i
dung chính được tham khảo trong tài li u [4].
2.2.1. Phân so liên tnc đơn giản của
√
D
M®t chuoi vô hạn (an)n≥1 là chu kì neu ton tại m®t so nguyên dương s sao
cho
an+s = an(n ≥ 1). (2.6)
Trong trường hợp này, các chuoi hǎu hạn (a1, a2, ..., as) được goi là chu kì của
chuoi ban đau. Ta viet
(a1, a2, ...) = (a1, ..., as).
Neu s0 là so nguyên dương nhỏ nhat thỏa mãn (2.6) thì t p của s thỏa mãn
(2.6) là t p của các b®i dương của s0. Trong trường hợp này (a1, ..., as0 ) được
goi là chu kì cơ bản của chuoi ban đau.
47
Viết đề tài giá sinh viên – ZALO:0973.287.149-TEAMLUANVAN.COM
n n 1 1
0 1 1
Định lí 2.1. Cho D là so nguyên dương không chính phương. Phân so liên
tục đơn giản của
√
D là [a0, a1, ...] với a0 = [
√
D♩ .
(a) Chuoi (a1, a2, ...) là chu kì.
(b) Cho (x, y) là nghi m nguyên dương của phương trình Pell x2
− Dy2
= ± 1. Ton
tại s ≥ 1 sao cho x/y = [a0, ..., as− 1] và (a1, a2, ..., as− 1, 2a0) là chu kì của chuoi
(a1, a2, ...). Hơn nǎa as− i = ai với 1 ≤ i ≤ s − 1. Trong đó a1, ..., as− 1 là đeu như
nhau.
(c) Cho (a1, a2, ..., as− 1, 2a0) là chu kì của chuoi (a1, a2, ...). T p x/y = [a0, ..., as− 1]
thì x2
− Dy2
= (− 1)s
.
(d) Cho s0 là đ® dài của chu kì cơ bản với i ≥ 0 không là b®i so của s0, ta có
ai ≤ a0. Neu (a1, a2, ..., as− 1, 2a0) là chu kì của chuoi (a1, a2, ...) thì
√
D = [a , a1, ..., as− 1 ,2a0] = [a0 , a1, ..., as− 1 , a0 +
√
D].
Xét chu kì cơ bản (a1, a2, ..., as0−1, as0 ) của chuoi (a1, a2, ...). Bởi phan (b)
của định lí 2.1 ta có as = 2a0 và bởi phan (d), s0 là chỉ so nhỏ nhat i sao cho
ai > a0. Tà (b) và (c) trong định lí 2.1 nghi m cơ bản (x1, y1) của phương trình
Pell x2
− Dy2
= ±1 cho bởi x1/y1 = [a0, a1, ..., as − 1] và x2
− Dy2
= (−1)s0
. Do
đó neu s0 là chȁn thì không có nghi m của phương trình Pell x2
− Dy2
= − 1. Neu
s0 lẻ thì (x1, y1) là nghi m cơ bản của phương trình Pell x2
− Dy2
= − 1, trong
khi nghi m cơ bản (x2, y2) của phương trình Pell x2
− Dy2
= 1 được đưa ra
bởi công thác x2/y2 = [a0, a1, ..., a2s− 1]. Tà định lí 2.1 nói rang thá(ns0 −
1) h®i tụ
thỏa mãn :
xn/yn = [an, ..., ans0−1],
x + y
√
D = (x + y
√
D)n
. (2.7)
Chúng minh.
Bat đau với nghi m dương (x, y) của phương trình Pell:
x2
− Dy2
= ± 1,
0
48
Viết đề tài giá sinh viên – ZALO:0973.287.149-TEAMLUANVAN.COM
x y 1 0 1 0 1 0
1 0 1 0 1 0
x y δ 1 x + δy y
mà theo ton tại của m nh đe 1.1. Tà đó, Dy ≥ x và x > y ta có the sả dụng
bő đe 2.1 và h quả 2.1 với
a = Dy, b = c = x, d = y,
và viet
Dy x
!
=
a′
0 1
!
a′
1 1
!
...
as
′
1
!
, (2.8)
với so nguyên dương a′
0, ..., a′
s và viet a′
0 = [
√
D♩ . Sự khai trien phân so
liên tục của Dy/x là [a0
′
, ..., a′
s] và sự khai trien phân so liên tục của x/y là
[a0
′
, ..., a′
s−1]. Tà đó ma tr n phía bên trái của (2.8) là đoi xáng, a′
0, ..., a′
s là
đeu như nhau. Đ c bi t a′
s = a′
0. Xét chu kì phân so liên tục
δ = [a0
′
, a1
′
, ..., a′
s−1, 2a′
0].
So δ này thỏa mãn
δ = [a′
0, a′
1, ..., a′
s−1, a′
0 + δ].
Sả dụng phép nghịch đảo ma tr n
1 0
δ
Do v y ket quả của ma tr n liên quan đen phân so liên tục của δ,
a′
0 x
!
a′
1 x
!
...
a′
s−1 x
!
a′
0 + δ 1
!
,
Dy x
!
1 0
!
=
Dy + δx x
!
.
1 0
là
a′
0 1
!
đó là
0 1
1 −a′
0
!
.
Ta viet
a0
′
+ δ 1
1 0
!
=
a′
0 1
1 0
!
1 0
!
.
1
49
Viết đề tài giá sinh viên – ZALO:0973.287.149-TEAMLUANVAN.COM
±
0 1 s− 1 0
0 n
0 m
m m
2
x + δy i i
n n
2
Tà đó δ =
Dy + δx
, do v y δ2
= D. Như m®t h quả, a′
= a với 0 ≤ i ≤ s− 1,
trong đó a′
s = a0,as = 2a0. Đieu này cháng tỏ neu (x, y) là nghi m không tam
thường của phương trình Pell x2
+ Dy2
= 1 thì sự khai trien phân so liên
tục của
√
D có dạng
√
D = [a , a , ..., a , 2a ], (2.9)
với a1, ...., as− 1 là đeu như nhau, và x/y được đưa ra bởi sự h®i tụ.
x/y = [a0, a1, ..., as− 1]. (2.10)
Xét h®i tụ pn/qn = [a0, a1, ..., an]. Neu an+1 = 2a0, thì (1.22) với x =
√
D kéo
theo
|
√
D
pn
—
qn
| ≤
1
2a q2
,
và tà h quả 1.1 (pn, qn) là nghi m của phương trình Pell p2
− Dq2
= ± 1. Ta
thay ai < 2a0 khi i + 1 không là đ® dài của chu kì. Ta quy ước ước tính này
đen ai ≤ a0.
Giả sả an+1 ≥ a0 + 1. Tà đó chuoi (am)m≥1 là chu kì của chieu dài chu kì s0,
với bat kì đong dư m đen n modul s0. Ta có am+1 > a0. Cho m ta có
|
√
D
pm
—
qm
| ≤
1
(a + 1)q2
.
Cho m đủ lớn đong dư với n modul s ta có
(a0 + 1)q2
> q2
√
D + 1.
H quả 1.1 kéo theo (pm, qm) là m®t nghi m của phương trình Pell
m − Dqm = ± 1.
Cuoi cùng định lí 2.1 kéo theo m+1 là b®i so của s0, do v y n+1 cũng the. Q
p
50
Viết đề tài giá sinh viên – ZALO:0973.287.149-TEAMLUANVAN.COM
pn pn− 1
. . qn− pp
1 n−1
n−1 .
. qn
. . qn− 1
.
qn
. q
n
q
n
q
n
. qn− 1
. . qn− 1
Q
n n− 1
Chúng minh.
2.2.2. Xap xỉ Diophantine và phân so liên tnc đơn giản
Bo đe 2.6. (Langrange, 1770) Chuoi (|qnx − pn|)n≥0 là giảm tuy t đoi,
cho n ≥ 1 ta có:
Chúng minh.
|qnx − pn| < |qn− 1x − pn− 1|.
Ta sả dụng bő đe 1.6 hai lan: M®t m t,
1 1
|qnx − pn| =
vì xn+1 > 1. M t khác,
xn+1.q
n
+ qn− 1
<
qn + q
,
n− 1
1
|qn− 1x − pn− 1| =
x .q + qn− 2
1
>
(an+1).qn− 1 + q n− 2
1
= ,
qn + qn− 1
vì xn < an + 1. Q
H quả 2.4. Chuoi (|x − pn/qn|)n≥0 là giảm tuy t đoi với n ≥ 1 ta có:
.x − . < .x − . .
Cho n ≥ 1, tà khi qn− 1 < qn, ta có
pn 1
x − = |q x − p
1
| < |q x − p | = .x − . < .x − . .
Bo đe 2.7. Với n ≥ 0 và (p, q) ∈ Z × Z với q > 0 thỏa mãn
|qx − p| < |qnx − pn|.
Thì q ≥ qn+1.
Chúng minh.
H so của phương trình tuyen tính trong hai ȁn u, v
pnu + pn+1v = p
qnu + qn+1 v = q
(2.11)
. n n n− 1 n− 1
51
Viết đề tài giá sinh viên – ZALO:0973.287.149-TEAMLUANVAN.COM
ǁ ǁ
1
p 1 . . . .
. .
.x − . < .x − .,
nhat m®t trong chúng thỏa mãn x − < .
.
p pn
có định thác ± 1, do v y m®t nghi m (u, v) ∈ Z × Z. Tà , ta có v /= 0.
q
q qn
Neu u = 0 thì v =
qn+1
> 0, do v y v ≥ 1 và q ≥ qn+1. Giả sả u.v /= 0. Tà
q, qn và qn+1 là lớn hơn 0, nó không the cho u và v đeu âm. Trong trường hợp
u, v dương ta được ket quả tà moi quan h thá hai của (2.11). Do v y, giả sả
u và v của dau hi u đoi nhau tà qnx − pn và qn+1x − pn+1 cũng có dau hi u đoi
nhau, so u(qnx − pn) và v(qn+1x − pn+1) có cùng dau hi u và do đó:
|qnx − pn| ≤ |u(qnx − pn| + |v(qn+1x − pn+1| = |qx − p| < |qnx − pn|,
là mâu thuan. Q
H quả 2.5. Chuoi (qn)n≥0 của mau so h®i tụ của so thực vô t x là chuoi
tăng dan của so nguyên dương mà:
ǁ qnx ǁ<ǁ qx ǁ với 1 ≤ q ≤ qn.
Ket quả là:
qnx = min
≤q≤qn
ǁ qx ǁ .
Các lí thuyet của phân so liên tục được phát trien tà h quả 2.5 như m®t
định nghĩa của chuoi (qn)n≥0).
H quả 2.6. Cho n ≥ 0 và
p
q
∈ Q với q > 0 thỏa mãn
thì q > qn.
Chúng minh:
Cho q ≤ qn, ta có:
. q. . qn
.
.x − . = .|qx − p| > 1.|q
n
.x − pn
qn pn pn
| x − ≥ x − .
. q. q q q . qn
. . qn
Q
Bo đe 2.8. (Vahlen, 1895) Trong hai h®i tụ liên tiep
pn
qn
và
pn+1
qn+1
thì có ít
p 1
. q. 2q2
p pn
52
Viết đề tài giá sinh viên – ZALO:0973.287.149-TEAMLUANVAN.COM
− −
pn pn− pp 1
1 n
n−1
−
n
|
. qn
. . qn− 1
. .qn qn− 1
. 2q2 2.q2
n− 1
Chúng minh:
Tà x
pn
qn
và x
pn− 1
qn− 1
có dau hi u ngược nhau,
.x − . + .x − . = . − . =
1 1
< + .
Bat đȁng thác cuoi cùng là ab <
Do đó:
a2
+ b2
cho a
2
1
b với a =
q
n
và b =
1
.
qn− 1
ho c
x
p
n
qn
1
<
2.q2
|x −
pn−1
| <
1
.
qn− 1 2.q2
n− 1
Q
Bo đe 2.9. (É. Borel, 1930) Trong ba h®i tụ liên tiep
pn− 1
,
pn
và
pn+1
. p. 1
qn− 1 qn qn+1
thì có ít nhat m®t trong chúng thỏa mãn .x −
q. < √
5q2
.
Hang so
√
5 không tự thay the bởi m®t cái lớn hơn là cháng minh trong bő đe
2.13. Đieu này đúng cho bat kì so nào với khai trien phân so liên tục nhưng
hǎu hạn phan thương so bang 1( nghĩa là so vùng Φ và tat cả so hǎu tỉ tương
đương với modul ΦGL2(Z)).
Chúng minh:
Nhac lại bő đe 1.6. Cho n ≥ 0,
(− 1)n
qnx − pn =
xn+1.q
n
.
+ qn− 1
Do đó |qn x− pn| <
1
√
5.qn
neu và chỉ neu |xn+1.q
n
+qn− 1 | >
√
5.qn . Theo định
nghĩa rn =
qn− 1
. Thì nó tương đương với x
qn
n+1 + rn | >
√
5.
Nhac lại định nghĩa quy nạp của sự h®i tụ
xn+1 = an+1 +
x
1
.
n+2
n
qnqn− 1
53
Viết đề tài giá sinh viên – ZALO:0973.287.149-TEAMLUANVAN.COM
−
p
Ngoài ra sả dụng định nghĩa của rn, rn+1 và quan h quy nạp qn+1 = an+1qn +
qn− 1, ta có the viet
1
Loại bỏ an+1 :
rn+1
1
+
x r
= an+1 + rn.
1
= xn+1 + rn.
n+2 n+1
Giả sả |xn+1 + rn| ≤
√
5 và |xn+2 + rn+1| ≤
√
5. Ta suy ra:
1 1
√ + ≤
1
+
1
= x
+ r ≤
√
5.
Ta có:
5 − r n+1 rn+1 xn+2 rn+1
n+1 n
2
n+1 —
√
5.rn+1 + 1 ≤ 0.
Nghi m của đa thác X2
√
5X + 1 là Φ =
1 +
√
5
2 và Φ− 1
=
√
5 − 1
. Do v y
2
rn+1 > Φ− 1
.
Ước tính này sau giả thuyet
1 1
|qnx − pn| < √
5.qn
và |qn+1x − pn+1| < .
5.qn+1
Neu ta có |qn+2x − p n+2| <
1
√
5.qn+2
, ta suy ra rn+2 > Φ− 1
, tà đó ta thay
1 = (an+2 + rn+1).rn+2 > (1 + Φ− 1
)Φ− 1
= 1,
vô lí. Q
p
Bo đe 2.10. ( Legendre, 1798) Neu
q
p
∈ Q thỏa mãn .x —
q. ≤ 1
2q2
, thì
là m®t h®i tụ của x.
q
Chúng minh:
Cho r và s trong Z thỏa mãn 1 ≤ s ≤ q. Tà
1 ≤ |qr − ps| = |s(qx− p)− q(sx− r)| ≤ s|qx− p|+q|sx− r| ≤
s
2q
+q|sx− r|.
r
√
54
Viết đề tài giá sinh viên – ZALO:0973.287.149-TEAMLUANVAN.COM
√
0 0
0
Suy ra:
s
q|sx − r| ≥ 1 −
2q
>
1
2
≥ q|qx − p|.
p
Do v y |sx − r| > |qx − p| và do đó bő đe 2.7 kéo theo là m®t h®i tụ của
q
x. Q
Ket lu n: Trong phan này tác giả đã trình bày lí thuyet ve phân so liên tục
đơn giản, xap xỉ Diophantine và phân so liên tục đơn giản trong giải phương
trình Pell.
2.3. Ve m t tiêu chuan cho sfi ton tại nghi m của phương
trình Pell
đây tác giả sě giới thi u ve m®t tiêu chuȁn cho sự ton tại nghi m của
phương trình Pell. Cụ the hơn, đây là ket quả gan đây ve sự ton tại nghi m
của phương trình Pell x2
− Dy2
= − 1. N®i dung chính được tham khảo trong tài
li u [4].
M nh đe 2.1. ( R.A. Mollin, A Srinivasan) Cho D là so nguyên dương
không chính phương. Cho (x0, y0) là nghi m cơ bản của phương trình Pell
x2
− Dy2
= 1. Thì phương trình x2
− Dy2
= − 1 có 1 nghi m neu và chỉ neux0
≡ 1(mod2D).
Chúng minh.
Neu a2
− Db2
= − 1 là nghi m cơ bản của x2
− Dy2
= − 1, thì x0 + y0
√
D =
(a + b D)2
, do v y
x0 = a2
+ Db2
= 2Db2
= 2Db2
− 1 ≡ − 1(mod2D).
Ngược lại neu x0 = 2Dk − 1 thì x2
= 4D2
k2
− 4Dk + 1 = Dy2
+ 1. Do
v y 4Dk2
− 4k = y2
. Do đó y0 là chȁn, y0 = 2z và k(Dk − 1) = z2
. Tà khik
và Dk − 1 là nguyên to cùng nhau, cả hai đeu chính phương, k = b2
vàDk
− 1 = a2
, ta được a2
− Db2
= − 1. Q
55
Viết đề tài giá sinh viên – ZALO:0973.287.149-TEAMLUANVAN.COM
Tác giả đã giới thi u m®t tiêu chuȁn cho sự ton tại nghi m của phương Pell
cơ bản. Tà đó xác định được đieu ki n có nghi m của phương trình.
2.4. M t so m r ng của xap xỉ Diophantine
Trong phan này tác giả giới thi u m®t so mở r®ng của xap xỉ Diophantine,
tà đó có the áng dụng đe giải phương trình Pell nói riêng và phương trình
nghi m nguyên nói chung. N®i dung tham khảo trong tài li u [4].
2.4.1. Tiêu chí vô t
M nh đe 2.2. Cho ϑ là so thực, các đieu ki n sau tương đương:
(i) ϑ là so vô tỉ.
(ii) Cho bat kì ε > 0, ton tại (p, q) ∈ Z2
sao cho q > 0 và
0 < |qϑ − p| < ε.
(iii) Cho bat kì ε > 0, ton tai hai tuyen tính dạng tuyen tính đ®c l p trong
hai bien L0(X0, X1) = a0X0 + b0X1 và L1(X0, X1) = a1X0 + b1X1 với h so
nguyên sao cho max{|L0(1, ϑ)|, |L1(1, ϑ)|} < ε.
(iv) Cho so bat kì Q > 1 ton tại so nguyên q trong khoảng 1 ≤ q ≤ Q và so
nguyên p sao cho
1
0 < |qϑ − p| <
Q
.
(v) Ton tại vô so p/q ∈ Q sao cho:
p
|ϑ −
q
| <
(vi) Ton tại vô so p/q ∈ Q sao cho:
p
1
q2
.
1
|ϑ −
q
| < √
5q2
.
Hàm ý (vi) ⇒ (v) là tam thường. Ta sě cháng minh (i) ⇒ (vi) sau (trong
phan trên phân so liên tục). Bây giờ ta sě cháng minh sự tương đương giǎa
56
Viết đề tài giá sinh viên – ZALO:0973.287.149-TEAMLUANVAN.COM
i i
các đieu ki n của m nh đe 2.2 như sau:
(iv) ⇒ (ii) ⇒ (iii) ⇒ (i) ⇒ (iv) ⇒ (v) ⇒ (ii).
Neu đưa ra m®t so nguyên dương q, có ít nhat m®t giá trị p sao cho |qϑ− p| <
, cụ the so nguyên gan nhat đen qϑ. Do v y, khi ta xap xỉ ϑ bởi so hǎu t
2
p/q, ta có m®t tham so trong Z > 0, cụ the là q. Trong đieu ki n (v), không
can giả định ve trái khác 0, neu m®t p/q ∈ Q cho ket quả 0, thì tat cả các so
khác là không và m®t lan nǎa có vô so chúng.
1
Chúng minh của (iv) ⇒ (ii). Sả dụng (iv) với Q thỏa mãn Q > 1 và Q >
ε
ta thu được (ii). Q
Chúng minh của (v) ⇒ (ii). Theo (v) có dãy vô hạn của các so hǎu t riêng
bi t (pi/qi)i≥0 với qi > 0 sao cho:
pi
|ϑ −
q
| <
1
√
5q2
.
Với moi qi, có duy nhat giá trị cho tả so pi mà bat đȁng thác là thỏa mãn.
1
Do v y t p của qi là vô t n. Lay qi ≥
ε
suy ra (ii). Q
Chúng minh của (ii) ⇒ (iii). Cho ε > 0, tà (ii) ta suy ra sự ton tại của
(p, q) ∈ Z × Z với q > 0 và gcd(p, q) = 1 sao cho:
0 < |qϑ − p| < ε.
Ta sả dụng (ii) m®t lan nǎa với ε thay the bởi |qϑ− p|. Ton tại (p′
, q′
) ∈ Z× Z
với q′
> 0 sao cho:
0 < |q′
ϑ − p′
| < |qϑ − p|. (2.12)
Xác định L0(X0, X1) = pX0 − qX1 và L1(X0, X1) = p′
X0 − q′
X1. Nó kiem tra
L0(X0, X1) và L1(X0, X1) là đ®c l p tuyen tính. Cách khác, ton tại (s, t) ∈
Z2
(0, 0) sao cho sL0 = tL1. Do v y sp = tp′
, sq = tq′
và p/q = p′
/q′
. Tà khi
gcd(p, q) = 1, ta suy ra t = 1, p′
= sp, q′
= sq và q′
ϑ − p′
= s(qϑ − p). Đieu
này không tương đương với (2.12). Q
1
57
Viết đề tài giá sinh viên – ZALO:0973.287.149-TEAMLUANVAN.COM
Chúng minh của (ii) ⇒ (i). Giả sả ϑ ∈ Q, có ϑ =
a
với gcd(a, b) = 1 và
b
b > 0. Cho bat kì dạng tuyen tính khác không L ∈ ZX0 + ZX1 , đieu ki n
L(1, ϑ) /= 0 kéo theo |L(1, ϑ)| ≥
1
. Do v y với ε =
b
1
đieu ki n (iii) không
b
có. Q
Chúng minh của (i) ⇒ (iv) sả dụng nguyên lí h®p Dirichlet. Cho Q > 1 đưa ra
so thực. Xác định N = [Q| nghĩa là N là so nguyên sao cho N − 1 < Q ≤ N .
Tà Q > 1 ta có N ≥ 2. Cho ϑ ∈ R  Q Xét các t p con E của đơn vị khoảng
[0, 1] và xét phan tả N + 1
0, {ϑ}, {2ϑ}, {3ϑ}, ..., {(N − 1)ϑ}, 1.
Tà ϑ là so vô tỉ, phan tả N + 1 là đôi m®t khác nhau. Chia khoảng của N
phan tả,
j
Ij = [
N
;
j + 2
N
] (0 ≤ j ≤ N − 1).
Ít nhat m®t trong nhǎng khoảng cách N, có Ij0 , cháa ít nhat hai phan tả
của E: Phan tả 0 và 1, tat cả phan tả qϑ trong E với 1 ≤ q ≤ N − 1 là sovô
tỉ, do v y thu®c ve sự ket hợp của các khoảng mở (j/N, (j + 1)/N) với0 ≤
j ≤ N − 1. Neu j0 = N − 1 thì khoảng cách:
1
Ij0 = IN− 1 = [1 −
N
, 1]
cháa 1 cũng như m®t phan tả của E có dạng {qϑ} với 1 ≤ q ≤ N − 1. T p
p = [qϑ♩ + 1. Thì ta có 1 ≤ q ≤ N − 1 < Q và
p − qϑ = [qϑ♩ + 1 − [qϑ♩ − {qϑ} = 1 − {qϑ},
do đó
1 1
0 < p − qϑ <
N
≤
Q
.
Cách khác ta có 0 ≤ j0 ≤ N − 2 và Ij0 cháa hai phan tả {q1ϑ} và {q2ϑ} với
0 ≤ q1 ≤ q2 ≤ N − 1. T p
q = q2 − q1, q = [q2ϑ♩ − [q1ϑ♩ .
58
Viết đề tài giá sinh viên – ZALO:0973.287.149-TEAMLUANVAN.COM
a∈ Z
Thì ta có 0 < q = q2 − q1 ≤ N − 1 ≤ Q và
|qϑ − q| = |{q2ϑ} − {q1ϑ}| < 1/N ≤ 1/Q.
Q
Nh n xét 2.2. Với bat kì so thực ϑ, cho bat kì so thực Q > 1 ton tại so
nguyên q trong khoảng 1 ≤ q < Q và so nguyên p sao cho
p 1
|ϑ −
q
| ≤
qQ
.
Các cháng minh đưa đen bat đȁng thác ng t |qϑ − p| ≤ 1/Q trong trường hợp
Q không là so nguyên, trong trường hợp khi Q là so nguyên và ϑ hǎutỉ,
ket quả không đúng với bat đȁng thác nói chung. Ví dụ neu ϑ = a/b với
gcd(a, b) = 1 và b ≥ 2, m®t nghi m p/q với bat đȁng thác ng t cho Q = b + 1
nhưng không cho Q = b. Tuy nhiên Q là so nguyên và ϑ là so vô tỉ, so |qϑ − p|
là vô tỉ ( do q > 0) do v y không bang 1/Q.
Chúng minh của (iv) ⇒ (v). Giả sả (iv). Ta đã biet rang (iv) ⇒ (i). Do v yϑ
là so vô tỉ. Cho {q1, ..., qN } là t p hǎu hạn các so nguyên dương. Ta sě bieu dien
ton tại so nguyên dương q không thu®c {q1, ..., qN } thỏa mãn đieu ki n (v).
Bieu dien bởi ||.|| khoảng cách đen so nguyên gan nhat: Cho x ∈ R
||x|| = min |x − a|.
Tà ϑ là so vô tỉ, cho 1 ≤ j ≤ N, so ||qjϑ|| là so khác không. Cho Q > 1 thỏa
mãn
Q > min
1≤j≤n
− 1
||qjϑ|| .
Tà (iv) suy ra ton tại so nguyên q trong khoảng 1 ≤ q ≤ Q sao cho
1
0 < ||qϑi|| ≤
Q
.
Phía bên ve phải là < 1/q và sự lựa chon của Q có kéo theo q không thu®c
{q1, ..., qN }. Q
59
Viết đề tài giá sinh viên – ZALO:0973.287.149-TEAMLUANVAN.COM
Y
2.4.2. Bat đang thfíc Liouville
Bo đe 2.11. Cho α là so đại so của b c d ≥ 2 và đa thác toi thieuP
∈ Z[X]. Xác đinh c = |P ′
(α)|. Cho ε > 0 thì ton tại so nguyên q0 sao cho,
bat kì p/q ∈ Q với q > q0,
Chúng minh.
p
|α −
q
| ≥
1
(c + ε)qd
.
Ket quả là tam thường neu α không là so thực. M®t giá trị có the chap nh n
cho q0 là
q0 = (c|=m(α)|)− 1/d
.
Giả sả α là so thực. Cho q là so nguyên dương đủ lớn và p là so nguyên gan
nhat đen qα. Đ c bi t:
p 1
|α −
q
| ≤
2q
.
Bieu dien bởi a0 h so hàng đau của p và bởi α1, ..., αd nghi m với α1 = α.
Do v y
P(X) = a0(X − α1)(X − α2)...(X − αd),
và
d
qd
P(p/q) = a qd
Y
(
p
− α ). (2.13)
Cũng như
0 i i=1
d
P′
(α) = a0 (α − αi).
i=2
Ve bên trái của (2.13) là so nguyên. Nó khác không vì P không rút gon b c
≥ 2. Cho i ≥ 2 ta sả dụng các ước tính
p 1
|αi −
q
| ≤ |αi − α| +
2q
.
60
Viết đề tài giá sinh viên – ZALO:0973.287.149-TEAMLUANVAN.COM
n
q
Ta suy ra :
1 ≤ qd
a0|α
p
—
q
|
Y
i=2
|(αi
1
— α)| +
2q
.
Đoi với q đủ lớn bên ve phải bị ch n tà
qd
|α −
p
|(|P′
(α)| + ε).
Q
2.4.3. Bat đang thfíc Liouville b c hai
Xét bő đe 2.11 trong trường hợp đ c bi t d = 2 khi α là so đại so b c
hai. Viet đa thác toi thieu f(X) = aX2
+ bX + c và cho ∆ := b2
− 4ac là
bi t thác. √
Ta quan tâm đen xap xỉ của α bởi√
so hǎu tỉ, giả sả ∆ > 0. Neu
α =
− b ± ∆
2a
thì nghi m khác là α′
=
−b ∓ ∆
và
2a
f′
(α) = a(α − α′
) = ±
√
∆.
Bo đe 2.12. Cho α là đại so của b c 2 và đa thác toi thieu P ∈ Z[X]. Xác
định c = |P′
(α)|. Cho ε > 0 thì ton tại so nguyên q0 sao cho bat kì p/q ∈ Q
với q ≥ q0,
p
|α −
q
| ≥
1
(
√
∆ + ε)q2
.
Bi t thác dương nhỏ nhat của đa thác b c hai không rút gon với h so
trong Z là 5, mà giá trị của bi t thác X2
− X − 1, với nghi m Φ và − Φ− 1
khi
Φ = 1, 6180339887499... bieu thị tỉ l Gold. Ket quả tiep theo với dãy Fibonacci
(Fn)n≥0:
F0 = 0, F1 = 1, Fn = Fn− 1 + Fn− 2 (n ≥ 2).
Bo đe 2.13. Cho bat kì q ≥ 1 và bat kì p ∈ Z,
p
|Φ −
q
| >
1
√
5q2 +
q .
2
61
Viết đề tài giá sinh viên – ZALO:0973.287.149-TEAMLUANVAN.COM
n− 1
q
n− 1
n
|
M t khác,
Chúng minh.
lim
n→∞
F2
|Φ −
Fn
=
Fn− 1
1
√
5
.
Nó đủ đe cháng minh sự bị ch n thap hơn khi p là so nguyên gan nhat qΦ.
Tà X2
− X − 1 = (X − Φ)(X + Φ− 1
) ta suy ra
p2
− pq − q2
= q2
(
p
−
p
Φ)(
q
+ Φ− 1
).
Ve trái là so nguyên khác không, do v y có giá trị tuy t đoi nhỏ nhat là
1
1. Ta ràng bu®c giá trị tuy t đoi của ve phải tà trên. Tà p < qΦ + và
Φ + Φ− 1
=
√
5 ta có:
2
p
+ Φ− 1
<
√
5 +
1
.
q 2q
Phan đau tiên của bő đe 2.13 như sau: Không gian véctơ thực của dãy (vn)n≥0
thoản mãn vn = vn− 1 + vn− 2 có hai chieu, m®t cơ sở được cho bởi hai chuoi
(Φn
)n≥0 và ((− Φ− 1
)n
)n≥0. Tà cách này de dàng suy ra công thác:
1 n n − n
Fn = √
5
(Φ − (−1) Φ ).
Theo A.De.Moivre (1730), L.Euler (1765) và J.P.M.Binet (1843), Fn là so
nguyên gan nhat đen
1 n
√
5
Φ .
Do v y, chuoi (un)n≥2 của thương so Fibonacci
un = Fn/Fn− 1,
thoả mãn lim
n→∞
un = Φ. De dàng kiem tra
F 2
− FnFn− 1 − F 2
= (− 1)n− 1
,
n
chon n ≥ 1. Ve trái là F2
n− 1
(un − Φ)(un + Φ− 1
), như ta đã thay. Do v y
F2
n− 1|Φ − un
1
| =
Φ− 1 + u
,
62
Viết đề tài giá sinh viên – ZALO:0973.287.149-TEAMLUANVAN.COM
1
và giới hạn của ve phải là
1
=
Φ + Φ− 1
√
5
. Q
Nh¾n xét 2.3. Chuoi un = Fn/Fn− 1 cũng được xác định bởi
1
Do v y,
u2 = 2, un = 1 +
u
1
n− 1
(n ≥ 3).
1
un = 1 + 1
1 +
un− 2
= 1 +
1 +
1
1
1 + 1
= ...
un− 3
Nh¾n xét 2.4. Nó được biet đen rang neu k là so nguyên dương, neu so
thực vô tỉ ϑ có dạng khai trien phân so liên tục [a0; a1, a2, ...] với an ≥ k đoi
với vô so n thì
lim inf q
q→∞
|ϑ −
p
q
| ≤
1
√
4 + k2
.
Ket lu n: Tác giả đã trình bày m®t so mở r®ng của xap xỉ Diophantine đoi
với so b c hai. Tà đó áp dụng ket quả đe giải các phương trình Pell.
2.5. M t fíng dnng giải phương trình Pell âm
Trong phan này tác giả sě giới thi u m®t áng dụng giải phương trình Pell
âm. Trong đó, vai trò của x và y đã được đői cho cho nhau. N®i dung được
tham khảo trong tài li u [5].
De thay phương trình y2
= 3x2
− 1, y2
= 7x2
− 4 không có nghi m nguyên
trong khi y2
= 65x2
− 1, y2
= 202x2
− 1 có nghi m nguyên. Đ c bi t hơn, m®t
cách cho giá trị của D đoi với phương trình Pell âm y2
− Dx2
= − 1là giải
được hay không. Trong phan này, phương trình Pell âm đưa ra bởi y2
=
45x2
− 11 là xem xét và vô so nghi m nguyên thu được. M®t so quan h thú vị
trong so các giải pháp được hi n di n.
Phương pháp phân tích:
2
63
Viết đề tài giá sinh viên – ZALO:0973.287.149-TEAMLUANVAN.COM
n+1 n n
n+1 n n
Phương trình Pell âm miêu tả hypebon dưới sự xem xét là:
y2
= 45x2
− 11. (2.14)
Cho nghi m nguyên dương nhỏ nhat là x0 = 2, y0 = 13 . Các nghi m khác
thu được của (2.14), xét phương trình Pell y2
= 45x2
+ 1 cho nghi m tőng
∼ ∼ ∼ 1
∼
1
√
quát (x, y) được đưa ra bởi x = 2 45
gn, y = 2 fn. Nơi mà
fn = (161 + 24
√
45)n+1
+ (161 − 24
√
45)n+1
; (2.15)
gn = (161 + 24
√
45)n+1
− (161 − 24
√
45)n+1
. (2.16)
Đ t nam giǎa (x0, y0) và (
∼ ∼
), nghi m nguyên khác của (2.14) được đưa
ra bởi:
x0, y0
90x = 90f + 13
√
45g (2.17)
2y = 13f + 2
√
45g (2.18)
M®t so ví dụ của y thỏa mãn (2.14) đưa ra trong bảng sau
n xn yn
0 2 13
1 634 4253
2 204146 1369453
3 65734378 440959613
4 21166265570 141987625933
Lời nh¾n xét:
Tà bảng trên, ta quan sát m®t so quan h thú vị trong các nghi m khi được
xuat hi n dưới đây:
1) Giá trị x là chȁn khi giá trị y là lẻ.
2) Moi quan h phép truy toán thỏa mãn bởi nghi m của (2.14) được đưa ra
bởi:
xn+3 − 322xn+2 + xn+1 = 0; (2.19)
64
Viết đề tài giá sinh viên – ZALO:0973.287.149-TEAMLUANVAN.COM
n+2
n+1
n
yn+3 − 322yn+2 + yn+1 = 0. (2.20)
3) xn+3 = 322xn+2 − xn+1.
4) 24yn+1 = xn+2 − 161xn+1.
5)24yn+2 = 161xn+2 − xn+1.
6) 24yn+3 = 51841xn+2 − 161xn+1.
7) 51841xn+1 = xn+3 − 7728yn+1.
8) 25920xn+2 = 3864yn+2 − 24yn+1.
9) 25920xn+2 = 24yn+3 − 3864yn+2.
10) xn+2 = 161xn+3 − 24yn+3.
11) 51840xn+2 = 24yn+3 − 24yn+1.
12) 161xn+2 = xn+3 − 24yn+2.
13) 51841xn+2 = 161xn+3 − 24yn+1.
14) 25920xn+1 = 24yn+2 − 3864yn+1.
15) 51841x2
16) 51841x2
= 161xn+3xn+2 − 24yn+1xn+2.
= xn+1xn+3 − 7728xn+1yn+1.
17) 24yn+1xn+1 = 3864yn+2xn+1 − 25920xn+1xn+2.
18) 3864yn+1xn+2 = 24yn+2xn+2 − 25920xn+1xn+2.
19)
180x2n+2 − 26y2n+2 + 22
11
là m®t chính phương hoàn toàn.
Chúng minh:
Loại bỏ gn giǎa (2.17) và (2.18), ta được:
180xn+1 − 26yn+1 = 11fn. (2.21)
Tương tự bỏ fn giǎa (2.17) và (2.18), ta được:
4
√
45yn+1 − 26
√
45xn+1 = 11gn. (2.22)
Thay the n bởi 2n + 1 trong (2.21) ,nó trở thành
180x2n+2 − 26y2n+2 = 11[f2
− 2].
Hơn nǎa,
180x2n+2 − 26y2n+2 + 22
11
là so chính phương hoàn chỉnh.
Tương tự, moi ví dụ sau là m®t so chính phương hoàn chỉnh:
65
Viết đề tài giá sinh viên – ZALO:0973.287.149-TEAMLUANVAN.COM
n n
i)
1369453x2n+2 − 13x2n+4 + 85008
.
42504
ii)
1369453x2n+3 − 4253x2n+4 + 264
.
132
iii)
57060x2n+2 − 26y2n+3 + 3542
.
1771
iv)
18373140x2n+2 − 26y2n+4 + 1140502
.
570251
v)
4253x2n+2 − 13x2n+3 + 264
.
132
vi)
180x2n+3 − 8506y2n+2 + 3542
.
1771
viii)
57060x2n+3 − 8506y2n+3 + 22
.
11
20)
180x3n+3 − 26y3n+3 + 10626
11
là so nguyên b c ba.
Chúng minh:
Thay the n bởi 3n + 2 trong (2.21), nó trở thành
180x3n+3 − 26y3n+3 + 10626
11
Nh n xét quan trong:
I: Nó trở thành
là so nguyên b c ba.
f2
− g2
= 4. (2.23)
Xác định (X = 180xn+1 − 26yn+1, Y = 4
√
45yn+1 − 26
√
45xn+1). Nên
fn =
X Y
11
, gn =
11
.
Thay the các giá trị trên của (fn, gn) trong (2.23), ta có
X2
− Y 2
= 484
khi bieu dien m®t hypebon.
Tương tự, sả dụng ket hợp tuyen tính trong so các nghi m của (2.14), m®t
trong nhǎng nghi m nguyên tőng quát cho sự lựa chon khác của hypebon
được bieu dien trong bảng 1 sau:
Bảng 1
66
Viết đề tài giá sinh viên – ZALO:0973.287.149-TEAMLUANVAN.COM
II: Xác định (X = 180x2n+2 − 26y2n+2 + 22, Y = 4
√
45yn+1 − 26
√
45xn+1).
Do đó
2 X 2 Y 2
fn =
11
, gn =
112
.
S.NO HYPEBON (X, Y )
1 X2
− Y 2
= 7226360064
(1369453x − 13x ,
90xn+3 − 9186570xn+1
)
n+1 n+3 √
45
2 X2
− Y 2
= 484
180yn+1 − 1170xn+1
(180xn+1 − 26yn+1, √
45
)
3 X2
− Y 2
= 69696
28530xn+3 − 9186570xn+2
(1369453xn+2 − 4253xn+3, √
45
)
4 X2
− Y 2
= 1300744812004
(18373140x − 26y ,
180yn+3 − 123250770xn+1
)
n+1 n+3 √
45
5 X2
− Y 2
= 12545764
(57060x − 26y ,
180yn+3 − 382770xn+1
)
n+1 n+2 √
45
6 X2
− Y 2
= 69696
(4253x − 13x ,
90xn+2 − 28530xn+1
)
n+1 n+2 √
45
7 X2
− Y 2
= 12545764
57060yn+1 − 1170xn+2
(180xn+2 − 8506yn+1, √
45
)
8 X2
− Y 2
= 484
57060yn+2 − 382770xn+2
(57060xn+2 − 8506yn+2, √
45
)
67
Viết đề tài giá sinh viên – ZALO:0973.287.149-TEAMLUANVAN.COM
Thay the các giá trị trên của (f2
, g2
) trong (2.23), ta có
n n
khi bieu dien m®t parabol.
Y 2
= 11X − 484
Tương tự, sả dụng ket hợp tuyen tính trong so các nghi m của (2.14), m®t
trong nghi m nguyên tőng quát cho sự lựa chon khác của parabon được bieu
dien trong bảng 2 sau:
68
Viết đề tài giá sinh viên – ZALO:0973.287.149-TEAMLUANVAN.COM
S.NO PARABOL (X, Y )
1 Y 2
= 42504X − 7226360064 (1369453x− 13x + 85008,
90xn+3 − 9186570xn+1
)
2n+2 2n+4 √
45
2 Y 2
= 11X − 484
180yn+1 − 1170xn+1
(180x2n+2 − 26y2n+2 + 22, √
45
)
3 Y 2
= 132X − 69696
28503xn+3 − 9186570xn+2
(1369453x2n+3 − 4253x2n+4 + 264, √
45
)
4 Y 2
= 570251X − 1300744812004 (18373140x− 26y + 1140502,
180yn+3 − 123250770xn+1
)
2n+2 2n+4 √
45
5 Y 2
= 1771X − 12545764 (57060x− 26y + 3542,
180yn+3 − 382770xn+1
)
2n+2 2n+3 √
45
6 Y 2
= 132X − 69696 (4253x− 13x + 264,
90xn+2 − 28530xn+1
) 2n+2
2n+3 √
45
7 Y 2
= 1771X − 1254564
57060yn+1 − 1170xn+2
(180x2n+3 − 8506y2n+2 + 3542, √
45
)
8 Y 2
= 11X − 484
57060yn+2 − 382770xn+2
(57060x2n+3 − 8506y2n+3 + 22, √
45
)
III: Cho p, q với p > q > 0 là phan tả sinh của b® ba Pytago T(α, β, γ) , khi
α = 2pq, β = p2
− q2
, γ = p2
+ q2
, p > q > 0 . Cho A, P lan lượt bieu dien
Xap xỉ diophantine và phân so liên tục  trong giải Phương trình pell.docx
Xap xỉ diophantine và phân so liên tục  trong giải Phương trình pell.docx
Xap xỉ diophantine và phân so liên tục  trong giải Phương trình pell.docx
Xap xỉ diophantine và phân so liên tục  trong giải Phương trình pell.docx

More Related Content

Similar to Xap xỉ diophantine và phân so liên tục trong giải Phương trình pell.docx

Đề tài: Một số phương pháp giải phương trình vô tỉ, HOT, 9đ - Gửi miễn phí qu...
Đề tài: Một số phương pháp giải phương trình vô tỉ, HOT, 9đ - Gửi miễn phí qu...Đề tài: Một số phương pháp giải phương trình vô tỉ, HOT, 9đ - Gửi miễn phí qu...
Đề tài: Một số phương pháp giải phương trình vô tỉ, HOT, 9đ - Gửi miễn phí qu...Dịch vụ viết bài trọn gói ZALO: 0909232620
 
Xây Dựng Hệ Thống Phân Lịch Thi Tín Chỉ Tại Trường Cao Đẳng Thương Mại Đà Nẵn...
Xây Dựng Hệ Thống Phân Lịch Thi Tín Chỉ Tại Trường Cao Đẳng Thương Mại Đà Nẵn...Xây Dựng Hệ Thống Phân Lịch Thi Tín Chỉ Tại Trường Cao Đẳng Thương Mại Đà Nẵn...
Xây Dựng Hệ Thống Phân Lịch Thi Tín Chỉ Tại Trường Cao Đẳng Thương Mại Đà Nẵn...Dịch vụ viết thuê Luận Văn - ZALO 0932091562
 
Ứng Dụng Hình Học Giải Tích Vào Giải Phương Trình, Bất Phương Trình Và Hệ Phư...
Ứng Dụng Hình Học Giải Tích Vào Giải Phương Trình, Bất Phương Trình Và Hệ Phư...Ứng Dụng Hình Học Giải Tích Vào Giải Phương Trình, Bất Phương Trình Và Hệ Phư...
Ứng Dụng Hình Học Giải Tích Vào Giải Phương Trình, Bất Phương Trình Và Hệ Phư...Dịch vụ viết đề tài trọn gói 0934.573.149
 
Luận văn: Xây dựng và sử dụng hệ thống bài tập có nội dung gắn với thực tiễn ...
Luận văn: Xây dựng và sử dụng hệ thống bài tập có nội dung gắn với thực tiễn ...Luận văn: Xây dựng và sử dụng hệ thống bài tập có nội dung gắn với thực tiễn ...
Luận văn: Xây dựng và sử dụng hệ thống bài tập có nội dung gắn với thực tiễn ...Dịch vụ viết bài trọn gói ZALO: 0936 885 877
 
Luận Văn Đề Cương Lý Thuyết Xác Suất Thống Kê.doc
Luận Văn  Đề Cương Lý Thuyết Xác Suất Thống Kê.docLuận Văn  Đề Cương Lý Thuyết Xác Suất Thống Kê.doc
Luận Văn Đề Cương Lý Thuyết Xác Suất Thống Kê.docsividocz
 
Một số biện pháp nâng cao hiệu quả dạy học hóa học chương oxi lưu huỳnh lớp...
Một số biện pháp nâng cao hiệu quả dạy học hóa học chương oxi   lưu huỳnh lớp...Một số biện pháp nâng cao hiệu quả dạy học hóa học chương oxi   lưu huỳnh lớp...
Một số biện pháp nâng cao hiệu quả dạy học hóa học chương oxi lưu huỳnh lớp...https://www.facebook.com/garmentspace
 
Một số biện pháp nâng cao hiệu quả dạy học hóa học chương oxi lưu huỳnh lớp...
Một số biện pháp nâng cao hiệu quả dạy học hóa học chương oxi   lưu huỳnh lớp...Một số biện pháp nâng cao hiệu quả dạy học hóa học chương oxi   lưu huỳnh lớp...
Một số biện pháp nâng cao hiệu quả dạy học hóa học chương oxi lưu huỳnh lớp...NOT
 
Một số biện pháp nâng cao hiệu quả dạy học hóa học chương oxi lưu huỳnh lớp...
Một số biện pháp nâng cao hiệu quả dạy học hóa học chương oxi   lưu huỳnh lớp...Một số biện pháp nâng cao hiệu quả dạy học hóa học chương oxi   lưu huỳnh lớp...
Một số biện pháp nâng cao hiệu quả dạy học hóa học chương oxi lưu huỳnh lớp...https://www.facebook.com/garmentspace
 
Một số biện pháp nâng cao hiệu quả dạy học hóa học chương oxi lưu huỳnh lớp...
Một số biện pháp nâng cao hiệu quả dạy học hóa học chương oxi   lưu huỳnh lớp...Một số biện pháp nâng cao hiệu quả dạy học hóa học chương oxi   lưu huỳnh lớp...
Một số biện pháp nâng cao hiệu quả dạy học hóa học chương oxi lưu huỳnh lớp...https://www.facebook.com/garmentspace
 

Similar to Xap xỉ diophantine và phân so liên tục trong giải Phương trình pell.docx (20)

Về phương trình hàm Loại giá trị trung bình và áp dụng.docx
Về phương trình hàm Loại giá trị trung bình và áp dụng.docxVề phương trình hàm Loại giá trị trung bình và áp dụng.docx
Về phương trình hàm Loại giá trị trung bình và áp dụng.docx
 
Kĩ thuật tổng hợp Giải bất phương trình hỗn hợp.docx
Kĩ thuật tổng hợp Giải bất phương trình hỗn hợp.docxKĩ thuật tổng hợp Giải bất phương trình hỗn hợp.docx
Kĩ thuật tổng hợp Giải bất phương trình hỗn hợp.docx
 
Đề tài: Một số phương pháp giải phương trình vô tỉ, HOT, 9đ - Gửi miễn phí qu...
Đề tài: Một số phương pháp giải phương trình vô tỉ, HOT, 9đ - Gửi miễn phí qu...Đề tài: Một số phương pháp giải phương trình vô tỉ, HOT, 9đ - Gửi miễn phí qu...
Đề tài: Một số phương pháp giải phương trình vô tỉ, HOT, 9đ - Gửi miễn phí qu...
 
Xấp xỉ hàm đa điều hòa dưới Bởi hàm green đa cực.doc
Xấp xỉ hàm đa điều hòa dưới Bởi hàm green đa cực.docXấp xỉ hàm đa điều hòa dưới Bởi hàm green đa cực.doc
Xấp xỉ hàm đa điều hòa dưới Bởi hàm green đa cực.doc
 
Xây Dựng Hệ Thống Phân Lịch Thi Tín Chỉ Tại Trường Cao Đẳng Thương Mại Đà Nẵn...
Xây Dựng Hệ Thống Phân Lịch Thi Tín Chỉ Tại Trường Cao Đẳng Thương Mại Đà Nẵn...Xây Dựng Hệ Thống Phân Lịch Thi Tín Chỉ Tại Trường Cao Đẳng Thương Mại Đà Nẵn...
Xây Dựng Hệ Thống Phân Lịch Thi Tín Chỉ Tại Trường Cao Đẳng Thương Mại Đà Nẵn...
 
M t so dạng toán Liên quan đen xác suat r i rạc và ứng dụng.docx
M t so dạng toán Liên quan đen xác suat r i rạc  và ứng dụng.docxM t so dạng toán Liên quan đen xác suat r i rạc  và ứng dụng.docx
M t so dạng toán Liên quan đen xác suat r i rạc và ứng dụng.docx
 
M T So Dạng Toán Liên Quan Đen Xác Suat R I Rạc Và Ứng Dụng.docx
M T So Dạng Toán Liên Quan Đen Xác Suat R I Rạc Và Ứng Dụng.docxM T So Dạng Toán Liên Quan Đen Xác Suat R I Rạc Và Ứng Dụng.docx
M T So Dạng Toán Liên Quan Đen Xác Suat R I Rạc Và Ứng Dụng.docx
 
Luận văn thạc sĩ - Đa thức trong các bài toán thi học sinh giỏi.doc
Luận văn thạc sĩ - Đa thức trong các bài toán thi học sinh giỏi.docLuận văn thạc sĩ - Đa thức trong các bài toán thi học sinh giỏi.doc
Luận văn thạc sĩ - Đa thức trong các bài toán thi học sinh giỏi.doc
 
Bat Phương Trình Hàm Sinh B I Các Đại Lư Ng Trung Bình B C Tùy Ý Và Các Dạng ...
Bat Phương Trình Hàm Sinh B I Các Đại Lư Ng Trung Bình B C Tùy Ý Và Các Dạng ...Bat Phương Trình Hàm Sinh B I Các Đại Lư Ng Trung Bình B C Tùy Ý Và Các Dạng ...
Bat Phương Trình Hàm Sinh B I Các Đại Lư Ng Trung Bình B C Tùy Ý Và Các Dạng ...
 
Ứng Dụng Hình Học Giải Tích Vào Giải Phương Trình, Bất Phương Trình Và Hệ Phư...
Ứng Dụng Hình Học Giải Tích Vào Giải Phương Trình, Bất Phương Trình Và Hệ Phư...Ứng Dụng Hình Học Giải Tích Vào Giải Phương Trình, Bất Phương Trình Và Hệ Phư...
Ứng Dụng Hình Học Giải Tích Vào Giải Phương Trình, Bất Phương Trình Và Hệ Phư...
 
Về hệ số nhị thức, hệ số đa thức Và một số bài toán liên quan.doc
Về hệ số nhị thức, hệ số đa thức Và một số bài toán liên quan.docVề hệ số nhị thức, hệ số đa thức Và một số bài toán liên quan.doc
Về hệ số nhị thức, hệ số đa thức Và một số bài toán liên quan.doc
 
Áp Dụng Phương Pháp Phân Hoạch Để Giải Toán Trung Học Phổ Thông.doc
Áp Dụng Phương Pháp Phân Hoạch Để Giải Toán Trung Học Phổ Thông.docÁp Dụng Phương Pháp Phân Hoạch Để Giải Toán Trung Học Phổ Thông.doc
Áp Dụng Phương Pháp Phân Hoạch Để Giải Toán Trung Học Phổ Thông.doc
 
Ứng Dụng Của Cap Và Chỉ So Cho So Nguyên Theo Modulo.docx
Ứng Dụng Của Cap Và Chỉ So Cho So Nguyên Theo Modulo.docxỨng Dụng Của Cap Và Chỉ So Cho So Nguyên Theo Modulo.docx
Ứng Dụng Của Cap Và Chỉ So Cho So Nguyên Theo Modulo.docx
 
Luận văn: Xây dựng và sử dụng hệ thống bài tập có nội dung gắn với thực tiễn ...
Luận văn: Xây dựng và sử dụng hệ thống bài tập có nội dung gắn với thực tiễn ...Luận văn: Xây dựng và sử dụng hệ thống bài tập có nội dung gắn với thực tiễn ...
Luận văn: Xây dựng và sử dụng hệ thống bài tập có nội dung gắn với thực tiễn ...
 
Luận văn: Sử dụng hệ thống bài tập có nội dung gắn với thực tiễn
Luận văn: Sử dụng hệ thống bài tập có nội dung gắn với thực tiễnLuận văn: Sử dụng hệ thống bài tập có nội dung gắn với thực tiễn
Luận văn: Sử dụng hệ thống bài tập có nội dung gắn với thực tiễn
 
Luận Văn Đề Cương Lý Thuyết Xác Suất Thống Kê.doc
Luận Văn  Đề Cương Lý Thuyết Xác Suất Thống Kê.docLuận Văn  Đề Cương Lý Thuyết Xác Suất Thống Kê.doc
Luận Văn Đề Cương Lý Thuyết Xác Suất Thống Kê.doc
 
Một số biện pháp nâng cao hiệu quả dạy học hóa học chương oxi lưu huỳnh lớp...
Một số biện pháp nâng cao hiệu quả dạy học hóa học chương oxi   lưu huỳnh lớp...Một số biện pháp nâng cao hiệu quả dạy học hóa học chương oxi   lưu huỳnh lớp...
Một số biện pháp nâng cao hiệu quả dạy học hóa học chương oxi lưu huỳnh lớp...
 
Một số biện pháp nâng cao hiệu quả dạy học hóa học chương oxi lưu huỳnh lớp...
Một số biện pháp nâng cao hiệu quả dạy học hóa học chương oxi   lưu huỳnh lớp...Một số biện pháp nâng cao hiệu quả dạy học hóa học chương oxi   lưu huỳnh lớp...
Một số biện pháp nâng cao hiệu quả dạy học hóa học chương oxi lưu huỳnh lớp...
 
Một số biện pháp nâng cao hiệu quả dạy học hóa học chương oxi lưu huỳnh lớp...
Một số biện pháp nâng cao hiệu quả dạy học hóa học chương oxi   lưu huỳnh lớp...Một số biện pháp nâng cao hiệu quả dạy học hóa học chương oxi   lưu huỳnh lớp...
Một số biện pháp nâng cao hiệu quả dạy học hóa học chương oxi lưu huỳnh lớp...
 
Một số biện pháp nâng cao hiệu quả dạy học hóa học chương oxi lưu huỳnh lớp...
Một số biện pháp nâng cao hiệu quả dạy học hóa học chương oxi   lưu huỳnh lớp...Một số biện pháp nâng cao hiệu quả dạy học hóa học chương oxi   lưu huỳnh lớp...
Một số biện pháp nâng cao hiệu quả dạy học hóa học chương oxi lưu huỳnh lớp...
 

More from DV Viết Luận văn luanvanmaster.com ZALO 0973287149

More from DV Viết Luận văn luanvanmaster.com ZALO 0973287149 (20)

Ảnh Hưởng Của Marketing Quan Hệ Đến Lòng Trung Thành Của Khách Hàng.Tình Huốn...
Ảnh Hưởng Của Marketing Quan Hệ Đến Lòng Trung Thành Của Khách Hàng.Tình Huốn...Ảnh Hưởng Của Marketing Quan Hệ Đến Lòng Trung Thành Của Khách Hàng.Tình Huốn...
Ảnh Hưởng Của Marketing Quan Hệ Đến Lòng Trung Thành Của Khách Hàng.Tình Huốn...
 
Phát triển nguồn nhân lực tại Uỷ ban nhân dân huyện Trà Bồng, tỉnh Quảng Ngãi...
Phát triển nguồn nhân lực tại Uỷ ban nhân dân huyện Trà Bồng, tỉnh Quảng Ngãi...Phát triển nguồn nhân lực tại Uỷ ban nhân dân huyện Trà Bồng, tỉnh Quảng Ngãi...
Phát triển nguồn nhân lực tại Uỷ ban nhân dân huyện Trà Bồng, tỉnh Quảng Ngãi...
 
Báo cáo tốt Nghiệp tài chính hợp nhất tại tổng công ty Indochina gol...
Báo cáo tốt Nghiệp  tài chính hợp nhất tại tổng công ty Indochina gol...Báo cáo tốt Nghiệp  tài chính hợp nhất tại tổng công ty Indochina gol...
Báo cáo tốt Nghiệp tài chính hợp nhất tại tổng công ty Indochina gol...
 
Tạo động lực thúc đẩy nhân viên làm việc tại ngân hàng TMCP Ngoại Thương Việt...
Tạo động lực thúc đẩy nhân viên làm việc tại ngân hàng TMCP Ngoại Thương Việt...Tạo động lực thúc đẩy nhân viên làm việc tại ngân hàng TMCP Ngoại Thương Việt...
Tạo động lực thúc đẩy nhân viên làm việc tại ngân hàng TMCP Ngoại Thương Việt...
 
Phát triển công nghiệp trên địa bàn Thành phố Tam Kỳ, Tỉnh Quảng Na...
Phát triển công nghiệp trên địa bàn Thành phố Tam Kỳ, Tỉnh Quảng Na...Phát triển công nghiệp trên địa bàn Thành phố Tam Kỳ, Tỉnh Quảng Na...
Phát triển công nghiệp trên địa bàn Thành phố Tam Kỳ, Tỉnh Quảng Na...
 
Giải pháp phát triển cho vay xuất nhập khẩu tại ngân hàng NN&PTNN ch...
Giải pháp phát triển cho vay xuất nhập khẩu tại ngân hàng NN&PTNN ch...Giải pháp phát triển cho vay xuất nhập khẩu tại ngân hàng NN&PTNN ch...
Giải pháp phát triển cho vay xuất nhập khẩu tại ngân hàng NN&PTNN ch...
 
Hoàn thiện công tác lập báo cáo tài chính hợp nhất tại tổng công ...
Hoàn thiện công tác lập báo cáo tài chính hợp nhất tại tổng công ...Hoàn thiện công tác lập báo cáo tài chính hợp nhất tại tổng công ...
Hoàn thiện công tác lập báo cáo tài chính hợp nhất tại tổng công ...
 
Luận Văn Thạc Sĩ Quản trị thành tích nhân viên tại Cục Hải quan TP Đà Nẵng.doc
Luận Văn Thạc Sĩ  Quản trị thành tích nhân viên tại Cục Hải quan TP Đà Nẵng.docLuận Văn Thạc Sĩ  Quản trị thành tích nhân viên tại Cục Hải quan TP Đà Nẵng.doc
Luận Văn Thạc Sĩ Quản trị thành tích nhân viên tại Cục Hải quan TP Đà Nẵng.doc
 
Hoàn thiện công tác quản lý thuế thu nhập cá nhân tại cục thuế Tỉ...
Hoàn thiện công tác quản lý thuế thu nhập cá nhân tại cục thuế Tỉ...Hoàn thiện công tác quản lý thuế thu nhập cá nhân tại cục thuế Tỉ...
Hoàn thiện công tác quản lý thuế thu nhập cá nhân tại cục thuế Tỉ...
 
Đề Tài Phát triển bền vững nông nghiệp Huyện Ba Tơ, Tỉnh Quảng Ngãi....
Đề Tài Phát triển bền vững nông nghiệp Huyện Ba Tơ, Tỉnh Quảng Ngãi....Đề Tài Phát triển bền vững nông nghiệp Huyện Ba Tơ, Tỉnh Quảng Ngãi....
Đề Tài Phát triển bền vững nông nghiệp Huyện Ba Tơ, Tỉnh Quảng Ngãi....
 
Hoàn thiện công tác bảo trợ xã hội trên địa bàn huyện Phong Điền, tỉnh Thừa T...
Hoàn thiện công tác bảo trợ xã hội trên địa bàn huyện Phong Điền, tỉnh Thừa T...Hoàn thiện công tác bảo trợ xã hội trên địa bàn huyện Phong Điền, tỉnh Thừa T...
Hoàn thiện công tác bảo trợ xã hội trên địa bàn huyện Phong Điền, tỉnh Thừa T...
 
Đề Tài Luận VănPhát triển sản phẩm du lịch tại thành phố Đà Nẵng.doc
Đề Tài Luận VănPhát triển sản phẩm du lịch tại thành phố Đà Nẵng.docĐề Tài Luận VănPhát triển sản phẩm du lịch tại thành phố Đà Nẵng.doc
Đề Tài Luận VănPhát triển sản phẩm du lịch tại thành phố Đà Nẵng.doc
 
Đào tạo nghề cho lao động thuộc diện thu hồi đất trên địa bàn Thàn...
Đào tạo nghề cho lao động thuộc diện thu hồi đất trên địa bàn Thàn...Đào tạo nghề cho lao động thuộc diện thu hồi đất trên địa bàn Thàn...
Đào tạo nghề cho lao động thuộc diện thu hồi đất trên địa bàn Thàn...
 
Tóm Tắt Luận Văn Thạc Sĩ Quản Trị Kinh Doanh Xây dựng chính sách Marketing tạ...
Tóm Tắt Luận Văn Thạc Sĩ Quản Trị Kinh Doanh Xây dựng chính sách Marketing tạ...Tóm Tắt Luận Văn Thạc Sĩ Quản Trị Kinh Doanh Xây dựng chính sách Marketing tạ...
Tóm Tắt Luận Văn Thạc Sĩ Quản Trị Kinh Doanh Xây dựng chính sách Marketing tạ...
 
Đề Tài Nghiên cứu rủi ro cảm nhận đối với mua hàng thời trang trực tuyến.docx
Đề Tài Nghiên cứu rủi ro cảm nhận đối với mua hàng thời trang trực tuyến.docxĐề Tài Nghiên cứu rủi ro cảm nhận đối với mua hàng thời trang trực tuyến.docx
Đề Tài Nghiên cứu rủi ro cảm nhận đối với mua hàng thời trang trực tuyến.docx
 
Giải pháp nâng cao động lực thúc đẩy người lao động tại công ty khai...
Giải pháp nâng cao động lực thúc đẩy người lao động tại công ty khai...Giải pháp nâng cao động lực thúc đẩy người lao động tại công ty khai...
Giải pháp nâng cao động lực thúc đẩy người lao động tại công ty khai...
 
Giải pháp phát triển dịch vụ ngân hàng điện tử tại ngân hàng đầu ...
Giải pháp phát triển dịch vụ ngân hàng điện tử tại ngân hàng đầu ...Giải pháp phát triển dịch vụ ngân hàng điện tử tại ngân hàng đầu ...
Giải pháp phát triển dịch vụ ngân hàng điện tử tại ngân hàng đầu ...
 
Giải pháp phát triển dịch vụ ngân hàng điện tử tại ngân hàng đầu ...
Giải pháp phát triển dịch vụ ngân hàng điện tử tại ngân hàng đầu ...Giải pháp phát triển dịch vụ ngân hàng điện tử tại ngân hàng đầu ...
Giải pháp phát triển dịch vụ ngân hàng điện tử tại ngân hàng đầu ...
 
Quản trị quan hệ khách hàng tại Chi nhánh Viettel Đà Nẵng – Tập đoàn Viễn thô...
Quản trị quan hệ khách hàng tại Chi nhánh Viettel Đà Nẵng – Tập đoàn Viễn thô...Quản trị quan hệ khách hàng tại Chi nhánh Viettel Đà Nẵng – Tập đoàn Viễn thô...
Quản trị quan hệ khách hàng tại Chi nhánh Viettel Đà Nẵng – Tập đoàn Viễn thô...
 
Đề Tài Đánh giá thành tích đội ngũ giảng viên trường Đại Học Phạm ...
Đề Tài Đánh giá thành tích đội ngũ giảng viên trường Đại Học Phạm ...Đề Tài Đánh giá thành tích đội ngũ giảng viên trường Đại Học Phạm ...
Đề Tài Đánh giá thành tích đội ngũ giảng viên trường Đại Học Phạm ...
 

Recently uploaded

NHững vấn đề chung về Thuế Tiêu thụ đặc biệt.ppt
NHững vấn đề chung về Thuế Tiêu thụ đặc biệt.pptNHững vấn đề chung về Thuế Tiêu thụ đặc biệt.ppt
NHững vấn đề chung về Thuế Tiêu thụ đặc biệt.pptphanai
 
ĐỀ KIỂM TRA CUỐI KÌ 2 BIÊN SOẠN THEO ĐỊNH HƯỚNG ĐỀ BGD 2025 MÔN TOÁN 10 - CÁN...
ĐỀ KIỂM TRA CUỐI KÌ 2 BIÊN SOẠN THEO ĐỊNH HƯỚNG ĐỀ BGD 2025 MÔN TOÁN 10 - CÁN...ĐỀ KIỂM TRA CUỐI KÌ 2 BIÊN SOẠN THEO ĐỊNH HƯỚNG ĐỀ BGD 2025 MÔN TOÁN 10 - CÁN...
ĐỀ KIỂM TRA CUỐI KÌ 2 BIÊN SOẠN THEO ĐỊNH HƯỚNG ĐỀ BGD 2025 MÔN TOÁN 10 - CÁN...Nguyen Thanh Tu Collection
 
Chương 6: Dân tộc - Chủ nghĩa xã hội khoa học
Chương 6: Dân tộc - Chủ nghĩa xã hội khoa họcChương 6: Dân tộc - Chủ nghĩa xã hội khoa học
Chương 6: Dân tộc - Chủ nghĩa xã hội khoa họchelenafalet
 
TỔNG HỢP HƠN 100 ĐỀ THI THỬ TỐT NGHIỆP THPT TIẾNG ANH 2024 - TỪ CÁC TRƯỜNG, ...
TỔNG HỢP HƠN 100 ĐỀ THI THỬ TỐT NGHIỆP THPT TIẾNG ANH 2024 - TỪ CÁC TRƯỜNG, ...TỔNG HỢP HƠN 100 ĐỀ THI THỬ TỐT NGHIỆP THPT TIẾNG ANH 2024 - TỪ CÁC TRƯỜNG, ...
TỔNG HỢP HƠN 100 ĐỀ THI THỬ TỐT NGHIỆP THPT TIẾNG ANH 2024 - TỪ CÁC TRƯỜNG, ...Nguyen Thanh Tu Collection
 
30 ĐỀ PHÁT TRIỂN THEO CẤU TRÚC ĐỀ MINH HỌA BGD NGÀY 22-3-2024 KỲ THI TỐT NGHI...
30 ĐỀ PHÁT TRIỂN THEO CẤU TRÚC ĐỀ MINH HỌA BGD NGÀY 22-3-2024 KỲ THI TỐT NGHI...30 ĐỀ PHÁT TRIỂN THEO CẤU TRÚC ĐỀ MINH HỌA BGD NGÀY 22-3-2024 KỲ THI TỐT NGHI...
30 ĐỀ PHÁT TRIỂN THEO CẤU TRÚC ĐỀ MINH HỌA BGD NGÀY 22-3-2024 KỲ THI TỐT NGHI...Nguyen Thanh Tu Collection
 
Hướng dẫn viết tiểu luận cuối khóa lớp bồi dưỡng chức danh biên tập viên hạng 3
Hướng dẫn viết tiểu luận cuối khóa lớp bồi dưỡng chức danh biên tập viên hạng 3Hướng dẫn viết tiểu luận cuối khóa lớp bồi dưỡng chức danh biên tập viên hạng 3
Hướng dẫn viết tiểu luận cuối khóa lớp bồi dưỡng chức danh biên tập viên hạng 3lamluanvan.net Viết thuê luận văn
 
Quản trị cơ sở Giáo dục nghề nghiệp
Quản trị cơ sở Giáo dục nghề nghiệpQuản trị cơ sở Giáo dục nghề nghiệp
Quản trị cơ sở Giáo dục nghề nghiệpaminh0502
 
C.pptx. Phát hiện biên ảnh trong xử lý ảnh
C.pptx. Phát hiện biên ảnh trong xử lý ảnhC.pptx. Phát hiện biên ảnh trong xử lý ảnh
C.pptx. Phát hiện biên ảnh trong xử lý ảnhBookoTime
 
Mở rộng hoạt động cho vay tiêu dùng tại Ngân hàng TMCP Hàng Hải Việt Nam (Mar...
Mở rộng hoạt động cho vay tiêu dùng tại Ngân hàng TMCP Hàng Hải Việt Nam (Mar...Mở rộng hoạt động cho vay tiêu dùng tại Ngân hàng TMCP Hàng Hải Việt Nam (Mar...
Mở rộng hoạt động cho vay tiêu dùng tại Ngân hàng TMCP Hàng Hải Việt Nam (Mar...lamluanvan.net Viết thuê luận văn
 
Tiểu luận tổng quan về Mối quan hệ giữa chu kỳ kinh tế và đầu tư trong nền ki...
Tiểu luận tổng quan về Mối quan hệ giữa chu kỳ kinh tế và đầu tư trong nền ki...Tiểu luận tổng quan về Mối quan hệ giữa chu kỳ kinh tế và đầu tư trong nền ki...
Tiểu luận tổng quan về Mối quan hệ giữa chu kỳ kinh tế và đầu tư trong nền ki...lamluanvan.net Viết thuê luận văn
 
Hoàn thiện hoạt động kiểm soát rủi ro tín dụng trong cho vay doanh nghiệp tại...
Hoàn thiện hoạt động kiểm soát rủi ro tín dụng trong cho vay doanh nghiệp tại...Hoàn thiện hoạt động kiểm soát rủi ro tín dụng trong cho vay doanh nghiệp tại...
Hoàn thiện hoạt động kiểm soát rủi ro tín dụng trong cho vay doanh nghiệp tại...lamluanvan.net Viết thuê luận văn
 
Giới Thiệu Về Kabala | Hành Trình Thấu Hiểu Bản Thân | Kabala.vn
Giới Thiệu Về Kabala | Hành Trình Thấu Hiểu Bản Thân | Kabala.vnGiới Thiệu Về Kabala | Hành Trình Thấu Hiểu Bản Thân | Kabala.vn
Giới Thiệu Về Kabala | Hành Trình Thấu Hiểu Bản Thân | Kabala.vnKabala
 
TUYỂN TẬP ĐỀ THI GIỮA KÌ, CUỐI KÌ 2 MÔN VẬT LÍ LỚP 11 THEO HÌNH THỨC THI MỚI ...
TUYỂN TẬP ĐỀ THI GIỮA KÌ, CUỐI KÌ 2 MÔN VẬT LÍ LỚP 11 THEO HÌNH THỨC THI MỚI ...TUYỂN TẬP ĐỀ THI GIỮA KÌ, CUỐI KÌ 2 MÔN VẬT LÍ LỚP 11 THEO HÌNH THỨC THI MỚI ...
TUYỂN TẬP ĐỀ THI GIỮA KÌ, CUỐI KÌ 2 MÔN VẬT LÍ LỚP 11 THEO HÌNH THỨC THI MỚI ...Nguyen Thanh Tu Collection
 
TỔNG HỢP HƠN 100 ĐỀ THI THỬ TỐT NGHIỆP THPT TOÁN 2024 - TỪ CÁC TRƯỜNG, TRƯỜNG...
TỔNG HỢP HƠN 100 ĐỀ THI THỬ TỐT NGHIỆP THPT TOÁN 2024 - TỪ CÁC TRƯỜNG, TRƯỜNG...TỔNG HỢP HƠN 100 ĐỀ THI THỬ TỐT NGHIỆP THPT TOÁN 2024 - TỪ CÁC TRƯỜNG, TRƯỜNG...
TỔNG HỢP HƠN 100 ĐỀ THI THỬ TỐT NGHIỆP THPT TOÁN 2024 - TỪ CÁC TRƯỜNG, TRƯỜNG...Nguyen Thanh Tu Collection
 
ĐỀ SỐ 1 Của sở giáo dục đào tạo tỉnh NA.pdf
ĐỀ SỐ 1 Của sở giáo dục đào tạo tỉnh NA.pdfĐỀ SỐ 1 Của sở giáo dục đào tạo tỉnh NA.pdf
ĐỀ SỐ 1 Của sở giáo dục đào tạo tỉnh NA.pdflevanthu03031984
 
GIỮ GÌN VÀ PHÁT HUY GIÁ TRỊ MỘT SỐ BÀI HÁT DÂN CA CÁC DÂN TỘC BẢN ĐỊA CHO HỌC...
GIỮ GÌN VÀ PHÁT HUY GIÁ TRỊ MỘT SỐ BÀI HÁT DÂN CA CÁC DÂN TỘC BẢN ĐỊA CHO HỌC...GIỮ GÌN VÀ PHÁT HUY GIÁ TRỊ MỘT SỐ BÀI HÁT DÂN CA CÁC DÂN TỘC BẢN ĐỊA CHO HỌC...
GIỮ GÌN VÀ PHÁT HUY GIÁ TRỊ MỘT SỐ BÀI HÁT DÂN CA CÁC DÂN TỘC BẢN ĐỊA CHO HỌC...lamluanvan.net Viết thuê luận văn
 
22 ĐỀ THI THỬ TUYỂN SINH TIẾNG ANH VÀO 10 SỞ GD – ĐT THÁI BÌNH NĂM HỌC 2023-2...
22 ĐỀ THI THỬ TUYỂN SINH TIẾNG ANH VÀO 10 SỞ GD – ĐT THÁI BÌNH NĂM HỌC 2023-2...22 ĐỀ THI THỬ TUYỂN SINH TIẾNG ANH VÀO 10 SỞ GD – ĐT THÁI BÌNH NĂM HỌC 2023-2...
22 ĐỀ THI THỬ TUYỂN SINH TIẾNG ANH VÀO 10 SỞ GD – ĐT THÁI BÌNH NĂM HỌC 2023-2...Nguyen Thanh Tu Collection
 
Trắc nghiệm CHƯƠNG 5 môn Chủ nghĩa xã hội
Trắc nghiệm CHƯƠNG 5 môn Chủ nghĩa xã hộiTrắc nghiệm CHƯƠNG 5 môn Chủ nghĩa xã hội
Trắc nghiệm CHƯƠNG 5 môn Chủ nghĩa xã hộiNgocNguyen591215
 
TUYỂN TẬP 50 ĐỀ LUYỆN THI TUYỂN SINH LỚP 10 THPT MÔN TOÁN NĂM 2024 CÓ LỜI GIẢ...
TUYỂN TẬP 50 ĐỀ LUYỆN THI TUYỂN SINH LỚP 10 THPT MÔN TOÁN NĂM 2024 CÓ LỜI GIẢ...TUYỂN TẬP 50 ĐỀ LUYỆN THI TUYỂN SINH LỚP 10 THPT MÔN TOÁN NĂM 2024 CÓ LỜI GIẢ...
TUYỂN TẬP 50 ĐỀ LUYỆN THI TUYỂN SINH LỚP 10 THPT MÔN TOÁN NĂM 2024 CÓ LỜI GIẢ...Nguyen Thanh Tu Collection
 

Recently uploaded (20)

NHững vấn đề chung về Thuế Tiêu thụ đặc biệt.ppt
NHững vấn đề chung về Thuế Tiêu thụ đặc biệt.pptNHững vấn đề chung về Thuế Tiêu thụ đặc biệt.ppt
NHững vấn đề chung về Thuế Tiêu thụ đặc biệt.ppt
 
ĐỀ KIỂM TRA CUỐI KÌ 2 BIÊN SOẠN THEO ĐỊNH HƯỚNG ĐỀ BGD 2025 MÔN TOÁN 10 - CÁN...
ĐỀ KIỂM TRA CUỐI KÌ 2 BIÊN SOẠN THEO ĐỊNH HƯỚNG ĐỀ BGD 2025 MÔN TOÁN 10 - CÁN...ĐỀ KIỂM TRA CUỐI KÌ 2 BIÊN SOẠN THEO ĐỊNH HƯỚNG ĐỀ BGD 2025 MÔN TOÁN 10 - CÁN...
ĐỀ KIỂM TRA CUỐI KÌ 2 BIÊN SOẠN THEO ĐỊNH HƯỚNG ĐỀ BGD 2025 MÔN TOÁN 10 - CÁN...
 
Chương 6: Dân tộc - Chủ nghĩa xã hội khoa học
Chương 6: Dân tộc - Chủ nghĩa xã hội khoa họcChương 6: Dân tộc - Chủ nghĩa xã hội khoa học
Chương 6: Dân tộc - Chủ nghĩa xã hội khoa học
 
TỔNG HỢP HƠN 100 ĐỀ THI THỬ TỐT NGHIỆP THPT TIẾNG ANH 2024 - TỪ CÁC TRƯỜNG, ...
TỔNG HỢP HƠN 100 ĐỀ THI THỬ TỐT NGHIỆP THPT TIẾNG ANH 2024 - TỪ CÁC TRƯỜNG, ...TỔNG HỢP HƠN 100 ĐỀ THI THỬ TỐT NGHIỆP THPT TIẾNG ANH 2024 - TỪ CÁC TRƯỜNG, ...
TỔNG HỢP HƠN 100 ĐỀ THI THỬ TỐT NGHIỆP THPT TIẾNG ANH 2024 - TỪ CÁC TRƯỜNG, ...
 
30 ĐỀ PHÁT TRIỂN THEO CẤU TRÚC ĐỀ MINH HỌA BGD NGÀY 22-3-2024 KỲ THI TỐT NGHI...
30 ĐỀ PHÁT TRIỂN THEO CẤU TRÚC ĐỀ MINH HỌA BGD NGÀY 22-3-2024 KỲ THI TỐT NGHI...30 ĐỀ PHÁT TRIỂN THEO CẤU TRÚC ĐỀ MINH HỌA BGD NGÀY 22-3-2024 KỲ THI TỐT NGHI...
30 ĐỀ PHÁT TRIỂN THEO CẤU TRÚC ĐỀ MINH HỌA BGD NGÀY 22-3-2024 KỲ THI TỐT NGHI...
 
Hướng dẫn viết tiểu luận cuối khóa lớp bồi dưỡng chức danh biên tập viên hạng 3
Hướng dẫn viết tiểu luận cuối khóa lớp bồi dưỡng chức danh biên tập viên hạng 3Hướng dẫn viết tiểu luận cuối khóa lớp bồi dưỡng chức danh biên tập viên hạng 3
Hướng dẫn viết tiểu luận cuối khóa lớp bồi dưỡng chức danh biên tập viên hạng 3
 
Quản trị cơ sở Giáo dục nghề nghiệp
Quản trị cơ sở Giáo dục nghề nghiệpQuản trị cơ sở Giáo dục nghề nghiệp
Quản trị cơ sở Giáo dục nghề nghiệp
 
C.pptx. Phát hiện biên ảnh trong xử lý ảnh
C.pptx. Phát hiện biên ảnh trong xử lý ảnhC.pptx. Phát hiện biên ảnh trong xử lý ảnh
C.pptx. Phát hiện biên ảnh trong xử lý ảnh
 
Mở rộng hoạt động cho vay tiêu dùng tại Ngân hàng TMCP Hàng Hải Việt Nam (Mar...
Mở rộng hoạt động cho vay tiêu dùng tại Ngân hàng TMCP Hàng Hải Việt Nam (Mar...Mở rộng hoạt động cho vay tiêu dùng tại Ngân hàng TMCP Hàng Hải Việt Nam (Mar...
Mở rộng hoạt động cho vay tiêu dùng tại Ngân hàng TMCP Hàng Hải Việt Nam (Mar...
 
Tiểu luận tổng quan về Mối quan hệ giữa chu kỳ kinh tế và đầu tư trong nền ki...
Tiểu luận tổng quan về Mối quan hệ giữa chu kỳ kinh tế và đầu tư trong nền ki...Tiểu luận tổng quan về Mối quan hệ giữa chu kỳ kinh tế và đầu tư trong nền ki...
Tiểu luận tổng quan về Mối quan hệ giữa chu kỳ kinh tế và đầu tư trong nền ki...
 
Hoàn thiện hoạt động kiểm soát rủi ro tín dụng trong cho vay doanh nghiệp tại...
Hoàn thiện hoạt động kiểm soát rủi ro tín dụng trong cho vay doanh nghiệp tại...Hoàn thiện hoạt động kiểm soát rủi ro tín dụng trong cho vay doanh nghiệp tại...
Hoàn thiện hoạt động kiểm soát rủi ro tín dụng trong cho vay doanh nghiệp tại...
 
Giới Thiệu Về Kabala | Hành Trình Thấu Hiểu Bản Thân | Kabala.vn
Giới Thiệu Về Kabala | Hành Trình Thấu Hiểu Bản Thân | Kabala.vnGiới Thiệu Về Kabala | Hành Trình Thấu Hiểu Bản Thân | Kabala.vn
Giới Thiệu Về Kabala | Hành Trình Thấu Hiểu Bản Thân | Kabala.vn
 
TUYỂN TẬP ĐỀ THI GIỮA KÌ, CUỐI KÌ 2 MÔN VẬT LÍ LỚP 11 THEO HÌNH THỨC THI MỚI ...
TUYỂN TẬP ĐỀ THI GIỮA KÌ, CUỐI KÌ 2 MÔN VẬT LÍ LỚP 11 THEO HÌNH THỨC THI MỚI ...TUYỂN TẬP ĐỀ THI GIỮA KÌ, CUỐI KÌ 2 MÔN VẬT LÍ LỚP 11 THEO HÌNH THỨC THI MỚI ...
TUYỂN TẬP ĐỀ THI GIỮA KÌ, CUỐI KÌ 2 MÔN VẬT LÍ LỚP 11 THEO HÌNH THỨC THI MỚI ...
 
Trích dẫn theo Harvard với Microsoft Word
Trích dẫn theo Harvard với Microsoft WordTrích dẫn theo Harvard với Microsoft Word
Trích dẫn theo Harvard với Microsoft Word
 
TỔNG HỢP HƠN 100 ĐỀ THI THỬ TỐT NGHIỆP THPT TOÁN 2024 - TỪ CÁC TRƯỜNG, TRƯỜNG...
TỔNG HỢP HƠN 100 ĐỀ THI THỬ TỐT NGHIỆP THPT TOÁN 2024 - TỪ CÁC TRƯỜNG, TRƯỜNG...TỔNG HỢP HƠN 100 ĐỀ THI THỬ TỐT NGHIỆP THPT TOÁN 2024 - TỪ CÁC TRƯỜNG, TRƯỜNG...
TỔNG HỢP HƠN 100 ĐỀ THI THỬ TỐT NGHIỆP THPT TOÁN 2024 - TỪ CÁC TRƯỜNG, TRƯỜNG...
 
ĐỀ SỐ 1 Của sở giáo dục đào tạo tỉnh NA.pdf
ĐỀ SỐ 1 Của sở giáo dục đào tạo tỉnh NA.pdfĐỀ SỐ 1 Của sở giáo dục đào tạo tỉnh NA.pdf
ĐỀ SỐ 1 Của sở giáo dục đào tạo tỉnh NA.pdf
 
GIỮ GÌN VÀ PHÁT HUY GIÁ TRỊ MỘT SỐ BÀI HÁT DÂN CA CÁC DÂN TỘC BẢN ĐỊA CHO HỌC...
GIỮ GÌN VÀ PHÁT HUY GIÁ TRỊ MỘT SỐ BÀI HÁT DÂN CA CÁC DÂN TỘC BẢN ĐỊA CHO HỌC...GIỮ GÌN VÀ PHÁT HUY GIÁ TRỊ MỘT SỐ BÀI HÁT DÂN CA CÁC DÂN TỘC BẢN ĐỊA CHO HỌC...
GIỮ GÌN VÀ PHÁT HUY GIÁ TRỊ MỘT SỐ BÀI HÁT DÂN CA CÁC DÂN TỘC BẢN ĐỊA CHO HỌC...
 
22 ĐỀ THI THỬ TUYỂN SINH TIẾNG ANH VÀO 10 SỞ GD – ĐT THÁI BÌNH NĂM HỌC 2023-2...
22 ĐỀ THI THỬ TUYỂN SINH TIẾNG ANH VÀO 10 SỞ GD – ĐT THÁI BÌNH NĂM HỌC 2023-2...22 ĐỀ THI THỬ TUYỂN SINH TIẾNG ANH VÀO 10 SỞ GD – ĐT THÁI BÌNH NĂM HỌC 2023-2...
22 ĐỀ THI THỬ TUYỂN SINH TIẾNG ANH VÀO 10 SỞ GD – ĐT THÁI BÌNH NĂM HỌC 2023-2...
 
Trắc nghiệm CHƯƠNG 5 môn Chủ nghĩa xã hội
Trắc nghiệm CHƯƠNG 5 môn Chủ nghĩa xã hộiTrắc nghiệm CHƯƠNG 5 môn Chủ nghĩa xã hội
Trắc nghiệm CHƯƠNG 5 môn Chủ nghĩa xã hội
 
TUYỂN TẬP 50 ĐỀ LUYỆN THI TUYỂN SINH LỚP 10 THPT MÔN TOÁN NĂM 2024 CÓ LỜI GIẢ...
TUYỂN TẬP 50 ĐỀ LUYỆN THI TUYỂN SINH LỚP 10 THPT MÔN TOÁN NĂM 2024 CÓ LỜI GIẢ...TUYỂN TẬP 50 ĐỀ LUYỆN THI TUYỂN SINH LỚP 10 THPT MÔN TOÁN NĂM 2024 CÓ LỜI GIẢ...
TUYỂN TẬP 50 ĐỀ LUYỆN THI TUYỂN SINH LỚP 10 THPT MÔN TOÁN NĂM 2024 CÓ LỜI GIẢ...
 

Xap xỉ diophantine và phân so liên tục trong giải Phương trình pell.docx

  • 1. ĐẠI HOC THÁI NGUYÊN TRƯ NG ĐẠI HOC KHOA HOC .................................................. Tải tài liệu tại sividoc.com Viết đề tài giá sinh viên – ZALO:0973.287.149-TEAMLUANVAN.COM Nguyen Thị Tuyet Mai XAP XỈ DIOPHANTINE VÀ PHÂN SO LIÊN TỤC TRONG GIẢI PHƯƠNG TRÌNH PELL LU N VĂN THẠC SĨ TOÁN HOC Thái Nguyên - 2017
  • 2. ĐẠI HOC THÁI NGUYÊN TRƯ NG ĐẠI HOC KHOA HOC .................................................. Tải tài liệu tại sividoc.com Viết đề tài giá sinh viên – ZALO:0973.287.149-TEAMLUANVAN.COM Nguyen Thị Tuyet Mai XAP XỈ DIOPHANTINE VÀ PHÂN SO LIÊN TỤC TRONG GIẢI PHƯƠNG TRÌNH PELL Chuyên ngành: PHƯƠNG PHÁP TOÁN SƠ CAP Mã so : 60 46 01 13 LU N VĂN THẠC SĨ TOÁN HOC NGƯ I HƯ NG DȀN KHOA HOC: TS. NGUYEN ĐÌNH BÌNH Thái Nguyên - 2017
  • 3. i Viết đề tài giá sinh viên – ZALO:0973.287.149-TEAMLUANVAN.COM Mnc lnc L I CẢM ƠN iii M ĐAU i 1 PHƯƠNG TRÌNH PELL 1 1.1. M®t so khái ni m và ket quả ve phương trình Pell . . . . . . . 1 1.1.1. Phương trình Pell Loại I . . . . . . . . . . . . . . . . . 1 1.1.2. Phương trình Pell Loại II . . . . . . . . . . . . . . . . 3 1.1.3. Phương trình Pell với tham so n . . . . . . . . . . . . . 4 1.2. Phân so liên tục - Phân so liên tục tőng quát - Phân so liên tục đơn giản . . . . . . . . . . . . . . . . . . . . . . . . . . . . 7 1.2.1. M®t trường hợp của phương trình Pell . . . . . . . . . 7 1.2.2. Phân so liên tục ..................................................................... 18 1.3. Bài toán áng dụng.......................................................................... 29 2 XAP XỈ DIOPHANTINE, M R NG PHƯƠNG TRÌNH PELL VÀ ỨNG DỤNG 35 2.1. Chu kì của phân so liên tục............................................................ 35 2.1.1. Bő đe chính......................................................................... 36 2.1.2. Chu kì phân so liên tục....................................................... 40 2.2. Xap xỉ Diophantine và phân so liên tục đơn giản......................... 46 2.2.1. Phân so liên tục đơn giản của √ D .......................................46
  • 4. ii Viết đề tài giá sinh viên – ZALO:0973.287.149-TEAMLUANVAN.COM 2.2.2. Xap xỉ Diophantine và phân so liên tục đơn giản..............50 2.3. Ve m®t tiêu chuȁn cho sự ton tại nghi m của phương trình Pell 54 2.4. M®t so mở r®ng của xap xỉ Diophantine.......................................55 2.4.1. Tiêu chí vô t.......................................................................55 2.4.2. Bat đȁng thác Liouville ...................................................59 2.4.3. Bat đȁng thác Liouville b c hai .......................................60 2.5. M®t áng dụng giải phương trình Pell âm......................................62 Tài li u tham khảo 72
  • 5. iii Viết đề tài giá sinh viên – ZALO:0973.287.149-TEAMLUANVAN.COM L I CẢM ƠN Được sự phân công của khoa Toán- Tin, trường Đại hoc Khoa Hoc Thái Nguyên và sự đong ý của thay giáo hướng dan TS. Nguyen Đình Bình, tôi đã thực hi n đe tài "Xap xỉ Diophantine và phân so liên tục trong giải phương trình Pell". Đe hoàn thành lu n này, tôi xin chân thành cảm ơn Ban Giám Hi u, khoa Toán - Tin và phòng đào tạo của trường Đại hoc Khoa Hoc Thái Nguyên. Tôi xin trân trong cảm ơn các thay cô giáo đã t n tình hướng dan, giảng dạy trong suot quá trình hoc t p, nghiên cáu và rèn luy n ở trường Đại hoc Khoa Hoc Thái Nguyên. Xin chân thành cảm ơn thay giáo hướng dan TS. Nguyen Đình Bình đã t n tình, chu đáo hướng dan tôi hoàn thành lu n văn này. Dù rat b n r®n trong công vi c, song thay van dành nhieu thời gian và tâm huyet hướng dan, đ®ng viên, khuyen khích tôi trong quá trình nghiên cáu đe tài. Cuoi cùng tôi xin chân thành cảm ơn đen gia đình, bạn bè, nhǎng người không ngàng đ®ng viên, ho trợ tạo moi đieu ki n tot nhat cho tôi trong suot thời gian hoc t p và nghiên cáu lu n văn. Tôi xin trân trong cảm ơn! Thái Nguyên, ngày ... tháng ... năm ... Tác giả Nguyen Thị Tuyet Mai
  • 6. i Viết đề tài giá sinh viên – ZALO:0973.287.149-TEAMLUANVAN.COM M ĐAU 1. Lý do chon đe tài Trong lịch sả phát trien của So hoc, phương trình Pell được biet đen là m®t phương trình női tieng trong dạng toán ve phương trình nghi m nguyên. Phương trình Pell được phát minh cách đây 1000 năm ở An Đ® cő đại bởi Brahmaguta. Trong nhieu năm sau đó, các nhà toán hoc bat đau nghiên cáu tìm lời giải cho phương trình này. Đen năm 1770, Lagrange đã phát trien lí thuyet tőng quát ve phương trình dựa trên phân so liên tục. Bên cạnh đó, các nhà toán hoc lớn như Legendre(1798), É. Borel(1903) cũng quan tâm nghiên cáu và có nhieu đóng góp cho vi c hoàn thi n và phát trien phương trình Pell. Ngày nay rat nhieu tài li u nghiên cáu sâu ve phương trình Pell ra đời như: Computational aspects of number theory( H. Cohen, 2001), The higher arithmetic (H. Davenport, 2008), Solving the Pell equation (M.J.Jacobson, Jr and H.C.Williams, 2009) tham khảo trong tài li u [4]. Tuy có rat nhieu công trình nghiên cáu ve phương trình Pell cũng như phương trình nghi m nguyên, song đó van còn là m®t ȁn so thách thác các nhà toán hoc cũng như các bạn trẻ yêu thích môn toán. Có the nói, phương trình Pell khá phong phú và đa dạng ve lịch sả ra đời, định nghĩa, trong phương pháp giải và cả áng dụng của nó trong So hoc. Bản thân nó đóng góp nhieu áng dụng trong vi c giải các bài toán ve So hoc hay và khó. Nhieu bài toán ve phương trình Pell qua các kì thi Olimpic toán quoc
  • 7. ii Viết đề tài giá sinh viên – ZALO:0973.287.149-TEAMLUANVAN.COM te, khu vực và trong nước ngày càng mới lạ thu hút sự quan tâm cũng như thách thác trí tu , sáng tạo của moi bạn trẻ. Và đe giải nó không nhǎng can nam được lí thuyet mà còn can các kĩ năng. Tuy nhiên hi n nay các bạn hoc sinh, đ c bi t là các bạn hoc sinh lớp chuyên, lớp chon còn biet rat ít ve dạng phương trình Pell. Đ c bi t, chúng ta có rat ít sách ve phương trình Pell và áng dụng của nó, chủ yeu là tham khảo các tài li u, bài báo nước ngoài. Do v y, dưới sự góp ý của thay hướng dan TS. Nguyen Đình Bình, tác giả chon đe tài “Xap xỉ Diophantine và phân so liên tục trong giải phương trình Pell”. Do phương trình Pell không còn là đe tài mới nên trong lu n văn tác giả sě trình bày ngan gon các ket quả và ví dụ ve phương trình Pell cơ bản, xap xỉ Diophantine và phân so liên tục trong giải phương trình Pell. Đong thời lu n văn cũng phân tích mở r®ng phương trình và áng dụng của nó. Do thời gian có hạn và trình đ® còn hạn che nên lu n văn chỉ dàng lại ở vi c trình bày ket quả nghiên cáu ve xap xỉ Diophantine và phân so liên tục trong giải phương trình Pell, giới thi u áng dụng giải phương trình Pell âm. 2. Mnc tiêu nghiên cfíu của lu n văn Mục tiêu của lu n văn là nghiên cáu ve phương trình Pell cơ bản, nghiên cáu xap xỉ Diophantine, phân so liên tục trong giải phương trình Pell. Đong thời lu n văn cũng phân tích mở r®ng của phương trình Pell và áng ụng của nó. 3. Đoi tư ng và phạm vi nghiên cfíu - Đoi tượng nghiên cáu của lu n văn là phương trình Pell. - Phạm vi nghiên cáu của lu n văn là giới thi u xap xỉ Diophantine và phân so liên tục trong giải phương trình Pell, áng dụng giải phương trình Pell âm.
  • 8. iii Viết đề tài giá sinh viên – ZALO:0973.287.149-TEAMLUANVAN.COM 4. Phương pháp nghiên cfíu - Đoc sách liên quan đen đe tài và tìm kiem tài li u. - Đoc, hieu và dịch tài li u tà tieng Anh sang tieng Vi t. - Sả dụng phương pháp tőng quát đe h thong và trình bày các ket quả chính trong các tài li u tham khảo. 5. Bo cnc lu n văn Ngoài phan mở đau và ket lu n, n®i dung của lu n văn chia thành 2 chương: - Chương 1 trình bày m®t so khái ni m và ket quả của phương trình Pell cơ bản, h thong lí thuyet ve phân so liên tục. - Chương 2 trình bày ve xap xỉ Diophantine, phân so liên tục đơn giản trong giải phương trình Pell và áng dụng trong giải phương trình Pell. Chương 2 là chương trong tâm của lu n văn.
  • 9. 1 Viết đề tài giá sinh viên – ZALO:0973.287.149-TEAMLUANVAN.COM Chương 1 PHƯƠNG TRÌNH PELL Trong chương này tác giả sě trình bày m®t so khái ni m và ket quả ve phương trình Pell cơ bản, phân so liên tục. Đong thời tác giả trình bày m®t so bài t p áng dụng là các bài toán trong các kì thi hoc sinh giỏi các năm được chon loc. N®i dung chính được tham khảo tà các tài li u tham khảo [1], [2], [3], [4]. 1.1. M t so khái ni m và ket quả ve phương trình Pell Trong mục này, tác giả sě đưa ra h thong các định nghĩa và định lí ve công thác nghi m của phương trình Pell cơ bản, cùng m®t so ví dụ có kèm lời giải cho tàng loại phương trình Pell. N®i dung chính được tham khảo trong tài li u [1], [3]. 1.1.1. Phương trình Pell Loại I Phương trình Pell loại I là phương trình có dạng: x2 − Dy2 = 1, ( trong đó D là so nguyên dương). (1.1) Định lí 1.1. 1. Neu D là so chính phương, D = m2 , m ∈ Z thì (1.1) không có nghi m
  • 10. 2 Viết đề tài giá sinh viên – ZALO:0973.287.149-TEAMLUANVAN.COM nguyên dương. 2. Neu D là so nguyên âm thì (1.1) không có nghi m nguyên dương. 3. Phương trình (1.1) có nghi m nguyên dương khi và chỉ khi D là so nguyên dương và không chính phương. Định lí 1.2. Giả sả (a, b) là nghi m nhỏ nhat của phương trình x2 − Dy2 = 1 nghĩa là b là so nguyên bé nhat đe 1 + Db2 là so chính phương. Xét dãy xn và yn cho bởi h thác truy hoi sau: x0 = 1, x1 = a, xn+2 = 2axn+1 − xn, n = 0, 1, ... y0 = 0, y1 = b, yn+2 = 2ayn+1 − yn, n = 0, 1, ... (1.2) Khi đó (xn, yn) là tat cả các nghi m nguyên dương của phương trình Pell x2 − Dy2 = 1. Định lí 1.3. Cho phương trình Pell x2 − Dy2 = 1. Goi r là chu kì bieu dien phân so liên tục √ D, pk qk là phân so đơn giản thá k của √ D. Neu r chȁn thì tat cả các nghi m của phương trình Pell là: x = pkr− 1, y = qkr− 1. Neu r lẻ thì tat cả các nghi m của phương trình Pell là: x = p2tr− 1, y = q2tr− 1, t ∈ N∗ . Lưu ý. Neu r là so chȁn thì (pr− 1, qr− 1) là nghi m nhỏ nhat. Neu r là so lẻ thì (p2r− 1, q2r− 1) là nghi m nhỏ nhat. Ví dn 1.1. Giải phương trình nghi m nguyên: x2 − 7y2 = 1. Lời giải. Ta có √ 7 = [2, 1, 1, 1, 4]. Chu kì r = 4 là so chȁn. V y ta có nghi m nhỏ nhat là (8;3).
  • 11. 3 Viết đề tài giá sinh viên – ZALO:0973.287.149-TEAMLUANVAN.COM V y tat cả nghi m nguyên dương của phương trình xác định theo công thác: x0 = 1, x1 = 8, xn+2 = 16xn+1 − xn, n = 0, 1, ... y0 = 0, y1 = 3, yn+2 = 16yn+1 − yn, n = 0, 1, ... 1.1.2. Phương trình Pell Loại II Phương trình Pell loại II có dạng: x2 − Dy2 = − 1, (1.3) với D là so nguyên dương . Cũng giong như khi xét phương trình Pell loại I, ở đây ta chỉ quan tâm nghi m nguyên dương của phương trình loại II. Định lí 1.4. Phương trình Pell loại II không có nghi m nguyên dương khi D = m2 , m ∈ Z (khi D là so chính phương). Định lí 1.5. Phương trình Pell loại II không có nghi m nguyên dương khi D có ước nguyên to p = 4k + 3. Định lí 1.6. Neu D là so nguyên to thì phương trình Pell loại II có nghi m nguyên dương khi và chỉ khi D không có dạng 4k + 3. Định lí 1.7. (Đieu ki n đe phương trình Pell loại II có nghi m) Goi (a, b) là nghi m nhỏ nhat của phương trình Pell loại I liên ket với phương trình Pell loại II. Khi đó phương trình Pell loại II có nghi m khi và chỉ khi h (I) sau: có nghi m nguyên dương. a = x2 + Dy2 b = 2xy (1.4) Định lí 1.8. Công thác nghi m của phương trình Pell loại II. Xét phương trình Pell loại II (1.3). Cùng với đó xét phương trình Pell loại I (1.1) liên ket với nó. Giả sả (a, b) là nghi m nguyên bé nhat của (1.1). Xét h phương trình (I). Khi đó lay hai phương trình trong h (I) trà ve với ve
  • 12. 4 Viết đề tài giá sinh viên – ZALO:0973.287.149-TEAMLUANVAN.COM √ − 2uv = 2 có nghi m và (u, v) là nghi m duy nhat của nó. Xét hai dãy so nguyên dương {xn}, {yn} sau đây: x0 = u, x1 = u3 + 3Duv2 , xn+2 = 2axn+1 − xn, n = 0, 1, ... y0 = v, y1 = Dv3 + 3u2 v, yn+2 = 2ayn+1 − yn, n = 0, 1, ... Khi đó (xn, yn) là ngi m nguyên dương của phương trình Pell loại II. Định lí 1.9. Phương trình x2 − Dy2 = − 1 có nghi m khi và chỉ khi chukì r của bieu dien phân so liên tục của D là so lẻ. Trong trường hợp ay các nghi m của nó là x = p(2tr− r− 1), y = q(2tr− r− 1) với t = 1, 2, 3.... Ví dn 1.2. Xét phương trình x2 − 34y2 = − 1. Lời giải. Ta có: √ 34 = [5; 1, 4, 1, 10]. Chu kỳ r = 4 là so chȁn. V y phương trình vô nghi m. Ví dn 1.3. Giải phương trình: x2 − 2y2 = − 1. Lời giải. Phương trình Pell loại (II) liên ket x2 2y2 = 1. Ta có: √ 2 = [1; 2]. Có chu kỳ r = 1. Có nghi m nhỏ nhat (3; 2). Xét h phương trình: u2 + 2v2 = 3 De dàng thay (u, v) = (1; 1) là nghi m dương bé nhat của nó. Theo công thác nghi m ở định lí 1.8, thì phương trình Pell loại II x2 − 2y2 = − 1 có nghi m là: x0 = 1; x1 = 7; xn+2 = 6xn+1 − xn... y0 = 1; y1 = 5; yn+2 = 6yn+1 − yn... 1.1.3. Phương trình Pell v i tham so n Xét phương trình x2 − Dy2 = n, ở đây D là so nguyên dương và khôngchính phương, còn n là so nguyên. Phương trình này goi là phương trình Pell
  • 13. 5 Viết đề tài giá sinh viên – ZALO:0973.287.149-TEAMLUANVAN.COM với tham so n. Neu n = 1 ho c n = − 1 thì ta có phương trình Pell loại I và loại II. Định lj 1.10. Xét phương trình Pell với tham so n: x2 − Dy2 = n. (1.5) Phương trình (1.5) ho c vô nghi m ho c có vô so nghi m. V y đe tìm ra công thác tőng quát tat cả các nghi m của phương trình Pell có tham so n ta có các định lí sau: Định lj 1.11. Xét phương trình Pell với tham so n (1.5). Goi (x0, y0) là nghi m nguyên dương nhỏ nhat của (1.5). Ta có: y0 ≤ max{nb2 ; na2 D }, ở đây (a, b) là nghi m bé nhat của phương trình Pell loại I (1.1) áng với (1.5). Định lj 1.12. Xét phương trình Pell với tham so n (1.5). Giả sả (1.5) có nghi m và (α1, β1), (α2, β2), ..., (αm, βm) là tat cả các nghi m của (1.5) thỏa mãn bat đȁng thác 2 2 − na2 βi ≤ max{nb , D }. Xét m dãy sau đây. Dãy thá i : {xn,i, yn,i}, với i = 1, m được xác định như sau: x0,i = αi, y0,i = βi, xn+1,i = xn,ia + Dyn,ib, yn+1, i = xn,ib + yn.ia, ở đây (a, b) là nghi m bé nhat của phương trình Pell loại I (1.1) áng với (1.5). Khi đó các dãy nghi m {xn,i; yn,i} sě là tat cả các nghi m nguyên dương của phương trình Pell với tham so n. Ví dn 1.4. Giải phương trình Pell x2 − 5y2 = − 4. Lời giải. −
  • 14. 6 Viết đề tài giá sinh viên – ZALO:0973.287.149-TEAMLUANVAN.COM − ≤ 2 2 Xét phương trình Pell với tham so n = − 4 sau đây x2 − 5y2 = − 4. (i) Phương trình Pell loại I liên ket với nó có dạng: x2 − 5y2 = 1. (ii) Phương trình (ii) có nghi m nguyên dương nhỏ nhat là (a, b) = (9, 4). Khi đó: max{nb2 ; na2 2 d } = max{− 4.4 ; 4.92 5 } = 4.81 . 5 So nguyên dương β lớn nhat thỏa mãn β2 4.81 5 là β = 8. Xét phương trình (i): Neu y = 1 ⇒ x = 1; y = 2 ⇒ x = 4; y = 3; 4; 7; 8 thì (i) không dan đen x nguyên; y = 5 ⇒ x = 11. Như the bang cách thả trực tiep nói trên, ta thay có 3 nghi m (1, 1); (4, 2); (11, 5) của phương trình (i) thỏa mãn đieu ki n: β ≤ max{nb ; na2 D } Theo định lý 1.12, phương trình Pell áng với n = − 4 có 3 dãy nghi m: x0,1 = 1; y0,1 = 1; xn+1,1 = 9xn,1 + 20yn,1; yn+1,1 = 4xn,1 + 9yn,1 x0,2 = 4; y0,2 = 2; xn+1,2 = 9xn,2 + 20yn,2; yn+1,2 = 4xn,2 + 9yn,2 x0,3 = 11; y0,3 = 5; xn+1,3 = 9xn,3 + 20yn,3; yn+1,3 = 4xn,3 + 9yn,3 Ba dãy này vét het tat cả các nghi m của phương trình (i). Ket lu n: Tác giả đã trình bày m®t cách h thong khái ni m và m®t so ket quả của phương trình Pell cơ bản. Moi dạng của phương trình Pell, tác giả đã giới thi u m®t vài ví dụ đe làm sáng tỏ công thác nghi m, tà đó ta có the mở r®ng các bài toán khó tà bài toán tìm nghi m thông thường. −
  • 15. 7 Viết đề tài giá sinh viên – ZALO:0973.287.149-TEAMLUANVAN.COM 1 1.2. Phân so liên tnc - Phân so liên tnc tong quát - Phân so liên tnc đơn giản Trong mục này tác giả sě trình bày h thong ve lí thuyet phân so liên tục, cụ the hơn là phân so liên tục tőng quát, phân so liên tục đơn giản liên quan đen phương trình Pell. N®i dung chính được tham khảo trong tài li u [4]. 1.2.1. M t trư ng h p của phương trình Pell Cho D là m®t so nguyên dương không chính phương. √ D là so vô tỉ. Phương trình Diophantine có dạng: x2 − Dy2 = ± 1, (1.6) trong đó ȁn x, y ∈ Z được goi là phương trình Pell. 1.2.1.1. Ví dn ve phân so liên tnc đơn giản Ví dn 1.5. Cho D = a2 b2 + 2b, ở đó a, b là các so nguyên dương. M®t nghi m của x2 − (a2 b2 + 2b)y2 = 1, là c p (x, y) = (a2 b + 1, a). Ta thay dạng khai trien phân so liên tục đơn giản của √ D là √ a2b2 + 2b = [ab, a, 2ab]. Tà đó t = √ a2b2 + 2b ⇔ t = ab + 1 . a + t + ab Có the xét ví dụ tương tự D = a2 + 2 (đ t b = 1) và D = b2 + 2b (đ t a = 1). Cho a = 1 và b = c − 1, ta cũng được ví dụ D = c2 − 1.
  • 16. 8 Viết đề tài giá sinh viên – ZALO:0973.287.149-TEAMLUANVAN.COM ⇔ ⇔ ≡ Ví dn 1.6. Đ t D = a2 b2 + b, ở đó a, b là các so nguyên dương. M®t nghi m của x2 − (a2 b2 + b)y2 = 1, là c p (x, y) = (a2 b + 1, 2a). Dạng khai trien phân so liên tục của √ D là √ a2b2 + b = [ab, 2a, 2ab]. Tà đó t = √ a2b2 + b t = ab + 1 . 1 2a + t + ab Xét ví dụ tương tự D = b2 + b (đ t a = 1). Trường hợp b = 1, D = a2 + 1 là trường hợp đ c bi t. M®t nghi m nguyên của phương trình x2 − (a2 + 1)y2 = − 1, là (x, y) = (a, 1). Dạng khai trien phân so liên tục của √ D là √ a2 + 1 = [a, 2a]. V y t = √ a2 + 1 t = a + 1 . t + a Ví dn 1.7. Cho a, b là hai so nguyên dương sao cho b2 + 1 chia 2ab + 1. Ví dụ b = 2 và a 1 (mod 5). Viet 2ab + 1 = k(b2 + 1) và đ t D = a2 + k. Dạng khai trien phân so liên tục của √ D là [a, b, b, 2a]. Suy ra t = √ D thỏa mãn 1 t = a + 1 b + 1 b + = [a, b, b, a + t]. a + t M®t nghi m của phương trình x2 − Dy2 = − 1 là x = ab2 + a + b, y = b2 + 1. Trong trường hợp a = 1 và b = 2 (vì v y k = 1), phân so liên tục có duy nhat chieu dài chu kì 1 : √ 5 = [1, 2].
  • 17. 9 Viết đề tài giá sinh viên – ZALO:0973.287.149-TEAMLUANVAN.COM .√ D − x. < 1 . y . . Ba ví dụ trên đây là nhǎng trường hợp đ c bi t bởi O.Per-ron và liên quan đen các trường b c hai thực dạng Richard - Degert. 1.2.1.2. Sfi ton tại của nghi m nguyên Cho D là so nguyên dương không chính phương. Ta can cháng minh rang phương trình Pell (1.6) có m®t nghi m không nhỏ (x, y) ∈ Z × Z, đó là m®t nghi m (± 1, 0). M nh đe 1.1. Cho D là so nguyên dương không chính phương, ton tại (x, y) ∈ Z2 với x > 0 và y > 0 sao cho x2 − Dy2 = 1. Chúng minh. Bước đau tiên của cháng minh là chỉ ra rang ton tại m®t so nguyên khác không k sao cho phương trình Pell x2 − Dy2 = k có vô so nghi m (x, y) ∈ Z × Z, liên h các nghi m nguyên của phương trình Pell với xap xỉ hǎu t x/y của √ D. Ta thay √ D là so vô t , ta suy ra có vô so (x, y) ∈ Z × Z với y > 0 (và do đó x > 0) thỏa mãn: y2 Đoi với moi c p (x, y) như v y ta có: 0 < x < y √ D + 1 < y( √ D + 1). Do đó, 0 < |x2 − Dy2 | = |x − y √ D|.|x + y √ D| < 2 √ D + 1. Tà đó có duy nhat so nguyên k /= 0 trong khoảng − (2 √ D + 1) < k < 2 √ D + 1. M®t trong so các dạng của chúng là x2 − Dy2 có vô so c p (x, y). Bước thá hai nh n thay, các t p con của (x, y) (mod k) trong (Z/kZ)2 là hǎu hạn, là m®t t p con vô hạn E ⊂ Z × Z của các nghi m cho x2 − Dy2 = k
  • 18. 10 Viết đề tài giá sinh viên – ZALO:0973.287.149-TEAMLUANVAN.COM = ( 1 1 2 2 có cùng (x (mod k), y (mod k)). Cho (u1, v1) và (u2, v2) là hai thành phan riêng bi t trong E. Xác định (x, y) ∈ Q2 bởi √ u1 + v1 √ D x + y Tà u2 − Dv2 = k ta suy ra D = u2 + v2 √ D . 2 2 Do đó, x + y √ D 1 u k 1 + v1 √ D)(u — v2 √ D). x = u1u2 − Dv1v2 , y = − u1v2 + u2v1 . k k Tà u1 ≡ u2 (mod k), v1 ≡ v2 (mod k) và u2 − Dv2 = k, u2 − Dv2 = k. Ta suy ra 1 1 2 2 u1u2 − Dv1v2 ≡ u2 − Dv2 ≡ 0 (mod k), 1 1 và − u1v2 + u2v1 ≡ − u1v1 + u1v1 ≡ 0 (mod k). Do đó x, y ∈ Z. Hơn nǎa x2 − Dy2 = (x − y √ D)(x + y √ D) (u1 − v1 √ D)(u1 + v1 √ D) = √ √ (u2 − v2 D)(u2 + v2 D) u2 − Dv2 u2 − Dv2 = 1. Đây là đieu ki n đe kiem tra y /= 0. Neu y = 0 thì x = ± 1, u1v2 = u2v1, u1u2 − Dv1v2 = ± 1, và ku1 = ± u1(u1u2 − Dv1v2) = ± u2(u2 − Dv2 ) = ± ku2, 1 1 mở r®ng (u1, u2) = (v1, v2), mâu thuan. Cuoi cùng, neu x < 0 ta thay x bởi − x, tương tự neu y < 0, ta thay y bởi 2 =
  • 19. 11 Viết đề tài giá sinh viên – ZALO:0973.287.149-TEAMLUANVAN.COM 1 1 1 1 − y. Q Do v y, khi ta có m®t nghi m nguyên không tam thường (x, y) cho phương trình Pell, ta sě có vô so nghi m, thu được bang cách xét đieu ki n của x + y √ D. 1.2.1.3. Tat cả các nghi m nguyên M®t trình tự tự nhiên cho các nghi m nguyên dương của phương trình Pell có the xác định bang nhieu cách: sap xep chúng bang cách tăng giá trị của x ho c tăng giá trị của y ho c tăng giá trị x + y √ D - ta de dàng kiem tra rang đó là các cách như nhau. M®t nghi m nguyên dương toi thieu (x1, y1) được goi là nghi m cơ bản của phương trình Pell (1.6). Trong cách tương tự ta có m®t nghi m cơ bản của phương trình Pell (1.1). M nh đe 1.2. Giả sả (x1, y1) là nghi m cơ bản cho phương trình Pell (1.6). Tà đó tat cả các nghi m nguyên dương cho ket quả là dãy (xn, yn)n≥1, khi đó xn và yn được cho bởi xn + yn √ D = (x1 + y1 √ D)n , (n ∈ Z, n ≥ 1). Trong các đieu ki n khác, xn và yn được xác định bởi công thác phép truy toán: Hơn nǎa, xn+1 = xnx1 + Dyny1 và yn+1 = x1yn + xny1, (n ≥ 1). +) Neu x2 − Dy2 = 1, thì (x1, y1) là nghi m cơ bản của phương trình Pell loại (I) (1.1), và không là nghi m nguyên của phương trình Pell loại (II) (1.3). +) Neu x2 − Dy2 = − 1, thì (x1, y1) là nghi m cơ bản của phương trình Pell loại(II) (1.3), và nghi m nguyên của phương trình Pell loại (I)(1.1) là (x2, y2). T p hợp các nghi m nguyên dương của phương trình Pell loại(I) (1.1) là {(xn, yn); n ≥ 2 chȁn }, trong đó t p hợp các nghi m nguyên dương
  • 20. 12 Viết đề tài giá sinh viên – ZALO:0973.287.149-TEAMLUANVAN.COM của phương trình Pell (1.3) là {(xn, yn); n ≥ 1 lẻ }. T p hợp tat cả các nghi m (x, y) ∈ Z × Z của phương trình Pell (1.6) là t p hợp (± xn, yn)n∈ Z khi xn và yn cho bởi công thác xn + yn √ D = (x1 + y1 √ D)n , (n ∈ Z). Nghi m tam thường (1, 0) là (x0, y0), nghi m (− 1, 0) là m®t phan tả xoan b c hai trong nhóm các đơn vị của vành Z[D]. Chúng minh. Cho (x, y) là m®t nghi m nguyên dương của phương trình Pell x2 − Dy2 = ± 1. Kí hi u n ≥ 0 là so nguyên lớn nhat sao cho (x1 + y1 √ D)n ≤ x + y √ D. Vì v y, x + y √ D < (x1 + y1 √ D)n+1 . Xác định (u, v) ∈ Z × Z bởi u + v √ D = (x + y √ D)(x1 − y1 √ D)n . Tà u2 − Dv2 = ± 1 và 1 ≤ u + v √ D < x1 + y1 √ D, ta suy ra u = 1 và v = 0. Vì v y x = xn, y = yn. Q 1.2.1.4. Trên nhóm các đơn vị của Z[ √ D] Cho D là so nguyên dương không chính phương. Vành Z[ √ D] là vành con của R sinh bởi √ D. Ánh xạ σ : z = x + y √ D − → x − y √ D là tự đȁng cau Galois của vành này. Quy tac N : Z[ √ D] − → Z xác định bởi N(z) = zσ(z). Do đó N(x + y √ D) = x2 − Dy2 . Hạn che của N với nhóm của đơn vị Z[ √ D]× của vành Z[ √ D] là m®t đong cau tà nhóm nhân Z[ √ D]× đoi với nhóm của đơn vị Z× của Z. Tà khi Z× = {± 1} ta thay rang: Z[ √ D]× = {z ∈ Z[ √ D]; N(z) = ± 1}.
  • 21. 13 Viết đề tài giá sinh viên – ZALO:0973.287.149-TEAMLUANVAN.COM ± Do v y Z[ √ D]× không khác t p của x + y √ D khi (x, y) chạy qua t p cácnghi m so nguyên của phương trình Pell (1.6). M nh đe 1.1 nghĩa là Z[ √ D]× không giảm tại m®t nhóm con 1, trong khi m nh đe 1.2 đưa ra rat nhieu thông tin chính xác rang nhóm Z[ √ D]× là m®t (nhân lên) nhóm Abelian b c 1: đó ton tại đơn vị cơ bản u ∈ Z[ √ D]× sao cho Z[ √ D]× = {± un ; n ∈ Z}. Đơn vị cơ bản u > 1 là x1 + y1 √ D, trong đó (x1, y1) là nghi m cơ bản của phương trình Pell (1.6). Phương trình Pell (1.6) có nghi m nguyên neu và chỉ neu đơn vị cơ bản có tiêu chuȁn -1. B c Z[ √ D]× toi đa 1 là hình ảnh ánh xạ. Z[ √ D]× − → R2 z −→ (log |z|, log |z′ |) là riêng bi t trong R2 và cháa trong giới hạn t1 + t2 = 0 của R2 . Cách cháng minh này là không thực sự khác bi t so với cách cháng minh ta đưa ra ở m nh đe 1.2. Cháng minh rang nhóm con rời rạc của R có b c nhỏ hơn ho c bang 1 dựa vào sự phân chia của Euclid. Chú ý 1.1. Cho d là so nguyên không hǎu tỉ nhưng không là bình phương của so nguyên. Thì d không là bình phương của so hǎu tỉ và trường k = Q( √ d) là m®t phan mở r®ng b c hai của Q (nghĩa là m®t không gian vectơ Q hai chieu). M®t yeu to α ∈ k là m®t so nguyên đại so neu và chỉ neu nó thỏa mãn các đieu ki n tương đương sau: (i) α là nghi m của m®t đa thác loi với các h so nguyên. (ii) Đa thác loi không thu gon của α trên Q có h so nguyên. (iii) Đa thác không thu gon của α trên Z là loi. (iv) Vành Z[ √ α] là modul Z được tạo ra m®t cách hǎu hạn. (v) Vành Z[ √ α] được cháa trong m®t vành con của k là phan tả được tạo ra Z- modul.
  • 22. 14 Viết đề tài giá sinh viên – ZALO:0973.287.149-TEAMLUANVAN.COM . . . . √ √ 1 4 . T p Zk của các so nguyên đại so của k là vành Zk = Z + Z √ d 1 + √ d neu d ≡ 2 ho c 3 (mod 4) Z + Z 2 neu d ≡ 1 (mod 4) Do v y Zk = Z + Zα, trong đó α là m®t trong hai nghi m bat kì của X2 − d neu d ≡ 2 ho c 3 (mod 4) và m®t trong hai nghi m của đa thác X2 − X − (d − 1)/4 = (2X − 1)2 − d, neu d ≡ 1 (mod 4). Bi t thác Dk của k là bi t thác của vành so nguyên k. 2 0 det . = 4d neu d ≡ 2 ho c 3 (mod 4) Dk = 0 2d 2 1 det = d neu d 1 (mod 4) . 1 (1 + d)/2 . Vì v y bi t thác luôn phù hợp với 0 ho c 1 modul 4 và trường b c hai là k = Q[ √ Dk]. Nhóm các đơn vị của k theo định nghĩa là nhóm các đơn vị Z× K của vành Zk. Đoi với d < 0 de dàng kiem tra xem nhóm các đơn vị trong k là nhóm hǎu hạn của nghi m hợp nhat trong k. •{1, i, − 1, − i} neu k có bi t thác -4, nghĩa là k = Q(i). •{1, Q, Q2 , − 1, − Q, − Q2 } neu k có bi t thác -3, trong đó Q là nghi m của X2 + X + 1. Trường b c hai với bi t thác -3 là k = Q( √ Q) = Q( − 3) và Q là nghi m nguyên thủy của phan tả đơn vị. •{± 1} theo cách khác. Giả sả d > 0. Nghi m của phan tả đơn vị trong k chỉ là ±1 và nhóm Z× k của các đơn vị Zk là m®t Z-mođul của b c 1. Do đó, nó là đȁng cau với {± 1} × Z. Đoi với d ≡ 2 và d ≡ 3 (mod 4), các đơn vị Z× k của k là các phan tả x + y Dk ∈ k sao cho (x, y) ∈ Z × Z là m®t nghi m của phương trình Pell ≡
  • 23. 15 Viết đề tài giá sinh viên – ZALO:0973.287.149-TEAMLUANVAN.COM √ − y x2 − Dky2 = ±1. Đoi với d ≡ 1 (mod 4), nhóm các đơn vị Z× k của k là t p hợp các phan tả x + y Dk ∈ k sao cho (x, y) ∈ Z × Z là m®t nghi m của phương trình Pell x2 − Dky2 = ± 4. 1.2.1.5. Ket noi v i xap xỉ hfiu t Bo đe 1.1. Cho D là so nguyên dương không chính phương. Giả sả x và y là các so nguyên dương, các đieu ki n sau đây là tương đương: (i) x2 − Dy2 = 1. x (ii) 0 < y 1 D < 2y2 √ D . (iii) 0 < x √ D < y Chúng minh. 1 1 y2 √ D + 1 . 1 2 2 Ta có 2y2 √ D < y2 √ D + 1 . Do đó (ii) kéo theo (iii). (i) kéo theo x > Dy , do đó x > y √ D và vì the x √ 1 1 √ (iii) kéo theo 0 < y − D = y(x + y √ 1 D) < 2y2 √ D . √ 2 x < y và D + y √ D < y D + , y Do v y y(x + y √ D) < 2y2 √ D + 2. 0 < x2 − Dy2 = y( x − √ D)(x + y √ D) < 2. Tà đó x2 − Dy2 là m®t so nguyên, nó bang với 1. Q Bo đe 1.2. Cho D là so nguyên dương không chính phương. Cho x và y là các so nguyên dương. Các đieu ki n sau là tương đương: (i) x2 − Dy2 = − 1. √ −
  • 24. 16 Viết đề tài giá sinh viên – ZALO:0973.287.149-TEAMLUANVAN.COM — − − y y y x √ x 1 (ii) 0 < D − y < 2y2 √ D − 1 . √ x 1 (iii) 0 < D − y < y2 √ D . Chúng minh. 1 1 Ta có 2y2 √ D − 1 < y2 √ D . Do v y (ii) kéo theo (iii). Đieu ki n (i) kéo theo y √ D > x. Ta sả dụng ước tính tam thường 2 √ D > 1 + 1 y2 và viet x2 = Dy2 1 > Dy2 2 √ D + 1 y2 = (y √ D 1 )2 . y Do v y xy > y2 √ D − 1. Tà (i) suy ra 1 = Dy2 − x2 = (y √ D − x)(y √ D + x) > √ D − x (y2 √ D + xy) > √ D − x (2y2 √ D − 1). (iii) kéo theo x < y √ D và y(y √ D + x) < 2y2 √ D. Do đó 0 < Dy2 − x2 = y( √ D − x )(y √ D + x) < 2. Tà đó Dy2 − x2 là so nguyên, nó là 1. Q H quả 1.1: Cho D là so nguyên dương không chính phương. Cho x và y là các so nguyên dương. Các đieu ki n sau là tương đương: (i) x2 − Dy2 = ± 1. (ii) . √ D − y . < 1 2y2 √ D − 1 . (iii) .√ D − x. < √ 1 . . y . y2 D + 1
  • 25. 17 Viết đề tài giá sinh viên – ZALO:0973.287.149-TEAMLUANVAN.COM x √ . − Chúng minh. Neu y > 1 ho c D > 3 ta có 2y2 √ D − 1 > y2 √ D + 1, nghĩa là (ii) kéo theo m®t phan nhỏ của (iii) và ta có the áp dụng các bő đe 1.1 và 1.2. Neu D = 2 và y = 1 thì moi đieu ki n (i),(ii) và (iii) thỏa mãn khi và chỉ khi x = 1. Đieu ki n này bat nguon tà 2 − √ 2 > 1 2 √ 2 − 1 > 1 √ 2 + 1 > √ 2 − 1. Neu D = 3 và y = 1 thì moi đieu ki n (i),(ii) và (iii) thỏa mãn khi và chỉ khi x = 2. Đieu ki n này bat nguon tà 3 − √ 3 > √ 3 − 1 > 1 2 √ 3 − 1 > 1 √ 3 + 1 > 2 − √ 3. Q Bo đe 1.3. Cho D là so nguyên dương không chính phương. Cho x và y là các so nguyên dương. Thì .√ 1 Chúng minh. . D − y . > 2y2 √ D + 1 . Neu x/y < √ D thì x ≤ y √ D và tà 1 ≤ Dy2 − x2 = (y √ D + x)(y √ D − x) ≤ 2y √ D(y √ D − x). Suy ra √ x 1 D − y > Ta khȁng định neu x/y > √ D thì 2y2 √ D . x 1 y D > 2y2 √ D + 1 . Th t vây, ước tính đúng neu x − y √ D ≥ 1/y, v y ta giả sả rang x − y √ D < 1/y. Khȁng định của ta sau đó tà 1 ≤ x2 − Dy2 = (x + y √ D)(x − y √ D) ≤ (2y √ D + 1/y)(x − y √ D).
  • 26. 18 Viết đề tài giá sinh viên – ZALO:0973.287.149-TEAMLUANVAN.COM 0 √ Q Đieu này cho thay xap xỉ hǎu t x/y đen D, chỉ yeu hơn giới hạn bởi bat đȁng thác Liouville, sě tạo ra nghi m của phương trình Pell (1.6). Khoảng cách | √ D − x/y| không nhỏ hơn 1/(2y2 √ D + 1), nhưng nó có the nhỏ như 1/(2y2 √ D − 1) và cho rang nó chỉ can nhỏ hơn 1/(y2 √ D + 1). 1.2.2. Phân so liên tnc Trước tiên xét các phân so liên tục tőng quát của bieu mau b1 a0 + b2 . Mà ta bieu dien bởi a1 + a2 + b3 ... a + b1| + b2| + b3| . |a1 |a2 ... Ta thu hep trường hợp đ c bi t khi b1 = b2 = ... = 1. Các phân so liên tục đơn giản có dạng a + 1| |a1 + 1| |a2 + ... = [a0 , a1 , a2, ...]. 1.2.2.1. Phân so liên tnc tong quát Được bat đau với a0, ..., an, ... và b1, ..., bn, ... là bien đ®c l p. Sau đó, ta sě chuyen hóa các so nguyên dương (trà a0 là âm). Xem xét 3 phân so hǎu t : Ta viet chúng như sau: a0, a0 + b1 a1 và a0 b1 + b2 . a1 + 2 A0 , A1 và A2 . B0 B1 B2 a 0
  • 27. 19 Viết đề tài giá sinh viên – ZALO:0973.287.149-TEAMLUANVAN.COM ! ! ! ! ! ! với A0 = a0, A1 = a0a1 + b1, A2 = a0a1a2 + a0b2 + a2b1; B0 = 1, B1 = a1, B2 = a1a2 + b2. Quan sát thay A2 = a2A1 + b2A0, B2 = a2B1 + b2B0. Viet h thác của chúng như sau: A2 = A1 A0 B2 B1 B0 a2 . b2 L p lại quá trình đe thu n ti n làm vi c với ma tr n 2 × 2 và viet A2 A1 B2 B1 = A1 A0 B1 B0 a2 1 . b2 0 Xác định phép quy nạp hai dãy của đa thác với h so hǎu t dương An và Bn cho n ≥ 3 bởi An An− 1 ! = An− 1 An− 2 ! an ! . (1.7) An = anAn− 1 + bnAn− 2, Bn = anBn− 1 + bnBn− 2. Quan h trên giǎ cho n ≥ 2. Nó sě giǎ cho n = 1 neu ta đ t A− 1 = 1 và B− 1 = 0: A1 A0 = B1 B0 ! ! và nó sě giǎ cho n = 0 neu ta đ t b0 = 1, A− 2 = 0 và B− 2 = 1: A0 A− 1 ! = 1 0 ! a0 1 ! B0 B− 1 0 1 b0 0 ! 1 Nghĩa là Bn Bn− 1 Bn− 1 Bn− 2 bn 0 a0 1 a1 1 1 0 b1 0
  • 28. 20 Viết đề tài giá sinh viên – ZALO:0973.287.149-TEAMLUANVAN.COM 0 0 0 0 De thay m®t định nghĩa tương đương với An An− 1 ! = a0 1 ! a1 1 ! ... an− 1 1 ! an 1 ! (1.8) Bn Bn− 1 b0 0 b1 0 bn− 1 0 bn 0 Quan h (1.8) giǎ cho n ≥ − 1, với ket quả rong (cho n = − 1) là ma tr n đong nhat. Do v y An ∈ Z[a0, ..., an, b1, ..., bn] là đa thác trong 2n + 1 bien trong khi Bn ∈ Z[a1, ..., an, b2, ..., bn] là đa thác trong 2n − 1 bien. Bo đe 1.4. Với n ≥ 0, a + b1| + ... + bn| = An . Chúng minh. |a1 |an Bn Bang phương pháp quy nạp. Ta kiem tra ket quả với n = 0, n = 1 và n = 2. Giả sả công thác đúng với n − 1 khi n ≥ 3. Ta viet a + b1| + ... + bn−1| + bn| = a + b1| + ... + bn−1| . 0 |a1 |an−1 |a n |a1 |x với x = an− 1 nghĩa (1.7) + bn . Ta có giả thiet theo phương pháp quy nạp và theo định an a + b1| + ... + bn−1| = An− 1 = an− 1An− 2 + bn− 1An− 3 . |a1 |an−1 Bn− 1 an− 1Bn− 2 + bn− 1Bn− 3 Tà đó An− 2, An− 3, Bn− 2 và Bn− 3 không phụ thu®c vào bien an− 1, ta suy ra a + b1| + ... + bn−1| = xAn− 2 + bn− 1An− 3 . |a1 |an− 1 Ket quả của tả so bởi an là xBn− 2 + bn− 1Bn− 3 T (anan− 1 + bn)An− 2 + anbn− 1An− 3 = = an(an− 1An− 2 + bn− 1An− 3) + bnAn− 2 anAn− 1 + bnAn− 2 = An. ương tự, ket quả của mau so bởi an là (anan− 1 + bn)Bn− 2 + anbn− 1Bn− 3 = an(an− 1Bn− 2 + bn− 1Bn− 3) + bnBn− 2 = anBn− 1 + bnBn− 2 = Bn.
  • 29. 21 Viết đề tài giá sinh viên – ZALO:0973.287.149-TEAMLUANVAN.COM Σ Σ 0 Q Tà (1.8), lay định thác, ta suy ra cho n ≥ − 1, n+1 AnBn − An− 1Bn = (− 1) b0...bn, (1.9) có the viet với n ≥ 1, An An− 1 (− 1)n+1 b0...bn Bn − B = n− 1 Bn− 1Bn . (1.10) Thêm tőng như trên ta được, với n ≥ 0, n k+1 An = A0 n + (− 1) b0...bk . (1.11) k=1 Bk− 1Bk Bây giờ, ta thay a0, a1, ... và b1, b2, ... so nguyên, tat cả đeu lớn hơn ho c bang 1, ngoại trà a0 có the nhỏ hơn ho c bang 0. Ta bieu thị bởi pn giá trị của An, tương tự ta bieu thị bởi qn giá trị Bn cho các giá trị đ c bi t này. Do v y pn và qn là so nguyên với qn > 0 cho n ≥ 0. Ket quả của bő đe 1.4 là: pn = a + b1| + ... + bn| cho n ≥ 0. qn Ta suy ra tà (1.7) |a1 |an pn = anpn− 1 + bnpn− 2, qn = anqn− 1 + bnqn− 2, cho n ≥ 0. và tà (1.9), pnqn− 1 − pn− 1qn = (− 1)n+1 b0...bn, cho n ≥ − 1. Có the viet được, cho n ≥ 1, pn pn− 1 (− 1)n+1 b0...bn qn − q = n− 1 qn− 1qn . (1.12) Thêm tőng sả dụng (1.11)) ta được tőng: n k+1 pn = a0 n + (− 1) b0...bk . (1.13) k=1 qk− 1qk B q
  • 30. 22 Viết đề tài giá sinh viên – ZALO:0973.287.149-TEAMLUANVAN.COM Σ q n qn− 1 qn− 1qn 2n−n0 bnbn−1...bn +1qn qn −1 0 0 Nhac lại cho các so thực a, b, c, d với b, d là các so dương, ta có: a c a b < d ⇒ b < a + c < b + d c . (1.14) d Tà đó an và bn là so dương với n ≥ 0, ta suy ra rang cho n ≥ 2, so hǎu t pn = anpn− 1 + bnpn− 2 qn anqn− 1 + bnqn− 2 nam giǎa pn− 1 qn− 1 và pn− 2 . Vì v y, ta có qn− 2 p2 < p4 q2 q4 < ... < p2n q2n < ... < p2m+1 q2m+1 < p3 q3 < p1 . (1.15) q1 Tà (1.12) suy ra, cho n ≥ 3, thì qn− 1 > qn− 2, do v y qn > (an + bn)qn− 2. Suy lu n trên là hợp l mà không có bat kì hạn che nào, giờ ta giả sả an ≥ bn cho tat cả n đủ lớn, nói n ≥ no. Sau đó cho n ≥ n0, sả dụng qn > 2bnqn− 2, ta được . pn − pn− 1 . = b0...bn bn...b0 < 0 0 0 = bn0...b0 , 2n−n0 qn qn −1 và ve phải tien dan đen 0 khi n tien đen vô cùng. Do đó chuoi (pn/qn)n≥0 có giới hạn, ta bieu dien bởi b1| bn− 1| bn| x = a0 + + ... + + + ... |a1 Tà (1.13), x được đưa ra bởi m®t dãy: ∞ |an−1 k+1 |an x = a0 + (− 1) b0...bk . k=1 qk− 1qk Bây giờ ta cháng minh rang x là so vô t . Xác định với n ≥ 0, xn = an + bn+1| + ... |an+1
  • 31. 23 Viết đề tài giá sinh viên – ZALO:0973.287.149-TEAMLUANVAN.COM 0 sao cho x = x0 và cho moi n ≥ 0, xn = an + bn+1 , x xn+1 n+1 bn+1 = , xn − an và an < xn < an + 1. Do v y, cho n ≥ 0, xn là so hǎu t neu và chỉ neu xn+1là so hǎu t , và do đó neu x hǎu t thì tat cả xn với n ≥ 0 là so hǎu t . Giảsả x là so hǎu t . Xét các so hǎu t xn với n ≥ n0 và chon m®t giá trị của n mà neu mau so v của xn là toi thieu, nói xn = u/v. Tà xn+1 bn+1 = xn − an bn+1v = u − anv , với 0 < u − an v < v, sau đó xn+1 có mau so nhỏ hơn v, mâu thuan. Vì v y x là so vô t . Ngược lại, cho m®t so vô t x và m®t chuoi b1, b2, ... của so nguyên dương, có duy nhat m®t so nguyên dương a0 và duy nhat dãy a1, ..., an, ... của so nguyên dương thỏa mãn an ≥ bn cho moi n ≥ 1, sao cho: x = a0 + b1| + ... + bn−1| + bn| + ... |a1 |an−1 |an Thât v y, nghi m duy nhat được đưa vào theo quy nạp như sau: a0 = [x♩ , x1 = b1/{x}, và m®t lan nǎa a0, ..., an− 1 và x1, ..., xn đã được biet, sau đó an và xn+1 được cho bởi an = [xn♩ , xn+1 = bn+1/{xn}, sao cho n ≥ 1, ta có 0 < xn − an < 1 và x = a0 + b1| + ... + bn−1| + bn| . Đây là đieu cháng minh. |a1 |an−1 |xn M nh đe 1.3. Cho so nguyên a0 và hai chuoi a0, a1, ... và b1, b2, ... của so nguyên dương với an ≥ bn cho moi n đủ lớn, phân so liên tục vô hạn a + b1| + ... + bn−1| + bn| + ... |a1 |an−1 |an
  • 32. 24 Viết đề tài giá sinh viên – ZALO:0973.287.149-TEAMLUANVAN.COM 0 ton tại và là so vô t . Ngược lại, cho m®t so x vô t và m®t chuoi b1, b2, ... của so nguyên dương, có duy nhat so nguyên dương a0 ∈ Z và duy nhat chuoi a1, ..., an, ... của so nguyên dương thỏa mãn an ≥ bn cho moi n ≥ 1, sao cho b1| bn− 1| bn| x = a0 + + ... + + + ... |a1 |an−1 |an Sau đây là ví dụ khác ve chuoi so nguyên: √ 1 = 1 + 2| + 4| + 6| + 8| + ... = 1541494082... e − 1 |3 |5 |7 |9 1 = 1| + 2| + 3| + 4| + ... = 0581976706... e − 1 |1 |2 |3 |4 Chú ý 1.2. M®t bien the của thu t toán các phân so liên tục đơn giản tiep theo. Cho hai dãy (an)n≥0 và (bn)n≥0 của các phan tả trong m®t trường K và m®t phan tả x trong K, m®t định nghĩa của chuoi (có the hǎu hạn)(xn)n≥1 của các phan tả trong K như sau: neu x = a0, chuoi rong. Neu không thì x1 được xác định bởi x = a0 + b1/x1. Bang quy nạp, moi x1, ..., xn được xác định, có hai trường hợp: • Neu xn = an, thu t toán dàng lại. • Cách khác, xn+1 được xác định bởi xn+1 = bn+1 xn − an . Do đó xn = an + bn+1 . xn+1 Neu thu t toán không dàng lại, thì cho bat kì n ≥ 1, có b1| bn− 1| bn| x = a0 + + ... + + |a1 |an−1 |an Trong trường hợp đ c bi t khi a0 = a1 = ... = b1 = b2 = ... = 1, t p của x sao cho thu t toán dàng sau hǎu hạn bước là t p (Fn+1/Fn)n≥1 của thương của các so Fibonacci liên tiep. Trong trường hợp đ c bi t này, giới hạn của a + b1| + ... + bn−1| + bn| |a1 |an−1 |an
  • 33. 25 Viết đề tài giá sinh viên – ZALO:0973.287.149-TEAMLUANVAN.COM ! 1 0 ! 1 0 là tỉ so Golden, với x đ®c l p. 1.2.2.2. Phân so liên tnc đơn giản Ta đã hạn che suy lu n của mục 1.2.2.1 với trường hợp b1 = b2 = ... = bn = ... = 1. Ta giǎ các kí hi u An và Bn mà đa thác trong Z[a0, a1, ..., an] và Z[a1, ..., an] tương áng và khi thay đői đen so nguyên a0, a1, ..., an, ... với an ≥ 1, cho n ≥ 1 ta sả dụng kí hi u pn và qn cho giá trị của An và Bn. Các moi quan h trong (1.7), với n ≥ 0, An An− 1 Bn Bn− 1 = An− 1 An− 2 Bn− 1 Bn− 2 ! an 1 ! . (1.16) Trong khi (1.8) trở thành, cho n ≥ − 1, An An− 1 ! = ao 1 ! a1 1 ! ... an− 1 1 ! an 1 ! . (1.17) Bn Bn− 1 1 0 1 0 1 0 1 0 Tà bő đe 1.4, suy ra với n ≥ 0, [a0, ..., an ] = An . Bn Lay định thác trong (1.17), ta suy ra trường hợp đ c bi t của (1.9): AnBn− 1 − An− 1Bn = (− 1)n+1 . Sự thay đői các moi quan h này với các giá trị tách rời của a0, a1, a2, ..., pn pn− 1 qn qn− 1 = pn− 1 pn− 2 qn− 1 qn− 2 ! an 1 ! với n ≥ 0, (1.18) pn pn− 1 ! = ao 1 ! a1 1 ! ... an− 1 1 ! an 1 ! với n ≥ − 1, qn qn− 1 1 0 1 0 1 0 1 0 [a0, ..., an ] = pn qn với n ≥ 0, (1.19)
  • 34. 26 Viết đề tài giá sinh viên – ZALO:0973.287.149-TEAMLUANVAN.COM − − ! ! và pnqn− 1 − pn− 1qn = (− 1)n+1 với n ≥ − 1. (1.20) Tà (1.20) sau đó với n ≥ 0, phân so pn/qn là thap nhat: gcd(pn, qn) = 1. Qua (1.19), với n ≥ − 1, pn qn ! = an 1 ! an− 1 1 ! ... a1 1 ! a0 1 ! , pn− 1 qn− 1 1 0 1 0 tà đó ta suy ra, với n ≥ 1, 1 0 1 0 [an, ..., a0 pn ] = pn− 1 và [an , ..., a0 qn ] = . qn− 1 Bo đe 1.5. Với n ≥ 0, pnqn− 2 − pn− 2qn = (− 1)n an. Chúng minh. Ta nhân cả hai m t của (1.18) ở bên trái bởi sự nghịch đảo của ma tr n: đó là: pn− 1 pn− 2 , qn− 1 qn− 2 Ta nh n được ( 1)n qn− 2 − pn− 2 . − qn− 1 pn− 1 ( 1)n pnqn− 2 − pn− 2qn pn− 1qn− 2 − pn− 2qn− 1 − pnqn− 1 + pn− 1qn 0 ! = an 1 ! . Q 1.2.2.2.1. Phân so liên tnc đơn giản của so hfiu t Cho u0 và u1 là hai so nguyên với u1 là so dương. Bước đau tiên trong thu t toán Euclid đe tìm gcd của u0 và u1 bao gom trong vi c chia uo bởi u1. u0 = a0u1 + u2, 1 0
  • 35. 27 Viết đề tài giá sinh viên – ZALO:0973.287.149-TEAMLUANVAN.COM với a0 ∈ Z và 0 ≤ u2 < u1. Nghĩa là: u0 = a0 1 + u2 . u1 Với giá trị đe chia so hǎu t x0 = u0/u1 với thương a0 và phan còn lại u2/u1 < 1. Thu t toán này tiep tục với um = amum+1 + um+2. Trong đó am là phan không tách rời của xm = um/um+1 và 0 ≤ um+2 < um+1, cho đen khi ul+2 là 0, trong trường hợp các thu t toán dàng lại với ul = alul + 1. Tà đó gcd của um và um+1 giong như gcd của um+1 và um+2, sau gcd của u0 và u1 là ul+1. Đây là cách chính qui đe khai trien phân so liên tục x0 = [a0, a1, ..., al], trong đó l = 0 trong trường hợp x0 là so nguyên, trong khi al ≥ 2 neu x0 là so hǎu t không là so nguyên. M nh đe 1.4. Bat kì m®t phân so liên tục hǎu hạn m®t phan [a0, a1, ..., an], trong đó a0, a1, ..., an là so hǎu t với ai ≥ 2 cho 1 ≤ i ≤ n và n ≥ 0, the hi n m®t so hǎu t . Ngược lại, bat kì so hǎu t x có hai đại di n như m®t phânso liên tục, đau tiên, đưa bởi thu t toán Euclid, là x = [a0, a1, ..., an], và thá hai là x = [a0, a1, ..., an− 1, an − 1, 1]. Neu x ∈ Z thì n = 0 và hai phân so liên tục đơn giản đại di n của x là [x] và [x − 1, 1], trong khi neu x không là so nguyên thì n ≥ 1 và an ≥ 2. u
  • 36. 28 Viết đề tài giá sinh viên – ZALO:0973.287.149-TEAMLUANVAN.COM 1.2.2.2.2. Phân so liên tnc vô hạn đơn giản của so vô t Cho so nguyên a0 và m®t dãy vô hạn các so nguyên dương a1, a2, ..., phân so liên tục [a0, a1, ..., an, ...], đại di n cho so vô t . Ngược lại, đưa so vô t x, có bieu dien duy nhat của x như m®t phân so liên tục vô hạn đơn giản x = [a0, a1, ..., an, ...]. Các định nghĩa. Các so an là thương riêng, so hǎu t là các h®i tụ và các so pn = [a0 n , a1 , ..., an], xn = [an, an+1, ...], là tỉ so hoàn chỉnh. Tà các định nghĩa này, ta suy ra, với n ≥ 0, x = [a0, a1, ..., an , xn+1 ] = xn+1pn + pn− 1 . (1.21) xn+1qn + qn− 1 Bo đe 1.6. Với n ≥ 0, (− 1)n Chúng minh. Tà (1.21) suy ra qnx − pn = xn+1qn . + qn− 1 qn xn+1pn + pn− 1 pn (− 1)n x − p = xn+1qn + qn− 1 — qn = (xn+1qn + qn− 1 . )qn Q H quả 1.2. Cho n ≥ 0, 1 1 qn+1 + qn < |qnx − pn| < . qn+1 q n
  • 37. 29 Viết đề tài giá sinh viên – ZALO:0973.287.149-TEAMLUANVAN.COM n a q Chúng minh. Tà an+1 là m®t phan không thieu của xn+1, ta có an+1 < xn+1 < an+1 + 1. Tà qn+1 = an+1qn + qn− 1, ta suy ra qn+1 < xn+1qn + qn− 1 < an+1qn + qn− 1 + qn = qn+1 + qn. Q Đ c bi t, tà xn+1 > an+1 và qn− 1 > 0>, m®t suy lu n tà bő đe 1.6: 1 (an+1 + 2) + q2 < |x pn — qn | 1 < 2 . (1.22) n+1 n Do đó bat kì h®i tụ p/q của x thỏa mãn |x − p/q| < 1/q2 . Hơn nǎa neu an+1 lớn, thì xap xỉ pn/qn là de nh n. Do đó, phan lớn thương so tot cho xap xỉ hǎu t bởi cat bỏ dạng khai trien phân so liên tục ngay trước khi đưa ra m®t phan thương nhat định. đây, tác giả đã đưa ra tóm tat lí thuyet ve phân so liên tục, đong thời áng dụng lí thuyet phân so liên tục trong giải phương trình Pell khi ket hợp với xap xỉ hǎu t . 1.3. Bài toán fíng dnng Trong mục này, tác giả trình bày m®t so bài t p sưu t p được qua m®t so kì thi hoc sinh giỏi của các năm kèm theo lời giải chi tiet. N®i dung được tham khảo trong tài li u [2], [3]. Bài toán 1.3. (CANADA). Cho hai dãy so {xn} và {yn} xác định như sau: x0 = 1; x1 = 2; xn+1 = 4xn − xn− 1;y0 = 1; y1 = 2; yn+1 = 4yn − yn− 1. Cháng minh rang với moi so nguyên n ta có: y2 = 3x2 + 1. n n Lời giải:
  • 38. 30 Viết đề tài giá sinh viên – ZALO:0973.287.149-TEAMLUANVAN.COM n n Xét phương trình Pell loại I :X2 − 3Y 2 = 1. Phương trình này có nghi m nhỏ nhat là (2; 1) nên tat cả các nghi m của phương trình là (Xn; Yn) sao cho: X0 = 1; X1 = 2; Xn+1 = 4Xn − Xn− 1; Y0 = 1; Y1 = 2; Yn+1 = 4Yn − Yn− 1. Do đó Xn = yn;Yn = xn hay (xn; yn) là nghi m của phương trình Pell loại I trên. V y y2 = 3x2 + 1. Q Bài toán 1.4.(VMO 1999) Cho hai dãy so {xn} và {yn} xác định như sau: x0 = 1; x1 = 4; xn+2 = 3xn+1 − xn; y0 = 1; y1 = 2; yn+2 = 3yn+1 − yn. Giả sả a, b là các so nguyên dương thỏa mãn a2 − 5b2 + 4 = 0. Cháng minh rang ton tại so tự nhiên k đe xk = a, yk = b. Lời giải: Xét phương trình Pell x2 − 5y2 = − 4. (i) Như ta đã biet , ba dãy so sau vét het các nghi m của phương trình (i): x0,1 = 1; y0,1 = 1; xn+1,1 = 9xn,1 + 20yn,1; yn+1,n = 4n,1 + 9n,1, x0,2 = 1; y0,2 = 1; xn+1,2 = 9xn,2 + 20yn,2; yn+2,n = 4n,2 + 9n,2, x0,3 = 1; y0,3 = 1; xn+1,3 = 9xn,3 + 20yn,3; yn+3,n = 4n,3 + 9n,3. Ta cháng minh (xn, yn) cũng vét het tat cả các nghi m nguyên dương của (i). Với moi so tự nhiên n thì n = 3m +r với r = 0; 1; 2. Ta cháng minh (xn, yn) = (xm,r+1; ym,r+1). Ta có: (x0, y0) = (x0,1; y0,1 = (1; 1); (x1, y1) = (x0,2; y0,2 = (4; 2); (x2, y2) = (x0,3; y0,3 = (11; 5).
  • 39. 31 Viết đề tài giá sinh viên – ZALO:0973.287.149-TEAMLUANVAN.COM − 2 √ x ! Phương trình đ c trưng của dãy {xn} và {yn} là X2 3X + 1 = 0, có hai nghi m X = 3 ± √ 5 . 2 Nên: xn = x3m+r = α 3 − √ 5 !3m+r 3 + √ 5 3m+r + β 3 − √ 5 !r (9 − 4 √ 5)m + β 3 + √ 5 !r (9 + 4 √ 5)m Đ t um = x3m+r Ta có dãy {um} có phương trình đ c trưng có 2 nghi m là 9 ± 4 5 nên: Suy ra: Tương tự: um+1 = 18um − um− 1. x3(m+1)+r = 18x3m+r − x3(m− 1)+r. (*) y3(m+1)+r = 18y3m+r − y3(m− 1)+r. (**) Vi c còn lại ta cháng minh {xm,i; ym,i} cũng thỏa mãn (*) và (**) với i = 1; 2; 3. Ta có: xm+1,i = 9xm,i + 20ym,i ym+1,i = 4xm,i + 4ym,i ⇒ y = xm+1,i − 9xm, i (a) y m,i m− 1,i 20 = m,i − 9xm− 1,i 20 (b) 2 2 2 ym,i = 4m− 1,i + 9ym− 1,i (c) = α
  • 40. 32 Viết đề tài giá sinh viên – ZALO:0973.287.149-TEAMLUANVAN.COM 0 } 0 ≤ 0 0 The (a), (b) vào (c) ta suy ra: xm+1,i = 18xm,i − xm− 1,i và ym+1,i = 18ym,i − ym− 1,i. Như v y {xn} ;{yn} vét het tat cả các nghi m của phương trình (i). Do đó luôn ton tại k đe {xk = a} ;{yk = b}. Q Bài toán 1.5. (VMO 2012) Xét các so tự nhiên lẻ a, b mà a là ước so của b2 + 2 và b là ước so của a2 + 2. Cháng minh rang a là các so hạng của dãy so tự nhiên (vn) xác định bởi v1 = v2 = 1 và vn = 4vn− 1 − vn − 2, n ≥ 2. Lời giải: Giả sả (a, b) là c p so tự nhiên mà a là ước so của b2 + 2 và b là ước so của a2 + 2. Trước het ta cháng minh (a, b) = 1. Th t v y, đ t d = (a, b) thì d là ước của a và a là ước của b2 + 2 nên d là ước của b2 + 2 suy ra d là ước của 2. Mà a, b lẻ nên d lẻ suy ra d = 1. Xét so N = a2 + b2 + 2 thì do a2 + 2 chia het cho b nên N chia het cho b. V y ton tại so nguyên dương k sao cho a2 + b2 + 2 = kab. (i) Tiep theo ta cháng minh với k = 4. Th t v y, đ t A = a + b|(a, b) ∈ N∗2 , a2 + b2 + 2 = kab . Theo giả sả ở trên thì A /= ∅ . Do tính sap thá tự tot của N, A có phan tả nhỏ nhat. Giả sả a0, b 0 là c p so thỏa mãn đieu ki n (i) với a0 + b0 nhỏ nhat. Không mat tính tőng quát, có the giả sả a0 ≥ b0. Xét phương trình a2 + kb0a + b2 = 0 có nghi m a0. Theo định lí Vi-ét thì phương trình trên có m®t nghi m nǎa là a1kb0 − a0 = b2 + 2 . Theo công thác nghi m thì rõ ràng a1 a0 nguyên dương. Như v y (a1, b0) cũng là nghi m của phương trình (i). Do tính nhỏ nhat của a0 + b0,ta có a0 + b0 ≤ a1 + b0. Tác là a0 ≤ kb0 − a0 suy ra a0 k b0 2 Ta có : a2 + b2 + 2 = ka0b0 suy ra a0 + b0 + 2 = k. (ii) b0 a0 a0b0
  • 41. 33 Viết đề tài giá sinh viên – ZALO:0973.287.149-TEAMLUANVAN.COM 2 1 0 0 Do a0 < k b0 2 và a0 ≥ b0 ≥ 1 nên tà đây ta có k < k + 1 + 2, suy ra k ≤ 6. M t khác áp dụng bat đȁng thác AM-GM, a2 + b2 ≥ 2a0b0. Nên k ≥ 2. Neu k /= 4 thì (a0, b0) /= (1, 1) do đó a0b0 ≥ 2. Lại dùng (ii) đe đánh giá, ta có k < + 1 + 2, suy ra k ≤ 4. V y các giá trị k = 5, 6 bị loại. 2 Neu k = 3 thì do a2 + b2 + 2 = 3a0b0, nên suy ra a2 + b2 + 2 chia het cho 0 0 0 0 3, suy ra m®t trong hai so a0 và b0 chia het cho 3, so còn lại không chia het cho 3. Neu b0 = 1 thì a0 chia het cho 3, khi đó ve trái không chia het cho 9 còn ve phải chia het cho 9, mau thuan. V y b0 > 1. Tà đó suy ra a0b0 ≥ 6. Lại sả dụng (ii) đe đánh giá ,ta có k 2 3 k ≤ 2 + 1 + 6 ⇒ k < 8 . Mà k nguyên suy ra k ≤ 2, mau thuan. Như v y ta cháng minh được neu a, b là các so tự nhiên lẻ thỏa mãn đieu ki n đe bài thì: a2 + b2 + 2 = 4ab. (iii) 0 0 Đ t ȁn phụ z = a − 2b. Phương trình (iii) trở thành: z2 − 3b2 = − 2. (iv) Giải phương trình Pell (iv) với tham so n = − 2, ta cháng minh được hoàn toàn bài toán. Q Nh n xét: Nhǎng bài toán trên là các bài toán được sưu tam tà các đe thi hoc sinh giỏi toán trong nước và ngoài nước, có kèm theo lời giải giúp bạn đoc thay được nhǎng áng dụng của phương trình Pell. Qua đó bạn đoc cũng thay được phương trình Pell nói riêng cũng như phương trình nghi m nguyên nói chung không có quy tac giải tőng quát ho c chỉ có với nhǎng dạng đơn giản. Bài t p đe nghị
  • 42. 34 Viết đề tài giá sinh viên – ZALO:0973.287.149-TEAMLUANVAN.COM 2xy = zu n Bài 1.6. (IMO Shorthist) Xét h phương trình x + y = z + u Tìm giá trị lớn nhat của hang so thực m sao cho m ≤ nguyên dương (x, y, z, u) của h mà x ≥ y. x với moi nghi m y Bài 1.7. Cháng minh rang neu 5x2 + 4 ho c 5x2 − 4 là so chính phương khi và chỉ khi x là so hạng của dãy Fibonacci. Bài 1.8. (Bulgaria 1999). Cháng minh rang phương trình x2 + y2 + z2 + t2 = 1999 có vô so nghi m ngyên dương. Bài 1.9. (IMO Shorthist) Cháng minh rang ton tại hai dãy so nguyên dương tăng (an), (bn) sao cho an(an + 1) là ước của b2 + 1 với moi n. Ket lu n: Trong chương 1, tác giả đã trình bày m®t cách h thong m®t so khái ni m và ket quả của phương trình Pell cơ bản, h thong lí thuyet ve phân so liên tục và m®t so bài toán áng dụng của phương trình Pell là các bài toán trong các kì thi hoc sinh giỏi trong nước và ngoài nước.
  • 43. 35 Viết đề tài giá sinh viên – ZALO:0973.287.149-TEAMLUANVAN.COM Chương 2 XAP XỈ DIOPHANTINE, M R NG PHƯƠNG TRÌNH PELL VÀ ỨNG DỤNG Trong chương này tác giả sě trình bày ve h thong các bő đe, h quả, m nh đe ve xap xỉ Diophantine, phân so liên tục trong giải phương trình Pell và áng dụng của nó. N®i dung được tác giả tham khảo trong tài li u [4], [5]. 2.1. Chu kì của phân so liên tnc Trong mục này tác giả sě trình bày bő đe chính nham ho trợ trong vi c cháng minh các m nh đe, h quả có trong các mục đe làm sáng tỏ n®i dung chính của chương: các van đe ve xap xỉ Diophantine và phân so liên tục, mở r®ng của xap xỉ Diophantine. N®i dung chính được tham khảo trong tài li u [4].
  • 44. 36 Viết đề tài giá sinh viên – ZALO:0973.287.149-TEAMLUANVAN.COM c d 1 0 1 0 1 0 2.1.1. Bo đe chính Bo đe 2.1. Cho ϵ = ± 1 và a, b, c, d là so nguyên thỏa mãn: ad − bc = ϵ, và d ≥ 1 thì có duy nhat m®t dãy hǎu hạn của các so nguyên a0, ..., as với s ≥ 1 và a1, ..., as− 1 dương sao cho: a b ! = a0 1 ! a1 1 ! ... as 1 ! . (2.1) Các so nguyên này được đ c trưng bởi: b d = [a0, a1 , ..., a c s− 1], d = [as, ..., a1 ], (− 1)s+1 = ϵ. (2.2) Ví dn 2.1. Khi d = 1, cho b và c là các so nguyên, bc + 1 b ! = b 1 ! c 1 ! , và bc − 1 b ! = b − 1 1 ! 1 1 ! c − 1 1 ! . c 1 Chúng minh bő đe 2.1. 1 0 1 0 1 0 Với tính duy nhat: Neu a0, .., as thỏa mãn các ket lu n của bő đe 2.1, thì bang cách sả dụng (2.1), ta thay b c d = [a0, a1, ..., as− 1], d = [as, ..., a1]. Tiep theo tính định thác ta có được (− 1)s+1 = ϵ. b c Theo sự co định đȁng thác cuoi ve tính chȁn lẻ của s và moi so hǎu tỉ , có d d m®t phan khai trien phân so liên tục mà chieu dài đưa ra tính chȁn lẻ (theo c 1 1 0 1 0
  • 45. 37 Viết đề tài giá sinh viên – ZALO:0973.287.149-TEAMLUANVAN.COM − m nh đe 1.4). Đieu này cháng tỏ tính duy nhat của sự phân tích thành nhân tả khi nó ton tại. c Với tính ton tại. Ta xét dạng khai trien phân so liên tục đơn giản của c d với chieu dài của tính chȁn lẻ đưa ra bởi công thác (2.2) là b d = [as, .., a1]. Cho a0 là so nguyên sao cho khoảng cách giǎa d và [a0, a1, ..., as− 1] là nhỏ 1 ′ ′ ′ ′ hơn ho c bang . Xác định a , b , c , d 2 bởi a′ b′ ! = a0 1 ! a1 1 ! ... as 1 ! . Ta có d′ > 0, a′ d′ b′ c′ = ϵ, c′ d′ = [as, ..., a c 1] = d . Tà gcd(c, d) = gcd(c′ , d′ ) = 1, c d = c′ , d > 0, d′ > 0 ta suy ra c′ = c, d′ = d. d′ Tà sự bình đȁng giǎa các định thác, ta suy ra a′ = a + kc, b′ = b + kd với m®t so k ∈ Z và tà b′ b d′ − d = k. V y k = 0, (a′ , b′ , c′ , d′ ) = (a, b, c, d). Do v y (2.1) được cháng minh. Q H quả 2.1. Giả sả các giả thiet của bő đe 2.1 là thỏa mãn (a)Neu c > d thì as ≥ 1 và (b)Neu b > d thì a0 ≥ 1 và a c = [a0, a1, ..., as]. a b = [as, ..., a1, a0]. Ví dụ sau cho thay giả thiet của h quả là không thàa 1 b ! = b 1 ! 0 1 ! 0 1 c′ d′ 1 0 1 0 1 0 1 0 1 0
  • 46. 38 Viết đề tài giá sinh viên – ZALO:0973.287.149-TEAMLUANVAN.COM 1 0 1 0 b − 1 b ! = b − 1 1 ! 1 1 ! 0 1 ! 1 1 1 0 1 0 1 0 và c − 1 1 ! = 0 1 ! 1 1 ! c − 1 1 ! . c 1 Chúng minh. 1 0 1 0 1 0 Bat kì so hǎu tỉ u/v > 1 có hai phân so liên tục. M®t trong chúng bat đau với 1 chỉ khi u/v = 1 và phân so liên tục là [0, 1]. Do v y, giả sả c/d bao hàm as > 0. Đieu này cháng tỏ phan (a), phan (b) là chuyen vị (ho c l p lại cháng minh). Q H quả 2.2. Cho a, b, c, d là các so nguyên với ad − bc = ± 1 và c > d > 0. Cho x, y là hai so vô tỉ thỏa mãn y > 1 và ay + b x = . cy + b Cho x = [a0, a1, ...] là dạng khai trien phân so liên tục đơn giản của x. Thì ton tại s ≥ 1 sao cho a = ps, b = ps− 1, c = qs, r = qs− 1, y = xs+1. Chúng minh. Sả dụng bő đe 2.1, ta viet a b ! = a′ 0 1 ! a′ 1 1 ! ... a′ s 1 ! , với a′ , ..., a′ b dương và = [a′ , a′ , ..., a′ ], c = [a′ , ..., a′ ]. Tà c > d và h 1 s− 1 d 0 1 s− 1 d s 1 quả 2.1, ta suy ra a′ s > 0 và a = [a′ , a′ , ..., a′ ] = ps ′ , x = ps ′ y + ps ′ −1 = [a′ , ..., a′ , y]. c 0 1 s qs ′ qs ′ y + qs ′ −1 Tà y > 1 là ai ′ = ai, p′ i = qi ′ với 0 ≤ i ≤ s và y = xs+1. Q c d 1 0 , 0 s
  • 47. 39 Viết đề tài giá sinh viên – ZALO:0973.287.149-TEAMLUANVAN.COM ≥ 0 1 q 1 Nh n xét 2.1. Đưa ra các so nguyên a0, a1, ... với ai > 0 cho i ≥ 1 và viet với n 0, pn qn hai công thác: = [a0, a1 , ..., an ] ta kiem tra bởi phương pháp quy nạp trên n, 1 ao ! 1 0 ! ... 1 an ! = pn− 1 pn ! neu n chȁn 0 1 a1 1 0 1 qn− 1 qn (2.3) ao ! 1 0 ! ... 1 0 ! = pn ! neu n lẻ Xác định hai ma tr n U và L trong GL2(R) của định thác +1 bởi U = 1 1 ! và L = 1 0 ! . 0 1 1 1 Cho p và q trong Z, ta có Up = 1 p ! và Lq = 1 0 ! . Do đó các công thác (2.3) có dạng: Ua0 La1 ...Lan = pn−1 pn qn− 1 qn ! neu n chȁn, và Ua0 La1 ...Lan = pn pn−1 qn qn− 1 ! neu n lẻ. Đieu liên quan với thu t toán Euclid là: U− p a b ! = a − pc b − pd ! và L− q a b ! = a b ! . c d c d c d c − qa d − qb 1 pn− 1 0 1 a1 1 an 1 qn qn− 1
  • 48. 40 Viết đề tài giá sinh viên – ZALO:0973.287.149-TEAMLUANVAN.COM 0 2.1.2. Chu kì phân so liên tnc M®t dãy so vô hạn an n≥0 được cho là chu kỳ sau cùng neu ton tại n0 ≥ 0 và s ≥ 1 sao cho: với moi n ≥ n0. an+s = an, (2.4) T p của s thỏa mãn thu®c tính (2.1.2) là t p b®i so dương của so nguyên s0 và (an0 , an0+1, ..., an0+s0−1) được goi là chu kỳ cơ bản. M®t phân so liên tục với dãy của các thương so thỏa mãn (2.4) sě được viet như sau: [a0, a1, ..., an0−1, an0, ..., an0+s−1]. Ví dn 2.2. Cho D là so nguyên dương không chính phương, t p a0 = [ √ D♩ , ta có bởi định lý 2.1 a + √ D = [2a , a1, ..., as− 1 ] và 1 √ D − a0 = [a1, ..., as− 1 , 2a0]. Bo đe 2.2. (Euler, 1737 ). Neu phân so vô hạn liên tục x = [a0, a1, ..., an, ...] là chu kỳ sau cùng, thì x là so vô t b c hai. Chúng minh. Tà phân so liên tục của x là vô hạn, x là so vô t . Giả sả đau tiên phân so liên tục là chu kì, trong (2.4) với n0 = 0, x = [a0, ..., as− 1]. Ta có the viet Do v y Suy ra x = [a0, ..., as− 1, x]. x = ps− 1x + ps− 2 . qs− 1x + qs− 2 qs− 1x2 + (qs− 2 − ps− 1)x − ps− 2, 0
  • 49. 41 Viết đề tài giá sinh viên – ZALO:0973.287.149-TEAMLUANVAN.COM n− 1 n− 1 n− 1 là m®t đa thác b c hai khác không với h so nguyên có x như là nghi m a. Vì x là so vô t , đa thác này không rút gon và x là b c hai. Trong trường hợp tőng quát tại (2.4) với n0 > 0,ta viet: x = [a0, a1, ..., an0−1, an0, ..., an0+s−1] = [a0, a1, ..., an0−1, y], với y = [an0, ..., an0+s−1] là chu kì phân so liên tục, vì v y x là b c hai. Nhưng x = pn0− 1 + pn0− 2 , qn0− 1 + qn0− 2 do v y x ∈ Q(y) là so vô t b c hai. Q Bo đe 2.3. (Lagrange, 1770) Neu x là so vô t b c hai thì phân so liên tục x = [a0, a1, ..., an, ...] là chu kì sau cùng. Chúng minh. Cho n ≥ 0, xác định dn = qnx − pn. Theo h quả 1.2, ta có | dn |< 1 . qn+1 Ax2 + Bx + C với A > 0 là đa thác b c hai không rút gon có x là nghi m. Với n ≥ 2, ta suy ra tà (1.21) rang h®i tụ xn là nghi m của đa thác b c hai Anx2 + Bnx + Cn với An = Ap2 + Bpn− 1qn− 1 + Cq2 , Bn = 2Apn− 1pn− 2 + B(pn− 1qn− 2 + pn− 2qn− 1) + 2Cqn− 1qn− 2,Cn = An− 1. Cho Ax2 + Bx + C = 0, ta suy ra: An = (2Ax + B)dn− 1qn− 1 + Ad2 , Bn = (2Ax + B)(dn− 1qn− 2 + dn− 2qn− 1) + 2Adn− 1dn− 2, có công thác tương tự bieu dien A, B, C như tő hợp tuyen tính đong nhat của An, Bn, Cn và tà (A, B, C) (0, 0, 0) ta có (An, Bn, Cn) = / (0, 0, 0). Tà khi xn là so vô t , suy ra An 0. Tà bat đȁng thác qn− 1 | dn− 2 |< 1, qn− 2 | dn− 1 |< 1, qn− 1 < qn, | dn− 1dn− 2 |< 1,
  • 50. 42 Viết đề tài giá sinh viên – ZALO:0973.287.149-TEAMLUANVAN.COM suy ra max{|An|, |Bn/2, |Cn|} < A + |2Ax + B|. Đieu này cho thay |An|, |Bn|, |Cn| là giới hạn đ®c l p của n. Bởi v y ton tại n0 ≥ 0 và s > 0 sao cho xn0 = xn0+s. Tà đó ta suy ra phân so liên tục của x là chu kì sau cùng. Q Bo đe 2.4. M®t phân so liên tục x = [a0, a1, ..., an...], là chu kì tuan hoàn khi và chỉ khi x là so vô t b c hai giảm. Trong trường hợp này, neu x = [a0, a1, ..., as− 1] và neu x′ là liên hợp Galois của x thì −1/x′ = [as− 1, ..., a1, a0]. Chúng minh. Giả sả phân so liên tục của x là chu kì tuan hoàn x = [a0, a1, ..., as− 1]. Tà as = a0 ta suy ra a0 > 0, do v y x > 1. Tà x = [a0, ..., as− 1] và tính duy nhat của dạng khai trien phân so liên tục ta suy ra x = ps− 1x + ps− 2 qs− 1x + qs− 2 và x = xs, do đó x là nghi m của đa thác b c hai Ps(x) = qs− 1x2 + (qs− 2 − ps− 1)x − ps− 2. Đa thác Ps này có nghi m dương, cụ the x > 1 và nghi m âm x′ với ket quả xx′ = −ps− 2/qs− 1. Ta hoán vị moi quan h ps− 1 ps− 2 ! = a0 1 ! a1 1 ! ... as− 1 1 ! qs− 1 qs− 2 và có được 1 0 1 0 1 0 ps− 1 qs− 1 ! = as− 1 1 ! ... a1 1 ! a0 1 ! . ps− 2 qs− 2 1 0 1 0 1 0
  • 51. 43 Viết đề tài giá sinh viên – ZALO:0973.287.149-TEAMLUANVAN.COM Định nghĩa như sau đe y > 1, y = [as− 1, ..., a1, a0], y = [as− 1, ..., a1 , a0 , y] = ps− 1y + qs− 1 , ps− 2y + qs− 2 và y là nghi m dương của đa thác Qs(x) = ps− 2x2 + (qs− 2 − ps− 1)x − qs− 1. Đa thác Ps, Qs có quan h bởi Qs(x) = −x2 Ps(−1/x). Do v y y = −1/x′ . Ngược lại, giả sả x > 1 và −1 < x′ < 0. Cho (xn)n≥1 là dãy các thương so hoàn chỉnh của x. Cho n ≥ 1, xác định x′ n như liên hợp Galois của xn. Bang quy nạp suy ra x′ n = an + 1/x′ xn ′ +1 và −1 < x′ n < 0( do v y xn giảm) và an là phan nguyên của −1/x′ n+1. Neu khai trien phân so liên tục của x là chu kì không tuan hoàn, ta có x = [a0, ..., ah− 1, ah, ..., ah+s− 1], với ah− 1 /= ah+s− 1. Bang tính chu kì ta có xh = [ah, ..., ah+s− 1, xh], do v y xh = xh+s, x′ h = xh ′ +s. Tà x′ h = x′ h+s lay phan nguyên, ta suy ra ah− 1 = ah+s− 1, mâu thuan. Q H quả 2.3. Neu r > 1 là so hǎu t không chính phương thì khai trien phân so liên tục của √ r có dạng √ r = [a0, a1, ..., as− 1, 2a0], với a1, ..., as− 1 có vai trò như nhau và a0 = [ √ r♩ . Ngược lại, neu khai trien phân so liên tục của so vô t t > 1 có dạng t = [a0, a1, ..., as− 1, 2a0], với a1, ..., as− 1 có vai trò như nhau thì t2 là so hǎu t . Chúng minh.
  • 52. 44 Viết đề tài giá sinh viên – ZALO:0973.287.149-TEAMLUANVAN.COM − Neu t2 = r là so hǎu t lớn hơn 1 thì cho a0 = [ √ t♩ và so x = t + a0 là giảm. Tà đó t + t′ = 0, ta có Do v y 1 − x′ = 1 . x − 2a0 1 x = [2a0, a1, ..., as− 1], − x′ = [as− 1, ..., a1, 2a0], với a1, ..., as− 1 có vai trò như nhau. Ngược lại, neu t = [a0, a1, ..., as− 1, 2a0] với a1, ..., as− 1 có vai trò như nhau thì x = t + a0 là chu kì giảm và liên hợp Galois của x′ thỏa mãn 1 − x′ = [a1, ..., as− 1, 2a0] = 1 , x − 2a0 nghĩa là t + t′ = 0, do v y t2 ∈ Q. Q Bo đe 2.5. (Serret, 1878) Cho x, y là hai so vô t với phân so liên tục x = [a0, a1, ..., an...] và y = [b0, b1, ..., bm, ...] tương áng thì hai thu®c tính sau tương đương: (i) Ton tại ma tr n a b c d ! , với h so nguyên và định thác ± 1 sao cho ax + b y = . cx + d (ii) Ton tại n0 ≥ 0 và m0 ≥ 0 sao cho an0+k = bm0+k với moi k ≥ 0. Đieu ki n (i) có nghĩa là x và y là modul tương đương sự tác đ®ng của GL2(Z) bởi sự đong nghĩa. Đieu ki n (ii) có nghĩa là ton tại so nguyên n0, m0 và so thực t > 1 sao cho x = [a0, a1, ..., an0−1, t] và y = [b0, b1, ..., bm0−1, t]. Ví dụ như neu x = [a0, a1, x2] thì x = [− a0 − 1, 1, a1 − 1, x2], a1 ≥ 2 [− a0 − 1, 1 + x2], a1 = 1 (2.5)
  • 53. 45 Viết đề tài giá sinh viên – ZALO:0973.287.149-TEAMLUANVAN.COM ! ! ! Chúng minh. Tà công thác (1.21) neu xn là m®t thương hoàn toàn của x, thì x và xn là modul tương đương GL2(Z), đieu ki n (ii) nghĩa là m®t phan thương của x và m®t phan thương của y là bình đȁng. Bởi tính bac cau của GL2(Z) tương đương, (ii) kéo theo (i). Ngược lại, giả sả (i): ax + b y = . cx + d Cho n là m®t so đủ lớn, tà với a b pn pn− 1 c d qn qn− 1 = un un− 1 , vn vn− 1 Ta suy ra un = apn + bqn, un− 1 = apn− 1 + bqn− 1,vn = cpn + dqn, vn− 1 = cpn− 1 + dqn− 1. y = unxn+1 + un− 1 . vnxn+1 + vn− 1 Ta có vn = (cx + d)qn + cδn với δn = pn − qnx. Ta có qn → ∞, q ≥ qn− 1 + 1và δn → 0 như n → ∞. Do v y, cho n đủ lớn, ta có vn > vn− 1 > 0. Tà phan 1 của h quả 2.1, ta suy ra: un un−1 ! = a0 1 ! a1 1 ! ... as 1 ! , vn vn− 1 1 0 1 0 1 0 với a0, ..., as ∈ Z và a1, ..., as dương. Do đó: y = [a0, a1, ..., as, xn+1]. Q Cháng minh khác được đưa ra bởi Bombieri. Ông sả dụng thực te là GL2(Z) được tạo bởi hai ma tr n: 1 1 ! và 0 1 ! . 0 1 1 0
  • 54. 46 Viết đề tài giá sinh viên – ZALO:0973.287.149-TEAMLUANVAN.COM Các phép bien đői tuyen tính phân đoạn có liên quan là K và J xác định bởi: 1 K(x) = x + 1 và J(x) = . x Ta có J2 = 1 và K([a0, t]) = [a0 + 1, t], K− 1 ([a0, t]) = [a0 − 1, t]. Có J([a0, t]) = [0, a0, t] neu a0 > 0 và J([0, t]) = [t]. Theo (2.5), phân so liên tục của x và − x chỉ khác nhau bởi dạng đau tiên. Cháng minh hoàn thành. Ket lu n: Tác giả đã trình bày bő đe chính và chu kì phân so liên tục sě được dùng làm ket quả đe cháng minh các kien thác của phan sau. 2.2. Xap xỉ Diophantine và phân so liên tnc đơn giản Trong phan này tác giả sě trình bày lí thuyet phân so của √ D, xap xỉ Diophantine và phân so liên tục đơn giải trong giải phương trình Pell. N®i dung chính được tham khảo trong tài li u [4]. 2.2.1. Phân so liên tnc đơn giản của √ D M®t chuoi vô hạn (an)n≥1 là chu kì neu ton tại m®t so nguyên dương s sao cho an+s = an(n ≥ 1). (2.6) Trong trường hợp này, các chuoi hǎu hạn (a1, a2, ..., as) được goi là chu kì của chuoi ban đau. Ta viet (a1, a2, ...) = (a1, ..., as). Neu s0 là so nguyên dương nhỏ nhat thỏa mãn (2.6) thì t p của s thỏa mãn (2.6) là t p của các b®i dương của s0. Trong trường hợp này (a1, ..., as0 ) được goi là chu kì cơ bản của chuoi ban đau.
  • 55. 47 Viết đề tài giá sinh viên – ZALO:0973.287.149-TEAMLUANVAN.COM n n 1 1 0 1 1 Định lí 2.1. Cho D là so nguyên dương không chính phương. Phân so liên tục đơn giản của √ D là [a0, a1, ...] với a0 = [ √ D♩ . (a) Chuoi (a1, a2, ...) là chu kì. (b) Cho (x, y) là nghi m nguyên dương của phương trình Pell x2 − Dy2 = ± 1. Ton tại s ≥ 1 sao cho x/y = [a0, ..., as− 1] và (a1, a2, ..., as− 1, 2a0) là chu kì của chuoi (a1, a2, ...). Hơn nǎa as− i = ai với 1 ≤ i ≤ s − 1. Trong đó a1, ..., as− 1 là đeu như nhau. (c) Cho (a1, a2, ..., as− 1, 2a0) là chu kì của chuoi (a1, a2, ...). T p x/y = [a0, ..., as− 1] thì x2 − Dy2 = (− 1)s . (d) Cho s0 là đ® dài của chu kì cơ bản với i ≥ 0 không là b®i so của s0, ta có ai ≤ a0. Neu (a1, a2, ..., as− 1, 2a0) là chu kì của chuoi (a1, a2, ...) thì √ D = [a , a1, ..., as− 1 ,2a0] = [a0 , a1, ..., as− 1 , a0 + √ D]. Xét chu kì cơ bản (a1, a2, ..., as0−1, as0 ) của chuoi (a1, a2, ...). Bởi phan (b) của định lí 2.1 ta có as = 2a0 và bởi phan (d), s0 là chỉ so nhỏ nhat i sao cho ai > a0. Tà (b) và (c) trong định lí 2.1 nghi m cơ bản (x1, y1) của phương trình Pell x2 − Dy2 = ±1 cho bởi x1/y1 = [a0, a1, ..., as − 1] và x2 − Dy2 = (−1)s0 . Do đó neu s0 là chȁn thì không có nghi m của phương trình Pell x2 − Dy2 = − 1. Neu s0 lẻ thì (x1, y1) là nghi m cơ bản của phương trình Pell x2 − Dy2 = − 1, trong khi nghi m cơ bản (x2, y2) của phương trình Pell x2 − Dy2 = 1 được đưa ra bởi công thác x2/y2 = [a0, a1, ..., a2s− 1]. Tà định lí 2.1 nói rang thá(ns0 − 1) h®i tụ thỏa mãn : xn/yn = [an, ..., ans0−1], x + y √ D = (x + y √ D)n . (2.7) Chúng minh. Bat đau với nghi m dương (x, y) của phương trình Pell: x2 − Dy2 = ± 1, 0
  • 56. 48 Viết đề tài giá sinh viên – ZALO:0973.287.149-TEAMLUANVAN.COM x y 1 0 1 0 1 0 1 0 1 0 1 0 x y δ 1 x + δy y mà theo ton tại của m nh đe 1.1. Tà đó, Dy ≥ x và x > y ta có the sả dụng bő đe 2.1 và h quả 2.1 với a = Dy, b = c = x, d = y, và viet Dy x ! = a′ 0 1 ! a′ 1 1 ! ... as ′ 1 ! , (2.8) với so nguyên dương a′ 0, ..., a′ s và viet a′ 0 = [ √ D♩ . Sự khai trien phân so liên tục của Dy/x là [a0 ′ , ..., a′ s] và sự khai trien phân so liên tục của x/y là [a0 ′ , ..., a′ s−1]. Tà đó ma tr n phía bên trái của (2.8) là đoi xáng, a′ 0, ..., a′ s là đeu như nhau. Đ c bi t a′ s = a′ 0. Xét chu kì phân so liên tục δ = [a0 ′ , a1 ′ , ..., a′ s−1, 2a′ 0]. So δ này thỏa mãn δ = [a′ 0, a′ 1, ..., a′ s−1, a′ 0 + δ]. Sả dụng phép nghịch đảo ma tr n 1 0 δ Do v y ket quả của ma tr n liên quan đen phân so liên tục của δ, a′ 0 x ! a′ 1 x ! ... a′ s−1 x ! a′ 0 + δ 1 ! , Dy x ! 1 0 ! = Dy + δx x ! . 1 0 là a′ 0 1 ! đó là 0 1 1 −a′ 0 ! . Ta viet a0 ′ + δ 1 1 0 ! = a′ 0 1 1 0 ! 1 0 ! . 1
  • 57. 49 Viết đề tài giá sinh viên – ZALO:0973.287.149-TEAMLUANVAN.COM ± 0 1 s− 1 0 0 n 0 m m m 2 x + δy i i n n 2 Tà đó δ = Dy + δx , do v y δ2 = D. Như m®t h quả, a′ = a với 0 ≤ i ≤ s− 1, trong đó a′ s = a0,as = 2a0. Đieu này cháng tỏ neu (x, y) là nghi m không tam thường của phương trình Pell x2 + Dy2 = 1 thì sự khai trien phân so liên tục của √ D có dạng √ D = [a , a , ..., a , 2a ], (2.9) với a1, ...., as− 1 là đeu như nhau, và x/y được đưa ra bởi sự h®i tụ. x/y = [a0, a1, ..., as− 1]. (2.10) Xét h®i tụ pn/qn = [a0, a1, ..., an]. Neu an+1 = 2a0, thì (1.22) với x = √ D kéo theo | √ D pn — qn | ≤ 1 2a q2 , và tà h quả 1.1 (pn, qn) là nghi m của phương trình Pell p2 − Dq2 = ± 1. Ta thay ai < 2a0 khi i + 1 không là đ® dài của chu kì. Ta quy ước ước tính này đen ai ≤ a0. Giả sả an+1 ≥ a0 + 1. Tà đó chuoi (am)m≥1 là chu kì của chieu dài chu kì s0, với bat kì đong dư m đen n modul s0. Ta có am+1 > a0. Cho m ta có | √ D pm — qm | ≤ 1 (a + 1)q2 . Cho m đủ lớn đong dư với n modul s ta có (a0 + 1)q2 > q2 √ D + 1. H quả 1.1 kéo theo (pm, qm) là m®t nghi m của phương trình Pell m − Dqm = ± 1. Cuoi cùng định lí 2.1 kéo theo m+1 là b®i so của s0, do v y n+1 cũng the. Q p
  • 58. 50 Viết đề tài giá sinh viên – ZALO:0973.287.149-TEAMLUANVAN.COM pn pn− 1 . . qn− pp 1 n−1 n−1 . . qn . . qn− 1 . qn . q n q n q n . qn− 1 . . qn− 1 Q n n− 1 Chúng minh. 2.2.2. Xap xỉ Diophantine và phân so liên tnc đơn giản Bo đe 2.6. (Langrange, 1770) Chuoi (|qnx − pn|)n≥0 là giảm tuy t đoi, cho n ≥ 1 ta có: Chúng minh. |qnx − pn| < |qn− 1x − pn− 1|. Ta sả dụng bő đe 1.6 hai lan: M®t m t, 1 1 |qnx − pn| = vì xn+1 > 1. M t khác, xn+1.q n + qn− 1 < qn + q , n− 1 1 |qn− 1x − pn− 1| = x .q + qn− 2 1 > (an+1).qn− 1 + q n− 2 1 = , qn + qn− 1 vì xn < an + 1. Q H quả 2.4. Chuoi (|x − pn/qn|)n≥0 là giảm tuy t đoi với n ≥ 1 ta có: .x − . < .x − . . Cho n ≥ 1, tà khi qn− 1 < qn, ta có pn 1 x − = |q x − p 1 | < |q x − p | = .x − . < .x − . . Bo đe 2.7. Với n ≥ 0 và (p, q) ∈ Z × Z với q > 0 thỏa mãn |qx − p| < |qnx − pn|. Thì q ≥ qn+1. Chúng minh. H so của phương trình tuyen tính trong hai ȁn u, v pnu + pn+1v = p qnu + qn+1 v = q (2.11) . n n n− 1 n− 1
  • 59. 51 Viết đề tài giá sinh viên – ZALO:0973.287.149-TEAMLUANVAN.COM ǁ ǁ 1 p 1 . . . . . . .x − . < .x − ., nhat m®t trong chúng thỏa mãn x − < . . p pn có định thác ± 1, do v y m®t nghi m (u, v) ∈ Z × Z. Tà , ta có v /= 0. q q qn Neu u = 0 thì v = qn+1 > 0, do v y v ≥ 1 và q ≥ qn+1. Giả sả u.v /= 0. Tà q, qn và qn+1 là lớn hơn 0, nó không the cho u và v đeu âm. Trong trường hợp u, v dương ta được ket quả tà moi quan h thá hai của (2.11). Do v y, giả sả u và v của dau hi u đoi nhau tà qnx − pn và qn+1x − pn+1 cũng có dau hi u đoi nhau, so u(qnx − pn) và v(qn+1x − pn+1) có cùng dau hi u và do đó: |qnx − pn| ≤ |u(qnx − pn| + |v(qn+1x − pn+1| = |qx − p| < |qnx − pn|, là mâu thuan. Q H quả 2.5. Chuoi (qn)n≥0 của mau so h®i tụ của so thực vô t x là chuoi tăng dan của so nguyên dương mà: ǁ qnx ǁ<ǁ qx ǁ với 1 ≤ q ≤ qn. Ket quả là: qnx = min ≤q≤qn ǁ qx ǁ . Các lí thuyet của phân so liên tục được phát trien tà h quả 2.5 như m®t định nghĩa của chuoi (qn)n≥0). H quả 2.6. Cho n ≥ 0 và p q ∈ Q với q > 0 thỏa mãn thì q > qn. Chúng minh: Cho q ≤ qn, ta có: . q. . qn . .x − . = .|qx − p| > 1.|q n .x − pn qn pn pn | x − ≥ x − . . q. q q q . qn . . qn Q Bo đe 2.8. (Vahlen, 1895) Trong hai h®i tụ liên tiep pn qn và pn+1 qn+1 thì có ít p 1 . q. 2q2 p pn
  • 60. 52 Viết đề tài giá sinh viên – ZALO:0973.287.149-TEAMLUANVAN.COM − − pn pn− pp 1 1 n n−1 − n | . qn . . qn− 1 . .qn qn− 1 . 2q2 2.q2 n− 1 Chúng minh: Tà x pn qn và x pn− 1 qn− 1 có dau hi u ngược nhau, .x − . + .x − . = . − . = 1 1 < + . Bat đȁng thác cuoi cùng là ab < Do đó: a2 + b2 cho a 2 1 b với a = q n và b = 1 . qn− 1 ho c x p n qn 1 < 2.q2 |x − pn−1 | < 1 . qn− 1 2.q2 n− 1 Q Bo đe 2.9. (É. Borel, 1930) Trong ba h®i tụ liên tiep pn− 1 , pn và pn+1 . p. 1 qn− 1 qn qn+1 thì có ít nhat m®t trong chúng thỏa mãn .x − q. < √ 5q2 . Hang so √ 5 không tự thay the bởi m®t cái lớn hơn là cháng minh trong bő đe 2.13. Đieu này đúng cho bat kì so nào với khai trien phân so liên tục nhưng hǎu hạn phan thương so bang 1( nghĩa là so vùng Φ và tat cả so hǎu tỉ tương đương với modul ΦGL2(Z)). Chúng minh: Nhac lại bő đe 1.6. Cho n ≥ 0, (− 1)n qnx − pn = xn+1.q n . + qn− 1 Do đó |qn x− pn| < 1 √ 5.qn neu và chỉ neu |xn+1.q n +qn− 1 | > √ 5.qn . Theo định nghĩa rn = qn− 1 . Thì nó tương đương với x qn n+1 + rn | > √ 5. Nhac lại định nghĩa quy nạp của sự h®i tụ xn+1 = an+1 + x 1 . n+2 n qnqn− 1
  • 61. 53 Viết đề tài giá sinh viên – ZALO:0973.287.149-TEAMLUANVAN.COM − p Ngoài ra sả dụng định nghĩa của rn, rn+1 và quan h quy nạp qn+1 = an+1qn + qn− 1, ta có the viet 1 Loại bỏ an+1 : rn+1 1 + x r = an+1 + rn. 1 = xn+1 + rn. n+2 n+1 Giả sả |xn+1 + rn| ≤ √ 5 và |xn+2 + rn+1| ≤ √ 5. Ta suy ra: 1 1 √ + ≤ 1 + 1 = x + r ≤ √ 5. Ta có: 5 − r n+1 rn+1 xn+2 rn+1 n+1 n 2 n+1 — √ 5.rn+1 + 1 ≤ 0. Nghi m của đa thác X2 √ 5X + 1 là Φ = 1 + √ 5 2 và Φ− 1 = √ 5 − 1 . Do v y 2 rn+1 > Φ− 1 . Ước tính này sau giả thuyet 1 1 |qnx − pn| < √ 5.qn và |qn+1x − pn+1| < . 5.qn+1 Neu ta có |qn+2x − p n+2| < 1 √ 5.qn+2 , ta suy ra rn+2 > Φ− 1 , tà đó ta thay 1 = (an+2 + rn+1).rn+2 > (1 + Φ− 1 )Φ− 1 = 1, vô lí. Q p Bo đe 2.10. ( Legendre, 1798) Neu q p ∈ Q thỏa mãn .x — q. ≤ 1 2q2 , thì là m®t h®i tụ của x. q Chúng minh: Cho r và s trong Z thỏa mãn 1 ≤ s ≤ q. Tà 1 ≤ |qr − ps| = |s(qx− p)− q(sx− r)| ≤ s|qx− p|+q|sx− r| ≤ s 2q +q|sx− r|. r √
  • 62. 54 Viết đề tài giá sinh viên – ZALO:0973.287.149-TEAMLUANVAN.COM √ 0 0 0 Suy ra: s q|sx − r| ≥ 1 − 2q > 1 2 ≥ q|qx − p|. p Do v y |sx − r| > |qx − p| và do đó bő đe 2.7 kéo theo là m®t h®i tụ của q x. Q Ket lu n: Trong phan này tác giả đã trình bày lí thuyet ve phân so liên tục đơn giản, xap xỉ Diophantine và phân so liên tục đơn giản trong giải phương trình Pell. 2.3. Ve m t tiêu chuan cho sfi ton tại nghi m của phương trình Pell đây tác giả sě giới thi u ve m®t tiêu chuȁn cho sự ton tại nghi m của phương trình Pell. Cụ the hơn, đây là ket quả gan đây ve sự ton tại nghi m của phương trình Pell x2 − Dy2 = − 1. N®i dung chính được tham khảo trong tài li u [4]. M nh đe 2.1. ( R.A. Mollin, A Srinivasan) Cho D là so nguyên dương không chính phương. Cho (x0, y0) là nghi m cơ bản của phương trình Pell x2 − Dy2 = 1. Thì phương trình x2 − Dy2 = − 1 có 1 nghi m neu và chỉ neux0 ≡ 1(mod2D). Chúng minh. Neu a2 − Db2 = − 1 là nghi m cơ bản của x2 − Dy2 = − 1, thì x0 + y0 √ D = (a + b D)2 , do v y x0 = a2 + Db2 = 2Db2 = 2Db2 − 1 ≡ − 1(mod2D). Ngược lại neu x0 = 2Dk − 1 thì x2 = 4D2 k2 − 4Dk + 1 = Dy2 + 1. Do v y 4Dk2 − 4k = y2 . Do đó y0 là chȁn, y0 = 2z và k(Dk − 1) = z2 . Tà khik và Dk − 1 là nguyên to cùng nhau, cả hai đeu chính phương, k = b2 vàDk − 1 = a2 , ta được a2 − Db2 = − 1. Q
  • 63. 55 Viết đề tài giá sinh viên – ZALO:0973.287.149-TEAMLUANVAN.COM Tác giả đã giới thi u m®t tiêu chuȁn cho sự ton tại nghi m của phương Pell cơ bản. Tà đó xác định được đieu ki n có nghi m của phương trình. 2.4. M t so m r ng của xap xỉ Diophantine Trong phan này tác giả giới thi u m®t so mở r®ng của xap xỉ Diophantine, tà đó có the áng dụng đe giải phương trình Pell nói riêng và phương trình nghi m nguyên nói chung. N®i dung tham khảo trong tài li u [4]. 2.4.1. Tiêu chí vô t M nh đe 2.2. Cho ϑ là so thực, các đieu ki n sau tương đương: (i) ϑ là so vô tỉ. (ii) Cho bat kì ε > 0, ton tại (p, q) ∈ Z2 sao cho q > 0 và 0 < |qϑ − p| < ε. (iii) Cho bat kì ε > 0, ton tai hai tuyen tính dạng tuyen tính đ®c l p trong hai bien L0(X0, X1) = a0X0 + b0X1 và L1(X0, X1) = a1X0 + b1X1 với h so nguyên sao cho max{|L0(1, ϑ)|, |L1(1, ϑ)|} < ε. (iv) Cho so bat kì Q > 1 ton tại so nguyên q trong khoảng 1 ≤ q ≤ Q và so nguyên p sao cho 1 0 < |qϑ − p| < Q . (v) Ton tại vô so p/q ∈ Q sao cho: p |ϑ − q | < (vi) Ton tại vô so p/q ∈ Q sao cho: p 1 q2 . 1 |ϑ − q | < √ 5q2 . Hàm ý (vi) ⇒ (v) là tam thường. Ta sě cháng minh (i) ⇒ (vi) sau (trong phan trên phân so liên tục). Bây giờ ta sě cháng minh sự tương đương giǎa
  • 64. 56 Viết đề tài giá sinh viên – ZALO:0973.287.149-TEAMLUANVAN.COM i i các đieu ki n của m nh đe 2.2 như sau: (iv) ⇒ (ii) ⇒ (iii) ⇒ (i) ⇒ (iv) ⇒ (v) ⇒ (ii). Neu đưa ra m®t so nguyên dương q, có ít nhat m®t giá trị p sao cho |qϑ− p| < , cụ the so nguyên gan nhat đen qϑ. Do v y, khi ta xap xỉ ϑ bởi so hǎu t 2 p/q, ta có m®t tham so trong Z > 0, cụ the là q. Trong đieu ki n (v), không can giả định ve trái khác 0, neu m®t p/q ∈ Q cho ket quả 0, thì tat cả các so khác là không và m®t lan nǎa có vô so chúng. 1 Chúng minh của (iv) ⇒ (ii). Sả dụng (iv) với Q thỏa mãn Q > 1 và Q > ε ta thu được (ii). Q Chúng minh của (v) ⇒ (ii). Theo (v) có dãy vô hạn của các so hǎu t riêng bi t (pi/qi)i≥0 với qi > 0 sao cho: pi |ϑ − q | < 1 √ 5q2 . Với moi qi, có duy nhat giá trị cho tả so pi mà bat đȁng thác là thỏa mãn. 1 Do v y t p của qi là vô t n. Lay qi ≥ ε suy ra (ii). Q Chúng minh của (ii) ⇒ (iii). Cho ε > 0, tà (ii) ta suy ra sự ton tại của (p, q) ∈ Z × Z với q > 0 và gcd(p, q) = 1 sao cho: 0 < |qϑ − p| < ε. Ta sả dụng (ii) m®t lan nǎa với ε thay the bởi |qϑ− p|. Ton tại (p′ , q′ ) ∈ Z× Z với q′ > 0 sao cho: 0 < |q′ ϑ − p′ | < |qϑ − p|. (2.12) Xác định L0(X0, X1) = pX0 − qX1 và L1(X0, X1) = p′ X0 − q′ X1. Nó kiem tra L0(X0, X1) và L1(X0, X1) là đ®c l p tuyen tính. Cách khác, ton tại (s, t) ∈ Z2 (0, 0) sao cho sL0 = tL1. Do v y sp = tp′ , sq = tq′ và p/q = p′ /q′ . Tà khi gcd(p, q) = 1, ta suy ra t = 1, p′ = sp, q′ = sq và q′ ϑ − p′ = s(qϑ − p). Đieu này không tương đương với (2.12). Q 1
  • 65. 57 Viết đề tài giá sinh viên – ZALO:0973.287.149-TEAMLUANVAN.COM Chúng minh của (ii) ⇒ (i). Giả sả ϑ ∈ Q, có ϑ = a với gcd(a, b) = 1 và b b > 0. Cho bat kì dạng tuyen tính khác không L ∈ ZX0 + ZX1 , đieu ki n L(1, ϑ) /= 0 kéo theo |L(1, ϑ)| ≥ 1 . Do v y với ε = b 1 đieu ki n (iii) không b có. Q Chúng minh của (i) ⇒ (iv) sả dụng nguyên lí h®p Dirichlet. Cho Q > 1 đưa ra so thực. Xác định N = [Q| nghĩa là N là so nguyên sao cho N − 1 < Q ≤ N . Tà Q > 1 ta có N ≥ 2. Cho ϑ ∈ R Q Xét các t p con E của đơn vị khoảng [0, 1] và xét phan tả N + 1 0, {ϑ}, {2ϑ}, {3ϑ}, ..., {(N − 1)ϑ}, 1. Tà ϑ là so vô tỉ, phan tả N + 1 là đôi m®t khác nhau. Chia khoảng của N phan tả, j Ij = [ N ; j + 2 N ] (0 ≤ j ≤ N − 1). Ít nhat m®t trong nhǎng khoảng cách N, có Ij0 , cháa ít nhat hai phan tả của E: Phan tả 0 và 1, tat cả phan tả qϑ trong E với 1 ≤ q ≤ N − 1 là sovô tỉ, do v y thu®c ve sự ket hợp của các khoảng mở (j/N, (j + 1)/N) với0 ≤ j ≤ N − 1. Neu j0 = N − 1 thì khoảng cách: 1 Ij0 = IN− 1 = [1 − N , 1] cháa 1 cũng như m®t phan tả của E có dạng {qϑ} với 1 ≤ q ≤ N − 1. T p p = [qϑ♩ + 1. Thì ta có 1 ≤ q ≤ N − 1 < Q và p − qϑ = [qϑ♩ + 1 − [qϑ♩ − {qϑ} = 1 − {qϑ}, do đó 1 1 0 < p − qϑ < N ≤ Q . Cách khác ta có 0 ≤ j0 ≤ N − 2 và Ij0 cháa hai phan tả {q1ϑ} và {q2ϑ} với 0 ≤ q1 ≤ q2 ≤ N − 1. T p q = q2 − q1, q = [q2ϑ♩ − [q1ϑ♩ .
  • 66. 58 Viết đề tài giá sinh viên – ZALO:0973.287.149-TEAMLUANVAN.COM a∈ Z Thì ta có 0 < q = q2 − q1 ≤ N − 1 ≤ Q và |qϑ − q| = |{q2ϑ} − {q1ϑ}| < 1/N ≤ 1/Q. Q Nh n xét 2.2. Với bat kì so thực ϑ, cho bat kì so thực Q > 1 ton tại so nguyên q trong khoảng 1 ≤ q < Q và so nguyên p sao cho p 1 |ϑ − q | ≤ qQ . Các cháng minh đưa đen bat đȁng thác ng t |qϑ − p| ≤ 1/Q trong trường hợp Q không là so nguyên, trong trường hợp khi Q là so nguyên và ϑ hǎutỉ, ket quả không đúng với bat đȁng thác nói chung. Ví dụ neu ϑ = a/b với gcd(a, b) = 1 và b ≥ 2, m®t nghi m p/q với bat đȁng thác ng t cho Q = b + 1 nhưng không cho Q = b. Tuy nhiên Q là so nguyên và ϑ là so vô tỉ, so |qϑ − p| là vô tỉ ( do q > 0) do v y không bang 1/Q. Chúng minh của (iv) ⇒ (v). Giả sả (iv). Ta đã biet rang (iv) ⇒ (i). Do v yϑ là so vô tỉ. Cho {q1, ..., qN } là t p hǎu hạn các so nguyên dương. Ta sě bieu dien ton tại so nguyên dương q không thu®c {q1, ..., qN } thỏa mãn đieu ki n (v). Bieu dien bởi ||.|| khoảng cách đen so nguyên gan nhat: Cho x ∈ R ||x|| = min |x − a|. Tà ϑ là so vô tỉ, cho 1 ≤ j ≤ N, so ||qjϑ|| là so khác không. Cho Q > 1 thỏa mãn Q > min 1≤j≤n − 1 ||qjϑ|| . Tà (iv) suy ra ton tại so nguyên q trong khoảng 1 ≤ q ≤ Q sao cho 1 0 < ||qϑi|| ≤ Q . Phía bên ve phải là < 1/q và sự lựa chon của Q có kéo theo q không thu®c {q1, ..., qN }. Q
  • 67. 59 Viết đề tài giá sinh viên – ZALO:0973.287.149-TEAMLUANVAN.COM Y 2.4.2. Bat đang thfíc Liouville Bo đe 2.11. Cho α là so đại so của b c d ≥ 2 và đa thác toi thieuP ∈ Z[X]. Xác đinh c = |P ′ (α)|. Cho ε > 0 thì ton tại so nguyên q0 sao cho, bat kì p/q ∈ Q với q > q0, Chúng minh. p |α − q | ≥ 1 (c + ε)qd . Ket quả là tam thường neu α không là so thực. M®t giá trị có the chap nh n cho q0 là q0 = (c|=m(α)|)− 1/d . Giả sả α là so thực. Cho q là so nguyên dương đủ lớn và p là so nguyên gan nhat đen qα. Đ c bi t: p 1 |α − q | ≤ 2q . Bieu dien bởi a0 h so hàng đau của p và bởi α1, ..., αd nghi m với α1 = α. Do v y P(X) = a0(X − α1)(X − α2)...(X − αd), và d qd P(p/q) = a qd Y ( p − α ). (2.13) Cũng như 0 i i=1 d P′ (α) = a0 (α − αi). i=2 Ve bên trái của (2.13) là so nguyên. Nó khác không vì P không rút gon b c ≥ 2. Cho i ≥ 2 ta sả dụng các ước tính p 1 |αi − q | ≤ |αi − α| + 2q .
  • 68. 60 Viết đề tài giá sinh viên – ZALO:0973.287.149-TEAMLUANVAN.COM n q Ta suy ra : 1 ≤ qd a0|α p — q | Y i=2 |(αi 1 — α)| + 2q . Đoi với q đủ lớn bên ve phải bị ch n tà qd |α − p |(|P′ (α)| + ε). Q 2.4.3. Bat đang thfíc Liouville b c hai Xét bő đe 2.11 trong trường hợp đ c bi t d = 2 khi α là so đại so b c hai. Viet đa thác toi thieu f(X) = aX2 + bX + c và cho ∆ := b2 − 4ac là bi t thác. √ Ta quan tâm đen xap xỉ của α bởi√ so hǎu tỉ, giả sả ∆ > 0. Neu α = − b ± ∆ 2a thì nghi m khác là α′ = −b ∓ ∆ và 2a f′ (α) = a(α − α′ ) = ± √ ∆. Bo đe 2.12. Cho α là đại so của b c 2 và đa thác toi thieu P ∈ Z[X]. Xác định c = |P′ (α)|. Cho ε > 0 thì ton tại so nguyên q0 sao cho bat kì p/q ∈ Q với q ≥ q0, p |α − q | ≥ 1 ( √ ∆ + ε)q2 . Bi t thác dương nhỏ nhat của đa thác b c hai không rút gon với h so trong Z là 5, mà giá trị của bi t thác X2 − X − 1, với nghi m Φ và − Φ− 1 khi Φ = 1, 6180339887499... bieu thị tỉ l Gold. Ket quả tiep theo với dãy Fibonacci (Fn)n≥0: F0 = 0, F1 = 1, Fn = Fn− 1 + Fn− 2 (n ≥ 2). Bo đe 2.13. Cho bat kì q ≥ 1 và bat kì p ∈ Z, p |Φ − q | > 1 √ 5q2 + q . 2
  • 69. 61 Viết đề tài giá sinh viên – ZALO:0973.287.149-TEAMLUANVAN.COM n− 1 q n− 1 n | M t khác, Chúng minh. lim n→∞ F2 |Φ − Fn = Fn− 1 1 √ 5 . Nó đủ đe cháng minh sự bị ch n thap hơn khi p là so nguyên gan nhat qΦ. Tà X2 − X − 1 = (X − Φ)(X + Φ− 1 ) ta suy ra p2 − pq − q2 = q2 ( p − p Φ)( q + Φ− 1 ). Ve trái là so nguyên khác không, do v y có giá trị tuy t đoi nhỏ nhat là 1 1. Ta ràng bu®c giá trị tuy t đoi của ve phải tà trên. Tà p < qΦ + và Φ + Φ− 1 = √ 5 ta có: 2 p + Φ− 1 < √ 5 + 1 . q 2q Phan đau tiên của bő đe 2.13 như sau: Không gian véctơ thực của dãy (vn)n≥0 thoản mãn vn = vn− 1 + vn− 2 có hai chieu, m®t cơ sở được cho bởi hai chuoi (Φn )n≥0 và ((− Φ− 1 )n )n≥0. Tà cách này de dàng suy ra công thác: 1 n n − n Fn = √ 5 (Φ − (−1) Φ ). Theo A.De.Moivre (1730), L.Euler (1765) và J.P.M.Binet (1843), Fn là so nguyên gan nhat đen 1 n √ 5 Φ . Do v y, chuoi (un)n≥2 của thương so Fibonacci un = Fn/Fn− 1, thoả mãn lim n→∞ un = Φ. De dàng kiem tra F 2 − FnFn− 1 − F 2 = (− 1)n− 1 , n chon n ≥ 1. Ve trái là F2 n− 1 (un − Φ)(un + Φ− 1 ), như ta đã thay. Do v y F2 n− 1|Φ − un 1 | = Φ− 1 + u ,
  • 70. 62 Viết đề tài giá sinh viên – ZALO:0973.287.149-TEAMLUANVAN.COM 1 và giới hạn của ve phải là 1 = Φ + Φ− 1 √ 5 . Q Nh¾n xét 2.3. Chuoi un = Fn/Fn− 1 cũng được xác định bởi 1 Do v y, u2 = 2, un = 1 + u 1 n− 1 (n ≥ 3). 1 un = 1 + 1 1 + un− 2 = 1 + 1 + 1 1 1 + 1 = ... un− 3 Nh¾n xét 2.4. Nó được biet đen rang neu k là so nguyên dương, neu so thực vô tỉ ϑ có dạng khai trien phân so liên tục [a0; a1, a2, ...] với an ≥ k đoi với vô so n thì lim inf q q→∞ |ϑ − p q | ≤ 1 √ 4 + k2 . Ket lu n: Tác giả đã trình bày m®t so mở r®ng của xap xỉ Diophantine đoi với so b c hai. Tà đó áp dụng ket quả đe giải các phương trình Pell. 2.5. M t fíng dnng giải phương trình Pell âm Trong phan này tác giả sě giới thi u m®t áng dụng giải phương trình Pell âm. Trong đó, vai trò của x và y đã được đői cho cho nhau. N®i dung được tham khảo trong tài li u [5]. De thay phương trình y2 = 3x2 − 1, y2 = 7x2 − 4 không có nghi m nguyên trong khi y2 = 65x2 − 1, y2 = 202x2 − 1 có nghi m nguyên. Đ c bi t hơn, m®t cách cho giá trị của D đoi với phương trình Pell âm y2 − Dx2 = − 1là giải được hay không. Trong phan này, phương trình Pell âm đưa ra bởi y2 = 45x2 − 11 là xem xét và vô so nghi m nguyên thu được. M®t so quan h thú vị trong so các giải pháp được hi n di n. Phương pháp phân tích: 2
  • 71. 63 Viết đề tài giá sinh viên – ZALO:0973.287.149-TEAMLUANVAN.COM n+1 n n n+1 n n Phương trình Pell âm miêu tả hypebon dưới sự xem xét là: y2 = 45x2 − 11. (2.14) Cho nghi m nguyên dương nhỏ nhat là x0 = 2, y0 = 13 . Các nghi m khác thu được của (2.14), xét phương trình Pell y2 = 45x2 + 1 cho nghi m tőng ∼ ∼ ∼ 1 ∼ 1 √ quát (x, y) được đưa ra bởi x = 2 45 gn, y = 2 fn. Nơi mà fn = (161 + 24 √ 45)n+1 + (161 − 24 √ 45)n+1 ; (2.15) gn = (161 + 24 √ 45)n+1 − (161 − 24 √ 45)n+1 . (2.16) Đ t nam giǎa (x0, y0) và ( ∼ ∼ ), nghi m nguyên khác của (2.14) được đưa ra bởi: x0, y0 90x = 90f + 13 √ 45g (2.17) 2y = 13f + 2 √ 45g (2.18) M®t so ví dụ của y thỏa mãn (2.14) đưa ra trong bảng sau n xn yn 0 2 13 1 634 4253 2 204146 1369453 3 65734378 440959613 4 21166265570 141987625933 Lời nh¾n xét: Tà bảng trên, ta quan sát m®t so quan h thú vị trong các nghi m khi được xuat hi n dưới đây: 1) Giá trị x là chȁn khi giá trị y là lẻ. 2) Moi quan h phép truy toán thỏa mãn bởi nghi m của (2.14) được đưa ra bởi: xn+3 − 322xn+2 + xn+1 = 0; (2.19)
  • 72. 64 Viết đề tài giá sinh viên – ZALO:0973.287.149-TEAMLUANVAN.COM n+2 n+1 n yn+3 − 322yn+2 + yn+1 = 0. (2.20) 3) xn+3 = 322xn+2 − xn+1. 4) 24yn+1 = xn+2 − 161xn+1. 5)24yn+2 = 161xn+2 − xn+1. 6) 24yn+3 = 51841xn+2 − 161xn+1. 7) 51841xn+1 = xn+3 − 7728yn+1. 8) 25920xn+2 = 3864yn+2 − 24yn+1. 9) 25920xn+2 = 24yn+3 − 3864yn+2. 10) xn+2 = 161xn+3 − 24yn+3. 11) 51840xn+2 = 24yn+3 − 24yn+1. 12) 161xn+2 = xn+3 − 24yn+2. 13) 51841xn+2 = 161xn+3 − 24yn+1. 14) 25920xn+1 = 24yn+2 − 3864yn+1. 15) 51841x2 16) 51841x2 = 161xn+3xn+2 − 24yn+1xn+2. = xn+1xn+3 − 7728xn+1yn+1. 17) 24yn+1xn+1 = 3864yn+2xn+1 − 25920xn+1xn+2. 18) 3864yn+1xn+2 = 24yn+2xn+2 − 25920xn+1xn+2. 19) 180x2n+2 − 26y2n+2 + 22 11 là m®t chính phương hoàn toàn. Chúng minh: Loại bỏ gn giǎa (2.17) và (2.18), ta được: 180xn+1 − 26yn+1 = 11fn. (2.21) Tương tự bỏ fn giǎa (2.17) và (2.18), ta được: 4 √ 45yn+1 − 26 √ 45xn+1 = 11gn. (2.22) Thay the n bởi 2n + 1 trong (2.21) ,nó trở thành 180x2n+2 − 26y2n+2 = 11[f2 − 2]. Hơn nǎa, 180x2n+2 − 26y2n+2 + 22 11 là so chính phương hoàn chỉnh. Tương tự, moi ví dụ sau là m®t so chính phương hoàn chỉnh:
  • 73. 65 Viết đề tài giá sinh viên – ZALO:0973.287.149-TEAMLUANVAN.COM n n i) 1369453x2n+2 − 13x2n+4 + 85008 . 42504 ii) 1369453x2n+3 − 4253x2n+4 + 264 . 132 iii) 57060x2n+2 − 26y2n+3 + 3542 . 1771 iv) 18373140x2n+2 − 26y2n+4 + 1140502 . 570251 v) 4253x2n+2 − 13x2n+3 + 264 . 132 vi) 180x2n+3 − 8506y2n+2 + 3542 . 1771 viii) 57060x2n+3 − 8506y2n+3 + 22 . 11 20) 180x3n+3 − 26y3n+3 + 10626 11 là so nguyên b c ba. Chúng minh: Thay the n bởi 3n + 2 trong (2.21), nó trở thành 180x3n+3 − 26y3n+3 + 10626 11 Nh n xét quan trong: I: Nó trở thành là so nguyên b c ba. f2 − g2 = 4. (2.23) Xác định (X = 180xn+1 − 26yn+1, Y = 4 √ 45yn+1 − 26 √ 45xn+1). Nên fn = X Y 11 , gn = 11 . Thay the các giá trị trên của (fn, gn) trong (2.23), ta có X2 − Y 2 = 484 khi bieu dien m®t hypebon. Tương tự, sả dụng ket hợp tuyen tính trong so các nghi m của (2.14), m®t trong nhǎng nghi m nguyên tőng quát cho sự lựa chon khác của hypebon được bieu dien trong bảng 1 sau: Bảng 1
  • 74. 66 Viết đề tài giá sinh viên – ZALO:0973.287.149-TEAMLUANVAN.COM II: Xác định (X = 180x2n+2 − 26y2n+2 + 22, Y = 4 √ 45yn+1 − 26 √ 45xn+1). Do đó 2 X 2 Y 2 fn = 11 , gn = 112 . S.NO HYPEBON (X, Y ) 1 X2 − Y 2 = 7226360064 (1369453x − 13x , 90xn+3 − 9186570xn+1 ) n+1 n+3 √ 45 2 X2 − Y 2 = 484 180yn+1 − 1170xn+1 (180xn+1 − 26yn+1, √ 45 ) 3 X2 − Y 2 = 69696 28530xn+3 − 9186570xn+2 (1369453xn+2 − 4253xn+3, √ 45 ) 4 X2 − Y 2 = 1300744812004 (18373140x − 26y , 180yn+3 − 123250770xn+1 ) n+1 n+3 √ 45 5 X2 − Y 2 = 12545764 (57060x − 26y , 180yn+3 − 382770xn+1 ) n+1 n+2 √ 45 6 X2 − Y 2 = 69696 (4253x − 13x , 90xn+2 − 28530xn+1 ) n+1 n+2 √ 45 7 X2 − Y 2 = 12545764 57060yn+1 − 1170xn+2 (180xn+2 − 8506yn+1, √ 45 ) 8 X2 − Y 2 = 484 57060yn+2 − 382770xn+2 (57060xn+2 − 8506yn+2, √ 45 )
  • 75. 67 Viết đề tài giá sinh viên – ZALO:0973.287.149-TEAMLUANVAN.COM Thay the các giá trị trên của (f2 , g2 ) trong (2.23), ta có n n khi bieu dien m®t parabol. Y 2 = 11X − 484 Tương tự, sả dụng ket hợp tuyen tính trong so các nghi m của (2.14), m®t trong nghi m nguyên tőng quát cho sự lựa chon khác của parabon được bieu dien trong bảng 2 sau:
  • 76. 68 Viết đề tài giá sinh viên – ZALO:0973.287.149-TEAMLUANVAN.COM S.NO PARABOL (X, Y ) 1 Y 2 = 42504X − 7226360064 (1369453x− 13x + 85008, 90xn+3 − 9186570xn+1 ) 2n+2 2n+4 √ 45 2 Y 2 = 11X − 484 180yn+1 − 1170xn+1 (180x2n+2 − 26y2n+2 + 22, √ 45 ) 3 Y 2 = 132X − 69696 28503xn+3 − 9186570xn+2 (1369453x2n+3 − 4253x2n+4 + 264, √ 45 ) 4 Y 2 = 570251X − 1300744812004 (18373140x− 26y + 1140502, 180yn+3 − 123250770xn+1 ) 2n+2 2n+4 √ 45 5 Y 2 = 1771X − 12545764 (57060x− 26y + 3542, 180yn+3 − 382770xn+1 ) 2n+2 2n+3 √ 45 6 Y 2 = 132X − 69696 (4253x− 13x + 264, 90xn+2 − 28530xn+1 ) 2n+2 2n+3 √ 45 7 Y 2 = 1771X − 1254564 57060yn+1 − 1170xn+2 (180x2n+3 − 8506y2n+2 + 3542, √ 45 ) 8 Y 2 = 11X − 484 57060yn+2 − 382770xn+2 (57060x2n+3 − 8506y2n+3 + 22, √ 45 ) III: Cho p, q với p > q > 0 là phan tả sinh của b® ba Pytago T(α, β, γ) , khi α = 2pq, β = p2 − q2 , γ = p2 + q2 , p > q > 0 . Cho A, P lan lượt bieu dien